Test Questions

Lakukan tugas rumah & ujian kamu dengan baik sekarang menggunakan Quizwiz!

Which of the following swallow maneuvers has been found to produce cardiac arrhythmia (irregular heartbeat) in patients with stroke or cardiovascular diseases?

Supraglottic swallow maneuver, Chaudhuri et al. (2002) found that 86 percent of people with history of stroke and/or cardiovascular disease developed cardiac arrhythmias while performing the supraglottic or super-supraglottic swallow, including supraventricular tachycardia and premature atrial and/or ventricular contractions.

Impedance Bridge

objective assessment method of the function of the middle ear. By increasing acoustic pressure in the external ear canal, impedance audiometry measures the sum of resistance which a sound wave encounters on its way through the middle ear to the cochlear receptor.

Paralinguistics

paralanguage refers to the non-verbal elements of communication used to modify meaning and convey emotion. Nonverbal elements that modify our speech: intonation, stress, emphasis, rate of delivery, pause or hesitation

The components of an AAC system are crucial to consider during an assessment. When assessing a young child, you evaluate the way the individual will use and interact with the system, including the user interface, selection method, and output. What type of components are being described?

Secondary components

A patient with a Glasgow Coma Scale score of 5 indicates what? Mild traumatic brain injury. Normal brain functioning. Severe traumatic brain injury. Malingering.

Severe traumatic brain injury. Explanation: Neurological features associated with TBI are standardized using the Glasgow Coma Scale. This test is scored from 1 to 15 points. Each of three measures (eye opening, best verbal response, and best motor response) is scored separately, and the combined score helps determine the severity of TBI. A total score of 3 to 8 reflects a severe TBI, 9 to 12 a moderate TBI, and 13 to 15 a mild TBI.

Athetoid cerebral palsy

slow, arrhythmic, writhing, involuntary movements of the extremities

Vowels are produced with a relatively open vocal tract. There will always be a fundamental and harmonics for vowels. We perceive vowel differences by the relationship between formants. How are vowels perceived?

Spacing of the formants

Which of the following is the most common form of mixed cerebral palsy?

Spastic-dyskinetic

Jon is a typically developing 4-year old preschooler. Which of the following structures would most likely be in his language repertoire?

Suggesting intention, emerging mastery of copula to be, identifies the first sound in a word

social influence

the ability to control another person's behavior

Positive Punishment

the administration of a stimulus to decrease the probability of a behavior's recurring Yelling "No!" at a dog jumping up on a person (adds scold to reduce behavior) Spanking a child Swatting a dog with a newspaper for peeing on the carpet.

The uses of language with the intent to persuade or change an opinion is typically seen in children over

the age of 5

social integration (social cohesion)

the degree to which members of a group or a society are united by shared values and other social bonds

F2

the most prominent band in the mouth, change based on oral cavity

social interaction

the process by which people act and react in relation to others

During production of vowels, the relative position of the tongue along the bottom-top dimension is expressed acoustically in: The frequency of the second formant (F2). The duration of the silent interval. The frequency of the first formant.(F1). The relative spacing of the harmonics.

The frequency of the first formant.(F1).

During production of vowels, the relative position of the tongue along the back-front dimension is expressed acoustically by: Whether or not a standing wave is produced. The frequency of the second formant (F2). The relative spacing of the harmonics. The frequency of the first formant (F1).

The frequency of the second formant (F2).

Van Riper's traditional approach

this approach is the foundation for motor approaches to articulation therapy. Focuses on auditory discrimination/perceptual training, phonetic placement, and drill like repetition and practice at increasingly complex motor levels until target phonemes were automatized

Top of Form After sustaining a CVA, Ms. Williams, age 75, was referred to an SLP for a speech and language evaluation. While Ms. Williams was describing the cookie-theft picture, the SLP observed that her grammatical structure appeared to be intact and her prosody was normal but that many of her sentences were meaningless, did not fit the context, and included nonsensical paraphasic errors. Additional testing also revealed that Ms. Williams exhibited poor repetition and naming skills, did not respond appropriately to many simple commands, and had difficulty reading. Ms. Williams appeared happy and talked excessively. She did not appear to be aware of her communication deficits. What is the most likely location of the lesion? A.Left posterior superior temporal gyrus B.Left inferior frontal gyrus C.Left superior frontal gyrus D.Left inferior parietal gyrus

Option (A) is correct. The symptoms that are described in the scenario are consistent with damage to the left posterior superior temporal gyrus.

Which of the following, if observed in the speech of an African American child, is most likely to represent a dialectical variation rather than an articulation error? A./f/ for /θ/ in postvocalic position B./θ/ for /s/ in all positions C.Affricates for fricatives in word-final position D.Dentals for velars in word-initial position

Option (A) is correct. The use of voiceless labiodental fricatives for voiceless interdental fricatives is a feature of African American Vernacular English (AAVE).

86. Top of Form Native speakers of a language possess several capacities or abilities that provide insight into their language competence. Sentences such as "Visiting friends can be a nuisance" are especially useful to test a person's ability to A.recognize syntactic ambiguity B.interpret metaphoric language C.distinguish homophones by means of syntactic cues D.recognize a semantically anomalous sentence

Option (A) is correct. There are two ways to parse or interpret the structure of the sentence "Visiting friends can be a nuisance." One interpretation is that it is bothersome to visit friends; the other is that friends who have come for a visit are bothersome. Therefore, this sentence would be appropriate as part of a test of a person's ability to recognize syntactic ambiguity.

109. Top of Form Language impairment in a child with Down syndrome is often determined by comparing performance on one or more standardized language tests with the child's mental age, rather than with the child's chronological age. Although mental age should not be used to specify the need for treatment, mental age can legitimately be used as a performance criterion because A.using chronological age would overidentify language disorders B.using chronological age would underidentify language disorders C.mental age always correlates with verbal performance D.language performance is expected to exceed mental age

Option (A) is correct. Using chronological age as a performance criterion for a child with Down syndrome would make the child's language disorders seem more pronounced than they are.

6. Top of Form A client exhibits weakness, atrophy, and fasciculations of the right side of the tongue and lower face. The client also has right vocal-fold weakness and nasal regurgitation of fluid when swallowing. These problems are the result of damage to which part of the nervous system? A.Brain stem B.Cerebellum C.Left cerebral cortex D.Right cerebral cortex

Option (A) is correct. Weakness, atrophy, fasciculations, and the other described symptoms are all consistent with a lower motor neuron locus and suspected cranial nerve abnormalities (primarily CN X and XII). These cranial nerves emerge directly from the brain stem and help mediate the transfer of messages from the brain to the brain stem and to the structures of the head and neck.

110. Top of Form Which of the following is the ratio of reinforcement that will most quickly cause a newly acquired behavior to be habituated? A.A random ratio of tokens to correct responses B.A ratio of 1 token to 1 correct response C.A ratio of 1 token to 4 correct responses only D.A ratio of 1 token to 10 correct responses only

Option (A) is correct. When the goal is to reinforce a behavior that has already been acquired, a random ratio of tokens to correct responses creates an intermittent reinforcement schedule and is the most effective. Such a reinforcement schedule decreases the client's dependence on the token reward.

84. Top of Form Which of the following provides the most important diagnostic information to an SLP making a differential diagnosis between childhood apraxia of speech and flaccid dysarthria in a child? A.The child's articulation performance at the sentence level B.A history of the child's development of chewing, eating, and swallowing C.A history of the child's language development D.The child's willingness to function in sociocommunicative events

Option (B) is correct. A child with childhood apraxia of speech does not have difficulties with chewing, eating, and swallowing, whereas a child with flaccid dysarthria is likely to have such difficulties.

An SLPS L P designs a study to examine the effect of a new therapy technique in comparison to the effect of the current standard approach. Patients are recruited and randomly assigned to either a control group or an experimental group. The outcomes of each group will then be compared. Which of the following best describes the SLP'sS L P's research project? • A.Cohort study • B.Controlled trial • C.Cross-sectional study • D.Case-Control study

Option (B) is correct. A controlled trial is the only study design option that involves experimental manipulation; one group would be provided a treatment or intervention, and the other would receive a placebo, a sham treatment, or some other form of control.

120. Top of Form Establishment of which of the following is most important in ensuring that the results of any diagnostic test of speech or language are replicable? A.Content validity B.Interjudge reliability C.Split-half reliability D.Face validity

Option (B) is correct. A test that has interjudge reliability is one whose results are replicable, even if different people administer the test.Top of Form

An SLPS L P just completed an aphasia evaluation with an aphasia battery. The results of the battery show that the patient with aphasia produces very fluent speech but makes many phonemic errors. When producing phonemic errors, the patient stops to try and get the word right with multiple attempts (e.g.for example., for the word screwdriver, the patient said "Cewdriver, dewdiver, screwdriver."). The patient also had relatively good comprehension but a pronounced repetition impairment. The patient's results are most consistent with which of the following type of aphasia? A.Broca's aphasia B.Conduction aphasia C.Wernicke's aphasia D.Anomic aphasia

Option (B) is correct. All the symptoms listed are classic symptoms of conduction aphasia and are distinguishable from other aphasia types.

Cognitive therapy for stuttering focuses on which of the following? A.Extinguishing the overt, dysfluent speech behavior by withholding positive reinforcement B.Changing distorted beliefs about self-efficacy and the need to speak with complete fluency C.Providing positive reinforcement during periods of fluent speech D.Reducing dysfluent speech behavior by using visual imaging

Option (B) is correct. Changing distorted beliefs about self-efficacy and the need to speak with complete fluency represents the only answer choice that is cognitive in nature.

93. Top of Form Which of the following errors is likely to persist the longest in the speech of children who are learning Standard American English (SAE) as a first language and are following the normal developmental course for speech and language acquisition? A.Assimilation B.Consonant cluster reduction C.Final-consonant deletion D.Velar fronting

Option (B) is correct. Consonant cluster reduction is the most persistent of the normal developmental processes listed.

A 9-year-old child is one year post tonsillectomy and adenoidectomy and presents with moderate hypernasality and consistent nasal emission. The child was referred for assessment, and velopharyngeal incompetency was identified. Which of the following would be the next appropriate step for the SLPS L P to take? A.Continue to monitor resonance as the child still may spontaneously improve B.Discuss surgical management options with the craniofacial team C.Initiate therapy to discriminate between hypernasal speech and oral speech D.Slow down rate of speech to help velopharyngeal closure

Option (B) is correct. Nasopharyngoscopy will be conducted to visualize the velopharyngeal mechanism and provide information for possible surgical management.

During videofluoroscopic evaluation of an adult patient, the SLPS L P notices material in both pyriform sinuses after the patient swallows. Upon review, the SLPS L P sees that both pyriform sinuses clear during the pharyngeal swallow but then refill quickly as the structures return to rest. Which of the following statements best explains this observation? • A.Pharyngeal stripping was not adequate to fully clear the pyriform sinuses, so there was material in the pyriform sinuses after each swallow. • B.The patient had small pharyngeal pouches (pharyngoceles) that filled with barium during the swallow and then emptied into the pyriform sinuses after each swallow. • C.Hyolaryngeal elevation was not sufficient, leaving material in the pyriform sinuses after each swallow. • D.The patient had a tracheoesophageal fistula, and contrast was deposited in the pyriform sinuses after each swallow.

Option (B) is correct. Pharyngoceles emptying into the sinuses after each swallow could be diagnosed by analyzing slow-motion videofluoroscopic images.

33. Top of Form Language intervention for a child at the one-word stage should be most strongly influenced by a consideration of the child's A.motor skills B.cognitive skills C.syntactic skills D.articulation skills

Option (B) is correct. The cognitive skills of a child at the one-word stage will most strongly influence the child's speech-language responses, so language intervention for the child should take into account the child's cognitive skills.

During MsMiss. Malone's assessment, the SLPS L P determines that she could potentially benefit from AACA A C services and initiates discussion of intervention goals. Once goals are established and MsMiss. Malone has her device, she begins to learn how to operate her device to produce common spoken messages. Over the course of intervention, MsMiss. Malone is ultimately able to produce novel sentences. Answer the question below by clicking on the correct response. Question: Regarding MsMiss. Malone's proposed AACA A C assessment, which of the following statements is true? A.Her choice of devices will be severely limited by her difficulties with speech onset. B.Her family should be involved in the AACA A C assessment as well as any subsequent therapy. C.Given her advanced level of education, only high-tech devices should be considered. D.Given the severity of her impairment, only low-tech devices should be considered.

Option (B) is correct. The success of AAC intervention success is largely dependent upon the support of people close to the patient.

1. An SLP tries to elicit correct production of the target sound /s/ Which of the following words provides the best coarticulatory conditions to meet the SLP's goal? Spoon Sea Sob Soup

Option (B) is correct. The target sound /s/forward slash s forward slash and the vowel /i/forward slash i forward slash that follows it both

A 70-year-old female has dysphagia characterized by poor posterior oral containment of the bolus during the oral preparatory stage, causing aspiration before the swallow. Cognition and the pharyngeal stage of the swallow are intact. Which of the following is the most appropriate treatment approach for the client? A.Providing a puree diet with thickened liquids B.Having the client flex her head forward (perform the chin-down posture) during oral preparation and transit stages of the swallow C.Having the client turn (rotate) her head to the right when swallowing D.Providing a diet that consists of thin liquids

Option (B) is correct. This treatment alters oral configuration to place the base of the tongue superior to the bolus inside the oral cavity. Since liquid cannot flow uphill against gravity, this intervention compensates for impaired palatoglossus contraction, which is causing impaired posterior oral containment.

When the patient attempted the chin-down posture during the VFSSV F S S, which of the following observations was the SLPS L P most likely to make? • A.Aspiration from the pyriform sinuses was eliminated. • B.Aspiration from the pyriform sinuses was not eliminated. • C.The space between the tongue base and posterior pharyngeal wall increased. • D.The vallecular space became more narrow, preventing aspiration.

Option (B) is correct. Using the chin-down posture eliminated aspiration in 50% of patients with stroke who aspirated due to a pharyngeal onset delay. All patients (50%) who continued to aspirate did so from the pyriform sinuses—the chin-down posture empties the pyriform sinuses into the airway.

Which of the following is an accurate statement about whispered speech? A.It is produced by approximating the arytenoid cartilages so that their medial surfaces are in direct contact. B.It is composed largely of aperiodic sounds. C.Spectrographic analysis of it reveals no discernible formants for the vowels. D.Most people can produce longer utterances per breath using it than they can using conventional phonation.

Option (B) is correct. Whispered speech is composed largely of aperiodic sounds, as the vocal folds do not vibrate while whispering is taking place.

SLPs often have a responsibility to communicate with the parents of children with severe disabilities. According to mourning theory, when are parents normally most receptive to information and advice provided by professionals regarding their child? A.When the parents are working through their feelings about the child's disabilities B.When the parents fully realize the extent of the child's disabilities and the limitations of treatment and education C.When the parents have acquired greater confidence in their capacity to care for the child and greater motivation to cope with the child's disabilities D.When the parents are making decisions regarding future care and protection of the child

Option (C) is correct. According to mourning theory, a grieving person is most receptive to new information about the source of grief after having just entered the recovery stage.

94. Top of Form For which of the following conditions is it most appropriate for the SLP to recommend that the patient's primary-care physician refer the patient to a prosthodontist for construction of a palatal-lift appliance? A.Submucous cleft palate B.Unrepaired cleft of the secondary palate C.Flaccid paralysis of the soft palate D.Congenitally short palate

Option (C) is correct. Construction of a palatal-lift appliance is appropriate for a patient with flaccid paralysis characterized by an intact palate that does not function.

To best apply the multiple oppositions method for phonological errors when treating a child, an SLPS L Pshould use pairs of words such as • A. "son" and "ton," "fun" and "pun," and "zoo" and "do" to help discriminate stops from fricatives • B. "torn" and "sore," "soup" and "two," and "fast" and "pat" to help produce final consonants • C. "doe" and "go," "doe" and "though," and "doe and Joe" to address using /d/d, as in the word dog for various sounds • D. "key" and "tea," "cone" and "tone," and "cap" and "tap" to address the use of velar fronting

Option (C) is correct. In multiple oppositions treatment, targets are selected that will address all phonemes affected by a phoneme collapse, not just the target phoneme and the phoneme produced in substitution. Multiple Oppositions—a variation of the minimal oppositions approach but uses pairs of words contrasting a child's error sound with three or four strategically selected sounds that reflect both maximal classification and maximal distinction (e.g., "door," "four," "chore," and "store," to reduce backing of /d/ to /g/

Which of the following procedures would be effective in remediating a falsetto voice for an adult male with a severe bilateral hearing loss? A.Development of phonation from coughing B.Pushing exercises C.Manual depression of the larynx D.Manual elevation of the larynx

Option (C) is correct. Manual depression of the larynx will serve to lengthen the vocal folds, allowing them to vibrate at a lower frequency.

70. Top of Form An otolaryngologist has referred a 45-year-old man for voice treatment following medialization thyroplasty for a paralyzed vocal fold. Which of the following is the most appropriate therapeutic strategy for the SLP to use? A.Assisting the patient to produce a soft glottal attack B.Digitally manipulation of the patient's neck to reduce strap-muscle tension C.Assisting the patient to produce a hard glottal attack D.Employing techniques aimed at increasing airflow

Option (C) is correct. Medialization thyroplasty moves the paralyzed vocal fold closer to the mid-glottis to allow better compensation by the unaffected fold. Only the production of a hard glottal attack addresses the compensatory behavior.

Joanna is a 36-month-old child who is enrolled in a treatment program for stuttering. The SLPS L P is involving Joanna's parents and 5-year-old brother in treatment activities. Every time Joanna's brother participates in a treatment activity, Joanna begins to stutter much more severely than she does when interacting with her parents. Which of the following actions is the most appropriate for the SLPS L P to take next? • A.Helping the parents devise strategies for limiting how often Joanna and her brother converse with each other at home • B.Engaging Joanna in a role-playing activity that seeks to identify how she would like her brother to interact with her during conversations • C.Analyzing recorded conversations between Joanna and her brother to identify discourse factors that might affect the severity of Joanna's stuttering • D.Teaching her brother to produce statements that support Joanna's attempts to talk

Option (C) is correct. Research has shown that external factors can precipitate stuttering or worsen its severity.

A number of research reports have described poor auditory memory in children with language impairments. Which of the following can most appropriately be concluded from these studies? A.Poor auditory memory can be improved by language-intervention programs that focus on teaching vocabulary and word meanings. B.Poor auditory memory is a reflection of a language impairment, and clinicians and researchers cannot effect improvement. C.Poor auditory memory could be a reflection or a cause of a language impairment or could be related to some other factor, and further research is needed to determine which is the case. D.Children with language impairments normally do not have well-developed representational skills.

Option (C) is correct. Research reports are restricted to the variables examined in the studies on which they are based. A relationship between poor auditory memory and language impairment has been found in some research studies. Other factors could be involved, however, suggesting that further research is needed. Conclusions about treatment or the effects of the impairment must be made through additional research

Which of the following statements regarding risk factors for developmental reading disorders is most accurate? • A. There are no risk factors for developmental reading disorders that an SLPS L Pcan assess. • B. Only those children with a family history of reading disorders are at risk for developing a reading disorder. • C. Children with speech sound disorders (SSDsS S Ds) are at increased risk of having a reading disorder. • D. Children with SSDsS S Dsare only at increased risk of a reading disorder if they do not receive speech therapy.

Option (C) is correct. SSDsS S Ds are a risk factor for later reading disability in school-age children.

A 65-year-old-male who is hospitalized with pneumonia completed a videofluoroscopic swallowing study. Findings showed aspiration after the swallow due to a reduced upper esophageal sphincter opening diameter separating the bolus tail, which was then retained in the pyriform sinuses. Which of the following interventions was designed to and is shown to improve upper esophageal sphincter opening? A.Effortful swallow B.Valsalva (breath hold) C.Shaker (head lift) exercise D.Lingual resistance exercise

Option (C) is correct. Shaker et aland others. (2002) showed that the head lift exercise significantly increased the diameter of the upper esophageal sphincter opening in people with dysphagia.

Top of Form Excessive nasality is associated with inadequate velopharyngeal closure. An SLP is training a client to self-monitor nasality during speech. Which of the following tactics will best allow the speaker to determine whether there is excessive nasal airflow? A.Looking in a mirror while speaking B.Being aware of vowel-sound productions C.Speaking/phonating while alternately leaving the nostrils open and pinching them closed D.Monitoring production of consonant blends

Option (C) is correct. Speaking while alternately leaving the nostrils open and pinching them closed is an easy way for a speaker to determine whether inappropriate nasal airflow is occurring. This technique allows one to determine the difference in airflow pattern when speech is produced with the nostrils occluded as compared with speech produced when the nostrils are open. For a speaker with velopharyngeal incompetence, closure of the nostrils will eliminate the nasal airflow in production of /s/.

Kimberly, a patient who suffered a brain stem stroke and has had a fairly successful recovery, complained of food sticking in her throat and needing to swallow two to three times to clear liquid and solid boluses. She underwent a videofluoroscopic swallowing study that showed a limited duration of upper esophageal sphincter opening. The SLPS L P decides to employ an intervention plan that includes the Mendelsohn maneuver. Which of the following strategies is most likely to be effective in accomplishing this goal? • A.Completing a fiberoptic endoscopic evaluation of swallowing (FEESF E E S) to determine if the patient is performing the maneuver properly • B.Performing cervical auscultation (CAC A) to quantify improved swallowing with the maneuver • C.Using submental surface electromyography (sEMGs E M G) as biofeedback during attempts to perform the maneuver • D.Teaching the patient to use palpation of her neck and a mirror to help her master the maneuver

Option (C) is correct. Submental surface EMGE M G has been shown to improve acquisition of the Mendelsohn maneuver in some people though the maneuver itself is very difficult to learn.

92. Top of Form Which of the following areas needs to be evaluated first for a 5 year old who says [pun] for "spoon" and [top] for "soap"? A.Auditory discrimination B.Dialectal differences C.Phonological system D.Receptive language

Option (C) is correct. The error patterns described are phonological in nature (cluster reduction and stopping, specifically) and are not consistent with a deficit in language comprehension (receptive language), dialectal differences, or problems with auditory discrimination.

Which of the following communication diagnoses would most likely require a treatment program that is focused on improving auditory language comprehension? • A.Right-hemisphere communication impairment • B.Global aphasia • C.Wernicke's aphasia • D.Broca's aphasia

Option (C) is correct. The hallmark of Wernicke's aphasia is deficits in auditory comprehension, because the site of lesions is most typically the left superior temporal gyrus.

According to literature on men's and women's voices, the prevalence of benign vocal fold lesions in women can best be explained by differences in • A.daily dairy consumption • B.speaking duration • C.rates of vocal fold vibration • D.neck circumference

Option (C) is correct. The increased fundamental frequency of most female voices results in an increased number of glottal contacts during a fixed period of voice use. This increased number of contacts is thought to increase the risk of development of benign vocal fold lesions.

Studies of the anatomy of human vocal folds and of mucosal behavior during phonation have led to the current cover-body characterization of the vocal fold. Which of the following is included in the vocal-fold transition? • A.The epithelium and superficial lamina propria • B.The epithelium and deep lamina propria • C.The intermediate and deep layers of the lamina propria • D.The lamina of the thyroid cartilage

Option (C) is correct. The intermediate and deep layers are considered transitional because of where they are located anatomically.

61. Top of Form A 67-year-old male patient with no history of swallowing problems has undergone a cardiothoracic surgical procedure. Postoperatively, he is found to be aspirating while swallowing and is diagnosed with a left vocal-fold paralysis and left pharyngeal paresis. Which of the following is the most likely etiology? A.An intraoperative CVA in the right pons B.Damage to the right recurrent laryngeal nerve C.Damage to the left recurrent laryngeal nerve D.A left hemispheric stroke

Option (C) is correct. The left recurrent laryngeal nerve courses under the aortic arch in its course back to innervate the left larynx and the inferior pharynx. The nerve can be damaged in cardiothoracic operations including aortic arch or valve repairs. Only the left (and not the right) recurrent laryngeal nerve has this course.

When completing speech sound evaluations, it is best practice for an SLPS L P to include a sample of connected speech because connected speech samples A.provide more reliable information than standardized tests B.contain a representative sample of all the phonemes in a language C.may result in different types of errors than those elicited by single-word elicitation tasks D.are easier to gloss than single-word productions

Option (C) is correct. The more complex task of producing connected speech tends to increase the types and numbers of errors, allowing for an accurate reflection of the patient's speech ability.

John is a 4 1/2 year old whose consonantal inventory includes word-initial [ w ], [ m ], [ n ], [ p ], [ b ], [ t ], [ d ], and [ f ]. He uses [ t ] for /k/, [ d ] for /ɡ/, [ b ] for /v/, and [ f ] for /θ/. He produces no consonant clusters. His word-final consonantal inventory consists of [m] and [n]. His word shape inventory includes V, CV, CVC, and CVCV. The information given most strongly indicates that the child has A.childhood apraxia of speech B.an oral motor impairment C.delayed phonological development D.a significant high-frequency hearing loss

Option (C) is correct. The pattern describes the phonological pattern of a younger child. The errors are consistent, unlike childhood apraxia of speech, which has an inconsistent error pattern. Children at this age typically have mastered consonant clusters and use word-final consonants.

An SLPS L P in a private practice receives a letter from the state licensing office informing her that her license expired the previous week. The SLPS L P was unaware that she had been practicing with a lapsed license for seven working days. Which THREE of the following actions are the best course of action for the SLPS L P? • A.Continuing to see patients as scheduled • B.Discontinuing treatment documentation • C.Informing the practice administrator • D.Reassigning patients to a licensed SLPS L P • E.Alerting patients to the lapse in licensure

Option (C) is correct. The practice administration can begin to move forward with corrective action with patients, families, payers, and the licensing body. Option (D) is correct. Without a license, the SLPS L P should not see patients, so patients should be reassigned for continuing care. Option (E) is correct. Without a license, the SLPS L P should not see patients, so patients should be reassigned for continuing care.

128. Top of Form When treating a client who is using an electronic augmentative-communication device, the speech-language pathologist's primary goal should be to A.ensure that the client develops skill in using every technical aspect of the aid B.ensure that the client's caregivers learn how to modify the aid's hardware and any applicable software to meet the client's communication needs C.train the client to use the aid as independently and interactively as possible in a variety of settings D.help the client develop the skills necessary for moving on to a more sophisticated device

Option (C) is correct. The primary goal of any intervention is to effect the best functional outcome for the client. Generalization with respect to the environments in which the augmentative device is used effectively would help accomplish this goal.

Kinetic energy is energy that is being used. Over time, kinetic energy is transformed to thermal energy and the result of this is damping or damped vibration. What is damping?

Opposition to motion

An SLPS L P works at a hospital with the adult acute-care inpatient team. During a clinical bedside swallow evaluation, a patient asks the SLPS L P how common it is for adults to experience a swallowing disorder. The SLPS L P tells the patient that recent research indicates the prevalence of adults with swallowing problems each year is • A.1 in 25 • B.1 in 50 • C.1 in 100 • D.1 in 200

Option (A) is correct because recent research indicates that 1 in 25 adults are affected by a swallowing problem per year.

Which of the following statements best represents a syntactic structure characteristic of Spanish-influenced English? A."She no do laundry today." B."He is going?" C."Lady her shoes." D."She be runnin' fast."

Option (A) is correct. "No" may be used in Spanish-influenced English as a negative marker instead of "not."

An SLP receives a referral regarding a 4-year-old boy who uses two words spontaneously and functionally, who began walking at 3 years of age, and who responds to his name inconsistently. On the basis of the information alone, the SLP can legitimately conclude that the child's communication profile reflects A.a developmental delay B.autism spectrum disorder C.a chromosomal anomaly D.a metabolic disorder

Option (A) is correct. A 4-year-old typically developing child would have 4 word utterances and would respond to his name consistently. Most children start to walk around age 1. Therefore, the delay in walking, along with the delay in language, indicates a general developmental delay.

During which of the following weeks of pregnancy did the infant's craniofacial structures not develop completely?

Option (A) is correct. A cleft lip results from incomplete closure of lip tissues as the lips form early in pregnancy, between weeks four and seven. Both genetic and environmental factors likely contribute to a cleft lip, a congenital birth defect; however, a singular cause has not been identified.

8. Top of Form A clinician who employs active listening is doing which of the following? A.Responding to both the content and the affect of the client's remarks B.Listening very carefully and taking extensive notes C.Conducting a clinician-directed interview D.Directing the client to specific answers to questions

Option (A) is correct. A clinician who employs active listening responds to both the content (the denotative message) and the affect (the emotional content) of a client's remarks.

A single exposure of several hours duration to continuous music with an overall level of 100 dB SPL will most likely produce A.tinnitus and a temporary threshold shift in high frequencies B.tinnitus and a distortion of speech perception C.a temporary threshold shift in the low frequencies D.a permanent threshold shift

Option (A) is correct. A single exposure of several hours duration to continuous music at a level of about 100 dB SPL will most likely produce tinnitus and a temporary threshold shift in the high frequencies.

A 28-year-old female self-refers for a voice evaluation. She exhibits a variable dysphonia (it is present in some sentences but not in others) which presents as mild-to-moderate roughness. The SLPS L P desires more objective data about the patient's voice quality. Based on the information obtained so far, which of the following assessments best complements the perceptual assessment to help the SLPS L P determine the underlying physiological impairment(s)? A. Performing acoustic assessment B. Obtaining history of the problem C. Using the Consensus Auditory-Perceptual Evaluation of Voice (CAPEC A P E-V) D. Administering the Voice Handicap Index (VHIV H I)

Option (A) is correct. Acoustic measures can support a perceptual judgement of voice quality and have been found in several studies to differentiate normal from pathologic conditions of the voice.

Which of the following must a child first be able to do before the child can produce narratives? • A.Produce several utterances on the same topic • B.Use past and future tenses • C.Express a sequence using "then" or "next" • D.Produce compound sentences using "and"

Option (A) is correct. As a first step toward the production of a cohesive narrative, a child must be able to connect a collection of ideas together.

105. Top of Form For which of the following reasons would the therapy for a client whose language loss is due to brain injury differ from the therapy for a client whose language loss is due to a progressive disorder? A.There is a much higher chance of spontaneous recovery for the client with brain injury. B.There is a much higher chance of spontaneous recovery for the client with a progressive disorder. C.The client with brain injury typically has problems with fatigue and mental confusion, whereas the client with a progressive disorder does not. D.The client with a progressive disorder typically has problems with attention and memory, whereas the client with brain injury typically does not.

Option (A) is correct. Brain injury is typically characterized by some degree of spontaneous improvement over a period of several days to weeks, which may be evidenced through gradual improvement in language abilities, whereas a progressive disorder is characterized by gradual loss/reduction in language skills.

Which of the following is characteristic of child directed speech? • A.Exaggerated pitch contours • B.Imprecise articulation • C.Increased speech rate • D.Shortening the pauses between words

Option (A) is correct. Child-directed speech refers to the way adults speak to young children and is characterized by exaggerated pitch contours, such as using a higher pitch than normal, and overly dramatic facial expressions.

24. Top of Form Children diagnosed as having specific language impairments are likely to exhibit the greatest deficits in which of the following? A.Production of sentences with appropriate inflectional morphology and syntax B.Acquisition of word meanings C.Comprehension of short sentences D.Motoric aspects of written expression

Option (A) is correct. Children with specific language impairments typically have difficulty producing utterances that are morphologically and syntactically well formed.

After completing an evaluation of a 5-year-old patient, an SLPS L P finds that the child's speech contains numerous phonological error patterns. If the SLPS L P follows a developmental sequence in planning intervention, which of the following is most appropriate to target first? • A.Consonant assimilation • B.Gliding • C.Deaffrication • D.Cluster reduction

Option (A) is correct. Consonant assimilation processes rarely persist beyond 3 years of age.

77. Top of Form Which of the following would be most likely to help a client who has aphonia? A.Development of phonation through coughing or throat clearing B.Pairing the production of /s/ and /z/ C.Respiratory exercises D.Easy initiation of phonation

Option (A) is correct. Development of phonation through coughing or throat clearing is an effective technique in the treatment of aphonia.

An endoscopic examination of swallowing is the best instrumental assessment when the SLPS L P wishes to evaluate the A.volume of and patient's response to pharyngeal secretions B.duration and extent of hyolaryngeal elevation C.penetration of material into the laryngeal vestibule during the swallow D.efficiency of lingual manipulation during bolus formation and anterior-posterior bolus transit

Option (A) is correct. Endoscopic viewing is superior to other examinations when the primary goal is to evaluate the volume of and the patient's response to pharyngeal secretions.

88. Top of Form A correct-response rate of 51 percent on a two-choice picture-pointing task would most likely indicate which of the following? A.A random pointing response B.Successful intervention C.Development of crucial discrimination skills by the client D.Readiness to progress to a three-picture point task

Option (A) is correct. If an individual is responding randomly on a two-choice task, then there should be no significant preponderance of correct responses over incorrect responses. A result of 51 percent represents such an expected pattern of random responses.

68. Top of Form A 9-month-old child was observed during a speech-language evaluation. To express herself, the child occasionally touched her mother, gained eye contact, and then gestured toward an object. If the child's development is normal, within the next month or so the child will begin to A.use consistent sound and intonation patterns as signals for specific intentions B.reach for objects that she desires C.establish joint reference with her caretaker D.use recognizable words and phrases to express her intentions

Option (A) is correct. In normal development, a child begins to use vocalizations to express specific intentions at around 10 months of age.

A 75-year-old man presents with conversational speech characterized by prolonged silent intervals and hypophonia. Resonance is normal, but voice quality is rough and tremulous. Pitch is relatively unaffected. There is no evidence of speech deterioration over time. Which of the following diagnoses is the most reasonable based on the patient's data? A.Hypokinetic dysarthria B.Ataxic dysarthria C.Hyperkinetic dysarthria D.Flaccid dysarthria

Option (A) is correct. Inappropriate silences, imprecise consonants, variable rate, short rushes of speech, increased rate in segments, and hoarse and tremulous voice quality are common characteristics of hypokinetic dysarthria. Speech deterioration over time and resonance are not typically affected.

Which of the following will most effectively increase the fundamental frequency of the voice? A.Lengthening the vocal folds B.Decreasing the stiffness of the vocal folds C.Decreasing subglottal pressure D.Decreasing cricothyroid muscle activity

Option (A) is correct. Lengthening the vocal folds causes the thyroarytenoid muscle to thin, which allows fundamental frequency to increase.

Which of the following procedures best establishes whether velopharyngeal dysfunction causing hypernasality is present in a patient with flaccid dysarthria? • A.A nasopharyngoscopy • B.A videofluoroscopic swallow study • C.A laryngoscopy • D.A manometry

Option (A) is correct. Nasopharyngoscopy is the only procedure that allows visualization of the velopharyngeal port during speech. Esophageal manometry is a test that is used to measure the function of the lower esophageal sphincter

59. Top of Form Naturalistic teaching chiefly involves which of the following? A.Establishing successful and useful communication B.Using multiple trials and training techniques C.Using more adult-initiated interactions than child-initiated interactions D.Using differential reinforcement, fading, and modeling

Option (A) is correct. Naturalistic teaching focuses on the successful production of utterances that are useful in context for communicating.

Which of the following statements best explains why thickened liquids for adult patients with dysphagia should be used with caution? • A.Patients dislike thickened liquids and therefore do not drink enough, resulting in dehydration. • B.The thickened liquid becomes thinner as it sits at the bedside, negating the liquid's benefit as a compensatory diet modification. • C.Patients need twice as much thickened liquid because it provides half the hydration that thin liquid provides. • D.Thickened liquids are aspirated more frequently than thin liquids.

Option (A) is correct. Research indicates that patients do not like thick liquids, and there is evidence that this leads to dehydration. A beginning clinician should be well aware of this and use caution when recommending thickened liquids to adult patients with dysphagia.

Joe, an 18-year-old male, sustained a severe traumatic brain injury following a motorcycle accident. He was in a coma for approximately four weeks before being transferred to a rehabilitation program. Joe demonstrates severe attention deficits, anomia, disorientation, and poor short-term memory. Which of the following should be the initial target area in the treatment plan? • A.Retraining attention as a discrete cognitive process • B.Using declarative memory as an active learning strategy • C.Using implicit memory as a passive learning strategy • D.Reality orientation training

Option (A) is correct. Retraining attention is the most appropriate initial target area in this situation; all the other responses require attention as a prerequisite.

Which of the following statements regarding statistical significance is true? • A.A statistically significant difference can occur between experimental groups and control groups even if the magnitude of difference between the groups is quite small. • B.If a statistically significant difference between groups occurs, it means that large and important change occurs in at least one of the groups. • C.Statistically significant findings in a clinical research study suggest that the treatment is successful and highly recommended. • D.The terms "statistically significant" and "clinically significant" are interchangeable.

Option (A) is correct. Statistical significance is a measure of how likely it would be to get the produced results by chance, not necessarily an indicator that the change or the difference was large.

A client with anomic aphasia is a native speaker of Spanish with fair proficiency in English. Production of the word "shoes" as [tʃuz] on a repetition task is most likely due to which of the following factors? A.Phonological interference from the speaker's native language B.Semantic interference from the speaker's native language C.Semantic paraphasia due to the aphasia D.Phonemic paraphasia due to the aphasia

Option (A) is correct. The Spanish language does not have /f/ in its phonemic inventory, and native speakers of Spanish typically substitute the affricate /t∫//t∫/ when producing English words with that phoneme. A native speaker of Spanish would most likely have made the substitution before acquiring an anomic aphasia.

Immediately following removal of a benign tumor from the base of the brain, a 76-year-old client exhibits severe nasalization and a weak, breathy voice. A four-month postsurgical assessment reveals no improvement. At this time, the remediation strategy for this client should focus on A.evaluation for prosthetic or surgical intervention B.strengthening exercises for the oral articulators C.a trial period using the yawn-sign technique D.complete vocal rest

Option (A) is correct. The client has a resonance and phonation disorder indicative of velopharyngeal and laryngeal problems. The velopharyngeal problem could be assisted by prosthetic or surgical intervention.

1. Inhalation during respiration is primarily carried out through the movement of which of the following muscles or muscle groups? Diaphragm Internal intercostals External intercostals Scalenes

Option (A) is correct. The diaphragm is the muscle that creates the most volume in the lungs

121. Linguistic approaches to the treatment of sound-production errors in children are based on the notion that the errors are systematic and rule-based and that the goal of treatment is to modify a child's rule system to approximate the rule system used by adults. Which of the following is a treatment objective that reflects a linguistic approach to treatment? A.The child will contrast alveolar stops with velar stops in meaningful word pairs. B.The child will produce voiceless alveolar sibilants correctly in unstructured conversation. C.The child will coarticulate stop plus liquid clusters as easily as other children of the same age. D.The child will recognize and identify phonetic distortions of his or her error sound

Option (A) is correct. The objective refers to speech sounds in terms of general distinctive-feature classes, rather than in terms of isolated phonemes or overly specific classifications.

98. Top of Form Which of the following is a typical symptom of cerebellar involvement? A.Overshooting or undershooting an intended target B.Rigidity during voluntary motions C.Spasticity during involuntary action D.Word-finding difficulty

Option (A) is correct. The overshooting or undershooting of an intended target is typical of individuals with deficits related to a lesion of the cerebellum.

You are working in a public school, and a distraught parent calls you. She states that her son who was prenatally exposed to drugs has been denied special education services including speech-language intervention. Why may children who are prenatally exposed to drugs be denied services in public schools?

Their language problems are not readily detected by standardized language measures.

Which of these ideas do the Universalist-Linguistic school of phonological acquisition theories include? Children develop their phonological concepts through passive learning Children develop phonological principles in a manner that is not linear Children demonstrate their phonological acquisition as active learners Children demonstrate individual differences in phonological acquisition

Theories of phonological acquisition termed Universalist-Linguistic maintain that children acquire phonological concepts as passive learners rather than actively. They also hold that phonological abilities do progress in a linear fashion; that all children universally develop phonological knowledge in the same way, without individual differences. The three incorrect choices all represent principles of cognitive theories of phonological acquisition.

A speech language pathologist is evaluating a child with Fragile X Syndrome. Which of the following characteristics can the clinician expect to observe? smooth philtrum wide mouth, full lips macroglossia elongated face, prominent ears

These facial characteristics are common in children with Fragile X Syndrome, elongated face, prominent ears

nodes and antinodes

Velocity and pressure are 90 degrees out of phase. So pressure and velocity for a sine wave are inversely related. A node represents maximum pressure but minimum volume velocity. There is ALWAYS a node at the closed end of the tube. At the closed end, velocity must be zero. And there is always a velocity maximum at the open end—the antinode. Remember that antinodes are points of maximum velocity. At the point of maximum velocity, a constriction in the vocal tract will lower the formant frequency. If we have an expansion at a point of maximum velocity, it increases the resonant frequency. Let's look at points of maximum pressure, which are the nodes. If we have a constriction at a node, it increases resonant frequency. And an expansion decreases the resonant frequency.

1. An SLP designs a series of treatment activities for Jake, a 4 year old who presents with severely delayed phonological development. During one of the activities, the SLP asks Jake to say various word pairs in which two phonologically dissimilar target sounds are contrasted (for example, "chip" and "rip"). Which of the following choices best describes the treatment approach that is being used? Multiple-oppositions approach Minimal-pairs approach Maximal-oppositions approach Metaphonological approach

· Option (C) is correct. The maximal-oppositions approach contrasts two errored sounds differing across place, manner, and voicing to gain the greatest amount of generalization

1. The head lift exercise published by Shaker et al. (2002) was designed to produce which of the following changes in swallow biomechanics? Increase in laryngeal closure duration Strengthening of the base of the tongue Increase in diameter and duration of the upper esophageal sphincter (UES) opening Prolonged relaxation of the cricopharyngeal portion of the inferior constrictor muscle

· Option (C) is correct. The results of Shaker et al. (2002) demonstrated a significant (p<.01p left angle bracket period zero one) increase in the opening duration of the UESU E S.

79. Alan's performance profile reveals which impairment typical to right hemisphere dysfunction? • Aprosodia • Unilateral spatial neglect • Hyperresponsiveness • Anosognosia

· Option (D) is correct. Anosognosia, the reduced awareness of deficits, is common after right hemisphere disorder Search Results Featured snippet from the web www.researchgate.net Aprosodia is a neurological condition characterized by the inability of a person to properly convey or interpret emotional prosody.

1. Which of the following instruments is often used to document stages of recovery after traumatic brain injury (TBIT B I) ? Glasgow Coma Scale Scale of Executive Function Sequenced Inventory of Communication Development Rancho Los Amigos Scales of Cognitive Function'

· Option (D) is correct. The Rancho Los Amigos Scales of Cognitive Function is the only truly "scaled" instrument developed specifically for rehabilitation documentation following TBIT B I.

1. Which of the following neuroimaging studies specifies which artery or arteries is/are occluded in a patient with a stroke? Electroencephalography Magnetoencephalography Evoked-response potentials CT or MR angiography

· Option (D) is correct. These studies will give results which specify which artery is occluded

108. According to empirical research with people who stutter, which TWO of the following are true regarding the age of symptom onset for most cases? • It is usually earlier for girls than for boys. • It is similar for girls and boys. • It most often occurs in the range of 2 to 5 years old. • It most often occurs in the range of 6 to 9 years old.

· Options (A) and (C) are correct. Research findings indicate that the age of onset for stuttering symptoms is somewhat earlier for girls than for boys, and in most cases, symptom onset occurs in the preschool years.

30. An SLP assesses a 3-year-old child with poor intelligibility. The SLP attempts to determine whether the child has a severe phonological disorder or childhood apraxia of speech (CASC A S). Which TWO of the following characteristics are primarily indicative of CASC A S? • Presenting with decreased strength and muscle coordination • Displaying inconsistencies in articulation performance • Making the same errors independent of length of speech utterance • Speaking with a disrupted rate, rhythm, and stress of speech

· Options (B) and (D) are correct. Children with CASC A S struggle to make consistent productions of speech sounds. Children with CASC A S speak with a rate, rhythm, and stress of speech that can be disrupted, and sometimes they can appear to be groping for placement.

Displacement

which negative feelings are transferred from the original source of the emotion to a less threatening person or object. The negative emotions elicited toward the source of the feelings are instead redirected toward a more powerless substitute.

115. Which of the following best represents an interprofessional treatment model? • The SLP designs an intervention plan to be implemented by an SLP assistant. • The SLP and general education teacher plan weekly language activities that they deliver jointly to the classroom. • The SLP conducts the evaluation and the team designs the treatment program. • Two district SLPs design language screening tools for the Response to Intervention (RTI) team.

· Option (B) is correct. This is the only option that involves two different professions working together on the same area or issue

118. Which of the following diagnoses is most appropriate for this patient? Person with a cognitive problem not due to a stroke • Cognitive communication deficit • Other symbolic dysfunction • Age-related cognitive decline • Mild cognitive impairment

· Option (B) is correct. When treating a patient for a cognitive impairment not caused by a stroke, SLPs should use a diagnosis of other symbolic dysfunction.

1. Early intentional communication typically emerges in the months leading up to a child's first birthday. Which of the following communicative functions typically emerges first in this period? Requesting Commenting Protesting Greeting

· Option (C) is correct. Early intentional communication develops around 8 to 10 months of age, and the first function that emerges is protesting.

1. The Family Educational Rights and Privacy Act (FERPA) guarantees parents access to their child's educational records. However, this mandate does not apply to the daily records kept by an SLP working in a school setting if these records are not used for treatment-related reimbursement from state or local government remain within the school district in which the SLP works are kept in the sole possession of the SLP are not distributed to anyone outside of the child's school

· Option (C) is correct. FERPA states that daily notes can be kept in the sole possession of an SLP as long as the purpose is to serve as a "memory jogger" for the creator of the record.

43. Which of the following speech-sampling contexts best assesses hyponasality? • Producing sentences with oral sonorants • Counting in numerical order from 60 to 70 • Saying single words with nasal consonants • Repeating words with oral-pressure consonants

· Option (C) is correct. Hyponasality (too little nasal resonance) is most easily detected during the production of nasal consonants.

1. Which of the following is an example of a reversible passive? The balloon was broken by the pin. The kite was flown by Ben Franklin. Jared was bullied by Michael. The girl put her doll by her friend.

· Option (C) is correct. In reversible passive constructions, participants can fit into either role.

1. Which of the following strategies can the SLP use to most effectively engage the patient in monitoring his progress? Recording their sessions for the patient to listen to later Reinforcing that the aphonia is not the patient's fault Helping the patient recognize his control over his vocal quality Asking the patient to keep a journal of why he feels he cannot use his voice

· Option (C) is correct. Once the patient understands that he is in control and can use his voice, he will be more likely to use his voice consistently.

22. Which of the following statements best indicates a screening tool with high sensitivity? • The majority of the children who passed the screening were found to have a communication disorder. • Many of the children who passed the screening did not have a communication disorder. • All of the children who failed the screening were found to have a communication disorder. • Most of the children who failed the screening did not have a communication disorder.

· Option (C) is correct. Sensitivity is the success rate in identifying all of the children who truly have a disorder.

What type of reinforcement results in the most rapid learning of new behavior? A fixed ratio B fixed interval C continuous D variable-interval

Continuous the desired behavior is reinforced every time it occurs, good for the initial stages of learning to create a strong association between the behavior and the response

Vowel perception is generally thought to be: Harmonic. Categorical. Transitional. Continuous.

Continuous.

Vowel errors and errors on sonorant (non-obstruent) consonants would be manifest as: Changes in voice onset time. Changes in the duration of the silent interval. Changes in the frequency of the noise. Changes in the formant structure.

Changes in the formant structure.

117. Top of Form An individual attempting to sustain /f/ is shown in the midsagittal view above. The most important reason why this speaker will not produce an acceptable /f/ is that A.the upper central incisors are tipped too far labially B.the lower central incisors tipped too far labially C.intraoral air pressure will be insufficient D.lip placement is inadequate

C.intraoral air pressure will be insufficient

106. Which of the following best distinguishes a dialect from an accent? • Differences in language as well as in pronunciation • Differences in pronunciation only • Differences due to the influence of a second language • Differences that are unique to a particular speaker

• Option (A) is correct. A dialect involves differences in language and pronunciation.

58. To compensate for the effects of normal aging on cognitive functions, older adults will naturally demonstrate • greater amounts of bilateral activation of prefrontal brain regions • significant reduction in the activation of the hippocampus • increased activation in areas of the cortex responsible for visual-spatial processing • an increase in semantic abilities that do not rely on long-term memory abilities

• Option (A) is correct. Alterations in the prefrontal areas in particular are some of the changes seen in normal aging, in part to compensate for reduced frontal activation.

124. Which of the following activities is a principal component of the Lidcombe Program for childhood stuttering? • Teaching caregivers to provide feedback to their child about the child's fluent and stuttered speech • Teaching caregivers to support their child's communication attempts but to avoid acknowledging the child's fluency performance • Teaching children to describe their emotional state to their caregiver when stuttering is anticipated on an upcoming word • Teaching children to present nonverbal cues to their caregivers when stuttering is anticipated on an upcoming word

• Option (A) is correct. The Lidcombe Program is considered a direct therapy approach in which primary caregivers explicitly provide feedback on the child's level of fluency. All other options are part of indirect therapy approaches.

85. An SLP working with a 45-year-old patient with acquired expressive communication deficits is emphasizing reengagement by focusing on realistic short-term goals of the patient's choice. The SLP is primarily using which of the following approaches to therapy? • Life Participation Approach to Aphasia (LPAA) • Visual Action Therapy (VAT) • Supported Communication Intervention (SCI) • Prompts for Reconstructing Oral Muscular Phonetic Targets (PROMPT)

• Option (A) is correct. The Life Participation Approach to Aphasia (LPAAL P A A) has the overriding philosophy to maximize clients' re-engagement in life and bases all decision making on the life concerns identified by the clients and their families

123. Clinician: "What do you have there?" Child: "I had a big bus." Clinician: "Is that right?" Child: "Uh, I have a big bus." Clinician: "Now, is that right?" Child: "Yeah." What language-stimulation technique is the clinician using in the above exchange? • Self-evaluation • Rephrasing • Reauditorization • Parallel talking

• Option (A) is correct. The clinician is using prompts to have the child self-evaluate his or her own statements.

73. An SLP is evaluating a preschool child referred for concerns regarding stuttering. In which of the following areas is performance likely to be weaker in a preschooler who stutters than in a typical preschooler? • Hearing acuity • Voice • Language • Swallowing

• Option (C) is correct. Some young children who stutter present with concomitant language delay/disorder. School-based SLPs report that among the children to whom they provide stuttering services, a significant percentage are also receiving language services. Some studies with preschoolers and children in early elementary grades who stutter have shown evidence of group-level deficits in various aspects of language functioning. Thus, in a number of studies, a significant subset of children who stutter have shown language skills that are weaker than those of typical children.

61. Esophageal and tracheoesophageal (TEP) techniques for producing alaryngeal voice are similar in that both • redirect expiratory airflow from the lungs to the esophagus • result in a vocal fundamental frequency similar to that of laryngeal phonation • require the speaker to occlude the stoma with either a thumb or a valve • depend on adequate vibration of the pharyngoesophageal (PE) segment

• Option (D) is correct. Both esophageal speech and tracheoesophageal speech require vibration of the P EP E segment. Esophageal speech uses air from stomach/lower esophagus to upper esophagus/pharynx, where it vibrates the wall. TEPT E P speech requires the patient to cover the stoma to redirect air and initiate vibration. Fundamental frequency is different than that of laryngeal phonation (e.g.for example, variability and prosody).

55. Place the names of the following theorists with their associated concepts -- , language generativity, , discriminative stimulus, object permanace, zone of proximal development • Piaget - • Skinner - • Vygotsky - • Chomsky -

• Piaget -object permanence • Skinner - discriminative stimulus • Vygotsky -zone of proximal development • Chomsky -language generativity • Discriminative stimulus is associated with Skinner. Zone of proximal development is associated with Vygotsky. Language generativity is associated with Chomsky. Object permanence is associated with Piaget. Any other pairings would be incorrect.

Which of the following consonant sound classes would show an overall drop in energy compared to the surrounding vowels? Stops. Liquids. Affricates. Nasals. All of the above.

All of the above.

Obstruent sounds differ from resonant sounds in that obstruent sounds:

Always include an aperiodic sound source

Is the most common type of dementia diagnosed when

Alzheimers, autopsy

Association fibers are

Association fibers are axons that connect cortical areas within the same cerebral hemisphere.

A child who shows slow, writhing, involuntary movements has which type of cerebral palsy

Atheotid

F0

Created by vocal folds

For example, she says things such as "My candy is tasty than yours" instead of "My candy is tastier than yours," or "He was happy than she was" instead of "He was happier than she was." Delanie is having difficulty specifically with which aspect of language? A. Pragmatics B. Semantics C. Syntax D. Morphology

D. Morphology

Inductive method

a process of making numerous observations until one feels confident in drawing generalizations and predictions from them

Negative reinforcement

a term described by B. F. Skinner in his theory of operant conditioning. a response or behavior is strengthened by stopping, removing, or avoiding a negative outcome or aversive stimulus Scratching an insect bite that itches (reinforces scratching behavior by removing itch) Rubbing itchy eyes (reinforces rubbing behavior by removing itch)

A study that is used to quantify the time and frequency of gastroesophageal reflux into the esophagus is called

a pH probe or intraluminal pH monitoring study.

If an African American adult patient in a hospital setting says, "I been had the measles when I was a kid," this is an example of A. circumlocution. B. semantic paraphasia. C. a perfective construction. D. the use of an additional auxiliary.

a perfective construction.

A 60-year-old male is referred to the SLPS L P in an acute rehabilitation center for a videofluoroscopic swallowing study (VFSSV F S S) following a first-time stroke: a lateral medullary stroke damaging the right nucleus ambiguus and other centers. The clinical examination revealed ptosis of the left eye, soft palate elevation to the right of midline, symmetrical lingual protrusion, and a breathy dysphonia with precise articulation of all consonants. The patient is unable to swallow his saliva and coughs when he attempts to do so. Based on the stroke type and clinical examination observations, which THREE of the following swallow impairments would the SLPS L P likely observe on the VFSSV F S S? • A.Incomplete clearance of material from the oral cavity • B.Impaired or absent opening of the upper esophageal sphincter • C.Anterior loss of bolus contents during the oral preparatory stage • D.Nasopharyngeal regurgitation during the pharyngeal stage • E.Adequate excursion of the hyolaryngeal complex • F.Unilateral pharyngeal paralysis

Options (B), (D), and (F) are correct. Impaired or absent opening of the upper esophageal sphincter, nasopharyngeal regurgitation during the pharyngeal stage, and unilateral pharyngeal paralysis are all to be expected based on this type of stroke and the clinical observations.

A 28-year-old classroom teacher complaining of frequent voice loss is seen by an SLPS L P and an otolaryngologist. It is determined the patient's symptoms are linked to significant vocal demands. Which of following recommendations is the most appropriate first step to treat the underlying disorder? • A.Advising the patient to undergo complete voice rest until the nodules improve or resolve • B.Educating the patient on the importance of hydration and behavioral antireflux strategies • C.Recommending voice amplification with resonant voice therapy • D.Training strategies to increase vocal loudness

Option (C) is correct. Voice amplification and voice therapy exercises that alter vibratory impact (like vocal function exercises and resonant voice therapy) have been shown to be more effective courses of treatment in heavy voice users as compared to vocal hygiene strategies or respiratory retraining strategies.

100. Top of Form Following anoxic encephalopathy, clients are likely to experience the most significant long-term impairments in the area of A.prosody B.resonance C.aphonia D.memory

Option (D) is correct. Anoxic encephalopathy, or brain damage resulting from oxygen deprivation, typically leads to global impairment that affects memory. It is difficult to reverse the effects of memory loss, so the impairment is usually long-term.

26. Top of Form The major objective of auditory training in the treatment of a client with a hearing loss is to A.improve the client's awareness of position and movements of the speech mechanism B.improve the client's kinesthetic and auditory awareness C.increase the client's kinesthetic and proprioceptive discrimination D.teach the client to make discriminations among speech sounds

Option (D) is correct. Auditory training focuses on the interpretation of auditory input and would thus teach a client to discriminate speech sounds.

According to research, which of the following structural factors has the most adverse effect on articulation? A.Short labial frenulum B.Micrognathia C.Unilateral facial palsy D.Class III malocclusion

Option (D) is correct. Class III malocclusion is known to cause obligatory distortions of sibilants and affricates due to placement of the tongue relative to the maxilla.

78. Top of Form A child with discourse problems is most likely to need remediation directed at which of the following? A.Morphology B.An initial lexicon C.Gestural communication D.Cohesive devices

Option (D) is correct. Cohesive devices, such as prenominal references, coordinating conjunctions, and conjunctive adverbs, are used to link clausal and sentential elements to form a coherent and unified message.

Which of the following plans is most appropriate when assessing a patient with a history of traumatic brain injury (TBI) to characterize cognitive aspects of communication that affect functional abilities? A.Administering an aphasia battery to form a comprehensive assessment of the patient's linguistic abilities B.Conducting an assessment to evaluate for the presence and severity of dysarthria C.Observing a conversation between the patient and a familiar individual for difficulties in functional communication D.Using standardized tests as needed and supplementing with discourse samples and dynamic assessment

Option (D) is correct. Combining carefully interpreted standardized tests with discourse and dynamic assessment is recommended to achieve the most representative view of a patient's cognitive and linguistic abilities.

76. Top of Form Research regarding the use of intensive phonemic-awareness treatment for children who have difficulty learning to read has demonstrated that the treatment A.is effective only for children from 4 to 8 years old B.is effective mainly with children who have remediated all phonological process errors C.is effective only when combined with a supplemental literacy program D.might have no direct relationship to improvement in reading abilities

Option (D) is correct. Intensive phonemic-awareness treatment programs are thought to improve reading by training children to better differentiate and process speech sounds. However, to date, research has not successfully separated the effects of intensive intervention, so no direct relationship has yet been proved.

74. Top of Form For a test of expressive morphology and syntax for speakers of African American Vernacular English (AAVE), the test item that would be considered LEAST biased against such speakers would be one requiring A.use of the auxiliary verb "be" in the present progressive tense B.use of the past-tense ending "-ed" C.use of sentences with multiple negation D.agreement of personal pronouns with their antecedents in gender and number

Option (D) is correct. The morphological feature listed is one in which African American Vernacular English (AAVE) does not differ from Standard American English (SAE).

Which of the following is an etiology for cortical dementia? A.Amyotrophic lateral sclerosis B.Parkinson's disease C.Huntington's disease D.Alzheimer's disease

Option (D) is correct. The most common etiology of dementia is a group of progressive dementing diseases (further categorized as either cortical, subcortical, or mixed). Alzheimer's disease is a cortical dementia.

A prospective client is described as a man in his forties who is under chronic stress. He uses his voice extensively in daily life has a hard-driving personality, and exhibits glottal fry. The client has the classic profile of a person at high risk for A.spastic dysphonia B.acute laryngitis C.vocal nodules D.contact ulcers

Option (D) is correct. The symptoms exhibited by this patient represent a classic profile of a person who has contact ulcers.

103. Top of Form A child repeatedly inserts an inappropriate sound in certain environments; for example, [fpɪʃ] for [fɪʃ] . Which of the following would likely be most helpful for the child as a target for treatment? A.Bisyllabic words for which a minimal-contrast pair can be easily identified B.Words containing phonemes that have distinctive features in common with the sound the child inserts inappropriately C.Repeated practice with the combinations of phonemes that the child finds particularly easy to produce D.Words that contrast the child's error pattern with the target pattern in the word

Option (D) is correct. Words that contrast the child's error pattern with the target pattern would be most helpful for this child.

115. Top of Form Which of the following factors contributes to UES opening? Select all that apply. A.Partial relaxation of the cricopharyngeal portion of the inferior constrictor muscle B.Superior and anterior hyolaryngeal excursion C.Velopharyngeal closure D.Posterior and inferior hyolaryngeal excursion

Options (A) and (B) are correct. Partial relaxation of the cricopharyngeal portion of the inferior constrictor muscle is part of the neurological sequence. Superior and anterior hyolaryngeal excursion provides a mechanical opening.

4. Top of Form Fela is a third-grade student in a public school. She is a speaker of African American Vernacular English (AAVE) who has difficulty with the Standard American English (SAE) dialect used in her classroom. Her teacher believes that Fela's language skills are affecting her academic performance and has referred her to the school's speech-language pathologist. Which of the following is an appropriate rationale for providing language intervention for Fela? Select all that apply. A.It will likely foster better communication with Fela's linguistically and culturally diverse peers. B.It will likely improve Fela's code-switching ability with her teacher and other adult speakers of SAE. C.It may expand Fela's later academic and vocational opportunities. D.It will likely lead Fela to adopt SAE as her primary dialect.

Options (A), (B), and (C) are correct. The intervention will foster better communication because Fela and her peers will have a common dialect. Also, the intervention will provide Fela with the ability to switch easily between dialects using SAE structures. Furthermore, being able to switch codes will enable Fela to participate in more educational and vocational opportunities.

Which THREE of the following are additional communication strategies that are valid for MsMiss. Malone? A.Incorporating multimodal communication strategies, such as gestures and writing B.Teaching regular communication partners how to listen and best respond to her C.Limiting social engagements, since language demands are beyond the severely impaired AACA A C user D.Encouraging her to take an active role in initiating communication E.Phasing out device use to increase the rate of spoken-language recovery

Options (A), (B), and (D) are correct. Option (A) is correct because multimodal strategies augment the entirety of communication. Option (B) is correct because AACA A C intervention works best when regular communication partners are involved. Option (D) is correct because taking an active role in initiating communication will help MsMiss. Malone incorporate the AACA A C intervention into her daily life and result in improved confidence and less frustration.

. Top of Form An SLP who is treating an adolescent who stutters designs a treatment plan that includes three fluency management strategies: prolonged speech, cancellation, and pullout. Which of the following is true about the use of these treatment strategies? Select all that apply. A.Use of prolonged speech is likely to reduce the frequency of part-word repetitions and sound prolongations significantly. B.Each of the three strategies entails deliberate regulation of speech motor movements. C.The client will seek to apply cancellation whenever he feels anxious about the possibility of stuttering overtly. D.The client will seek to apply pullout during the course of part-word repetition or sound prolongation.

Options (A), (B), and (D) are correct. Prolonged speech, deliberate regulation of speech motor movements, and pullout could all be appropriately applied.

Which of the following are the THREE types of velopharyngeal dysfunction? • A.Velopharyngeal insufficiency • B.Velopharyngeal mislearning • C.Velopharyngeal mismatching • D.Velopharyngeal ineffectiveness • E.Velopharyngeal incompetence

Options (A), (B), and (E) are correct. Velopharyngeal dysfunction (VDPV D P) is a general term. It is used to describe different disorders of the velopharyngeal valve. These include: velopharyngeal insufficiency (VPIV P I), which is due to abnormal structure, velopharyngeal incompetence (VPIV P I), which is due to abnormal movement, and velopharyngeal mislearning, which is due to abnormal speech sound production.

An SLPS L P recommends a patient perform a chin-down posture (CDPC D P) when swallowing liquids. Which THREE of the following statements accurately represent published evidence regarding the chin-down posture? • A. CDPC D P reduced thin liquid aspiration caused by delayed pharyngeal response in people who had a stroke. • B. CDPC D P increased the speed and completeness of oral transit in people with dysphagia after a stroke. • C. CDPC D P was shown to narrow the width of the laryngeal inlet and widen the vallecular space during swallowing. • D. CDPC D P resulted in an increased upper esophageal sphincter opening diameter in patients with Parkinson's disease. • E. CDPC D P does not reduce aspiration after the swallow in patients who had a stroke and aspirate pyriform sinus residue.

Options (A), (C), and (E) are correct. A study conducted by Shanahan et aland others. in 1993 showed that using the CDPC D P reduced thin liquid aspiration caused by delayed pharyngeal response in people who suffered had a stroke. Studies have shown that using CDPC D P during swallowing narrows the width of the laryngeal inlet and widens the vallecular space during swallowing when the CDPC D P is used. The 1993 study performed by Shanahan et aland others. showed that the CDPC D Pdoes not reduce aspiration after swallowing in patients who had a stroke and aspirate pyriform sinus residue.

Dysphagia screen results Failed screen 45 20 Passed screen 5 30 Click on your choices. Question: Which THREE of the following statements about this dysphagia screening test are true? • A.The screening test's sensitivity is 90% • B.The screening test's sensitivity is 69% • C.The screening test's specificity is 90% • D.The screening test's specificity is 60% • E.The number of false negatives was 5 • F.The number of false negatives was 20

Options (A), (D), and (E) are correct. The screening test's sensitivity is the proportion of patients who aspirated and failed the screen (top cell in the first column) to the total of all patients who aspirated on the VFSS (total of the first column); therefore, 45/50 = 0.9045 divided by 50, equals 0.90. The screening test's specificity is the proportion of patients who did not aspirate and passed the screen (bottom cell in the second column) to the total of all patients who did not aspirate on the VFSSV F S S (total of second column); therefore, 30/50 = 0.6030 divided by 50, equals 0.60. False negatives are patients who passed the screen who actually aspirated on the VFSSV F S S (bottom cell of first column).

Which of the following views make up a standard videofluoroscopic swallow study? Select all that apply. A.Frontal B.Lateral C.Transverse D.Anterior-posterior

Options (B) and (D) are correct. A lateral view is best for seeing all stages of a swallow, and an anterior-posterior view shows the symmetry (or asymmetry) of the swallow.

The following components of a case history were collected from a medical record review and interview with a patient and the patient's caregiver. The patient has a prior history of all of the following conditions, but only three of them are significant predictors of an elevated likelihood of clinically significant dysphagia. Which THREE of the following historical items at the time of the referral to the SLPS L P are suggestive of an elevated likelihood of clinically significant dysphagia in a 70-year-old adult? A.The patient had pneumonia as a child and then again 10 years ago. B.The patient has a history of stroke or other neurological disease. C.The patient has been losing weight since recovering from anterior cervical spine fusion. D.The patient was cured of pharyngeal cancer with radiation therapy 30 years ago. E.The patient was intubated for surgery and routinely extubated postoperatively 24 hours before the SLPS L P consult. F.The patient has a history of spasmodic dysphonia and underwent botulinum toxin injection 10 years ago.

Options (B), (C), and (D) are correct. There are numerous studies indicating the increased dysphagia/aspiration likelihood in people with a remote history of head-neck radiation, any neurological disease or stroke, or an otherwise-unexplained weight loss.

Which THREE of the following treatment strategies is most appropriate for the patient if the SLPS L P finds the infant is safe for continued oral breastfeeding and follows up with the infant and mother with therapy and education prior to discharge home from the hospital? • A.Introducing external pacing with the infant's intake rate to support airway protection • B.Providing the mother with contact information for the local cleft lip and palate team for ongoing care and support • C.Incorporating adaptive seating for the infant during oral feedings • D.Positioning the infant in the optimal feeding position at the mother's breast • E.Completing education with the infant's mother about feeding interactions, strategies, and oral care

Options (B), (D), and (E) are correct. Providing the mother with contact information for the local cleft lip and palate team for ongoing care and support would be an appropriate strategy. Positioning an infant with either the cleft lip at the top of the mother's breast or with the cleft lip against the mother's breast can aid in achieving lip closure around the nipple and/or help prevent leakage of milk from the infant's oral cavity. Completing education with this new mother will be very important, especially considering that this mother expressed concern and anxiety surrounding feeding her infant with a cleft lip

Which THREE of the following statements are true regarding the treatment of swallowing disorders? • A.Thickened liquids cause dehydration because they require more water to be metabolized. • B.The chin-down posture eliminates thin-liquid aspiration from the pyriform sinuses. • C.Supraglottic swallow may cause cardiac arrhythmia in patients with coronary artery disease or stroke history. • D.Mass practice and task specificity are important principles of exercise therapy. • E.Periodontal disease due to poor oral hygiene increases aspiration risk in people with dysphagia. • F.Shaker and jaw-opening exercises are designed to increase the upper esophageal sphincter opening.

Options (C), (D) and (F) are correct. Studies have found that patients with strokes or coronary artery disease who perform the supraglottic swallow maneuver can develop cardiac arrhythmia, the definition of motor learning theory states mass practice and task specificity are important principles of exercise therapy, and Shaker and jaw-opening exercises are designed to increase upper esophageal sphincter opening.

Rongomaiwhenua was diagnosed with apraxia of speech (AOS). She displays an inability to follow commands and perform voluntary speech movements. Recently, she has been diagnosed with nonverbal oral apraxia (NVOA), a diagnosis that often co-occurs with AOS. With this diagnosis, it can be expected that a lesion would most likely exist in the:

Frontal and central opercula

An sslp can use a direct intervention approach or an indirect intervention approach in providing fluency treatment for a preschool child who stutters. Which of the following statements most accurately characterizes the current evidence regarding the effectiveness of these two approaches to fluency treatment for children younger than five? A evidence clearly indicates more favorable outcomes using direct intervention B evidence clearly indicates more favorable outcomes using indirect C evidence clearly supports using a combination D there is not sufficient evidence to support one of the other

D

The lowest frequency of sound present in speech (regardless of the who is speaking) is usually called the: Harmonic frequency. Fundamental frequency. Resonant frequency. Transverse frequency.

Fundamental frequency.

The context of an augmentative and alternative communication (AAC) intervention program includes all of the following aspects except: A. The setting of intervention B. The teaching format used C. The use of various instructional methods D. The information being taught

D. The information being taught

Sensitivity to sound of the normal ear of a young adult is limited to A. 30 Hz to 30,000 Hz. B. 5 Hz to 5,000 Hz. C. 7 Hz to 70,000 Hz. D. 20 Hz to 20,000 Hz.

D. 20 Hz to 20,000 Hz.

What is the dB SPL if the sound pressure ratio is 10:1? A. 100 dB SPL B. 1000 dB SPL C. 10000 dB SPL D. 20 dB SPL

D. 20 dB SPL

Approximately when is the past tense regular -ed mastered by typically developing children? A. 19-28 months B. 24-33 months C. 18-32 months D. 26-48 months

D. 26-48 months

People with flaccid dysarthria report complaints that differ from those with other types of dysarthria. These complaints can help a clinician determine a speech diagnosis when they are due to damage to a single cranial nerve. Which of the following is not a cranial nerve that may be involved in flaccid dysarthria? A. Trigeminal B. Glossopharyngeal C. Vagus D. Accessory

D. Accessory

Which of the following does not typically occur in children with cerebral palsy? A. Impaired language development B. Learning difficulties C. Academic problems D. Apraxia of speech

D. Apraxia of speech

An important component of a literacy assessment is evaluation of narrative schema knowledge—which is the knowledge of story structure. Which of the following strategies is least likely to be used by a speech-language pathologist when assessing narrative schema knowledge? A. Asking the client to retell a known story such as Charlotte's Web B. Reading a short story to the client and asking him multiple comprehension questions such as, "What happened in the end of the story?" C. Asking the client to make up a fake story about winning the lottery D. Assigning a book for the client to read and having him write a paragraph about his favorite character

D. Assigning a book for the client to read and having him write a paragraph about his favorite character

Which of the following is not a major characteristic of hyperkinesias? A. Spasm B. Hemiballism C. Dystonia D. Bradykinesia

D. Bradykinesia-means slowness of movement and is one of the cardinal manifestations of Parkinson's disease. hyperkinesia-muscle spasm. hemiballism in its unilateral form is a very rare movement disorder. It is a type of chorea Dystonia is a movement disorder in which a person's muscles contract uncontrollably.

You are observing Nathan, who is 18 months old. It is suspected that Nathan has severe developmental delays. He is producing variations in consonants and vowels from syllable to syllable with smooth transitions between his vowels and consonants. What prelinguistic stage is he currently in and what is the average age associated with the stage? A. Cooing (4-6 months) B. Jargon (10-12 months) C. Vocal play (4-6 months) D. Canonical babbling (6-10 months)

D. Canonical babbling (6-10 months)

All of the following may be expected when treating a patient with Prader-Willi syndrome, except: A. Pitch variations in voice B. Hypotonia C. Delayed receptive and expressive language skills D. Cleft lip or palate

D. Cleft lip or palate

A professor is teaching a class the anatomy of the hearing mechanism. She asks her students, "What structure houses the sensory apparatus for hearing and resides between the scala vestibuli and the scala tympani?" A. Organ of Corti B. Stapedius C. Tensor tympani D. Cochlear duct

D. Cochlear duct

A young child named Kylie was referred to an audiologist's office after she was told that she could not benefit from hearing aids because of her profound hearing loss. Her mother is extremely interested in cochlear implants and wants to know more about them. She said, "I never really thought about cochlear implants until a doctor brought that up as a possibility. I do not know much about them and need to know everything before I even consider this option for Kylie." She then gave the audiologist some information that she found online but she was not sure how accurate this information was. For example, she mentioned to the audiologist that she read that cochlear implants deliver electrical impulses to the auditory nerve, have multiple electrode arrays, have an implanted receiver, and allow the patient to discriminate between speech sounds. When reading about the potential negatives, she got overwhelmed. The audiologist made sure to stress to Kylie's mother that cochlear implants help the user perceive sound; however, she also mentioned that professionals cannot be positive as to what exactly Kylie will be able to perceive. Which of the following statements said by the mother is not true regarding cochlear implants? A. The implant is surgically implanted and delivers electrical impulses directly to the auditory nerve. B. The cochlear implant consists of a microphone, processor, external transmitter, and implanted receiver. C. All contemporary devices make use of multiple electrode arrays. D. Cochlear implants will allow the patient to discriminate between speech sounds.

D. Cochlear implants will allow the patient to discriminate between speech sounds.

Due to Siobhan's history of multiple strokes, neuroimaging has been recommended. Which of the following diagnostic tools is a type of neuroimaging that would be most useful when confirming Siobhan's diagnosis? A. Functional near-infrared spectroscopy B. Diffuse correlation spectroscopy C. C-reactive protein test D. Computerized tomography

D. Computerized tomography Diffuse correlation spectroscopy (DCS) is an emerging optical modality used to measure cortical cerebral blood flow

There are several aspects involved in studying language. These include using language as a social tool, a learned behavior, and/or a linguistic means of communication. Which of the following is not an example of linguistic aspects of learning? A. Phonology B. Pragmatics C. Semantics D. Context

D. Context

Choose the statement that correctly describes an aspect of a client with a phonological disorder that pertains to phonemic contrasts. A. Meaning distinguishing contrasts have been established. B. The accurate production of sounds is emphasized. C. The contrastive use of phonemes is realized. D. Contrastive phoneme utilities have not been realized if two or more phonemes are characterized by the same production.

D. Contrastive phoneme utilities have not been realized if two or more phonemes are characterized by the same production.

What are cartilages that are cone shaped and are located at the posterior of the aryepiglottic folds and are occasionally fused with the arytenoid cartilages? A. Cricoids B. Laminas C. Cuneiforms D. Corniculates

D. Corniculates

Which of the following is not a general language deficit of children with autism spectrum disorder? A. Inadequate response to speech B. Perseveration on certain words or phrases C. Slow acquisition of speech sound production and language D. Delayed morphosyntactic skills

D. Delayed morphosyntactic skills

An important part of treating patients with traumatic brain injuries is providing support, reassurance, information, and direction to family members. Which is most important in the middle and later stages of recovery? A. Support B. Reassurance C. Information D. Direction

D. Direction

What is a reliable indirect indicator of the severity of the brain injury? A. Gender B. Age C. Non-acceleration injuries D. Duration of unconsciousness

D. Duration of unconsciousness

Peter is a young client who you normally treat for delayed language; however, recently, he has been demonstrating dysfluencies during conversation. A lot of clinicians agree that typically developing children experience a period of dysfluent speech during the preschool years. During therapy, you decide to monitor Peter's speech for the types of dysfluencies he displays. Which of the following is a type of dysfluency that is not typical for a child experiencing normal developmental dysfluencies? A. Whole word repetitions B. Phrase repetitions C. Syllable interjections D. Final word repetitions

D. Final word repetitions

Recognition is very important in caregiver-infant attachment because it is the ability of two individuals to attend to and recognize each other's presence, as distinguished from that of others. Before therapy begins, it is important to recognize an infant's signals for recognition. Infants signal recognition through all of the following except: A. Sucking B. Crying C. Calmness D. Fixation

D. Fixation

A procedure that promotes swallowing without modifying the physiological status of the patient is known as compensatory treatment. This type of treatment requires little or no patient effort and does not cause the patient fatigue. Modification of a patient's posture is a type of compensatory treatment that promotes better swallowing and reduces aspiration. Which of the following is a characteristic of the chin-up posture? A. Recommended for patients with neurogenic dysphagia B. Teaches the client to tuck the chin to the chest during swallowing C. Helps direct food to the more efficient side of the pharynx D. Helps drain food from the oral cavity because of gravity

D. Helps drain food from the oral cavity because of gravity

Blake, a 65-year-old man, comes to an audiologist's office and complains that he has an intense burning pain close to his ear. He states that his symptoms include vertigo and facial paralysis. He said, "I have not slept well this past week. I have been constantly feeling that my balance is off and that everything is spinning. It takes a while for the feeling to eventually stop." After sitting in the office for 10 minutes, the audiologist suddenly notices that he has small sac-like bodies on his face and neck. After further examination, the audiologist finds these same sac-like vesicles in his ear canal. To understand what Blake is experiencing, the audiologist conducts a hearing exam. Results of the testing indicate that Blake has a severe bilateral high-frequency hearing loss. The audiologist's initial impression is that Blake may have had a postnatal infection that caused cochlear damage. Which of the following specific conditions does Blake most likely have? A. Meniere disease B. Measles C. Bacterial meningitis D. Herpes zoster oticus

D. Herpes zoster oticus

According to Halliday, what are four of the seven functions of communicative intent that develop between 9 and 18 months of age? A. Imaginative, interactional, attribution, overextension B. Interactional, conversational, collaborative, attribution C. Collaborative, imaginative, attribution, conversational D. Heuristic, imaginative, interactional, personal

D. Heuristic, imaginative, interactional, personal

Tumors located on the free margin of one vocal fold with only minimal extension are usually treated with a laryngectomy known as all of the following except: A. Hemilaryngectomy B. Vertical laryngectomy C. Extended hemilaryngectomy D. Horizontal supraglottic laryngectomy

D. Horizontal supraglottic laryngectomy

What disability is not recognized under the Individuals with Disability Education Act? A. Autism spectrum disorder B. Deafness C. Emotional disturbance D. Language learning disabilities

D. Language learning disabilities

You are evaluating a 5-year-old boy whose mother has a history of alcohol abuse while she was carrying her son. In your assessment, you would look for specific speech and language problems; in addition, you would look for which of the following positive or negative signs? A. Normal motor and intellectual development, normal play activities, and normal facial and skull features B. Language problems, but no speech problems, coupled with good eye contact and generally compliant behavior C. Lack of gestures, good eye contact, and lack of attachment to new people D. Low birth weight and length, behavior problems, and possible swallowing difficulties

D. Low birth weight and length, behavior problems, and possible swallowing difficulties

In preparing for feeding and swallowing with a patient with a tracheostomy tube, which of the following is recommended while conducting therapy? A. Have the patient sit in the bed at a 120-degree angle before occluding the patient's tracheostomy. B. Use cervical auscultation to determine if there is any aspiration. C. There is no need to occlude the tracheostomy during and immediately after the swallow, as there is no chance of aspiration. D. Occlude the patient's tracheostomy during and immediately after the swallow, as the exhalatory airflow after the swallow may contribute to clearance of residual food from the top of the airway, reducing the chance of aspiration after the swallow.

D. Occlude the patient's tracheostomy during and immediately after the swallow, as the exhalatory airflow after the swallow may contribute to clearance of residual food from the top of the airway, reducing the chance of aspiration after the swallow.

What bone is located at the anterior border of the nasal cavity? A. Incisive fossa B. Premaxilla C. Palatine Process D. Palatine Bone

D. Palatine Bone

You are working in a hospital with a boy named Terrence, whose medical chart indicates that he has a complete bilateral cleft lip and palate. The muscles that contribute to velopharyngeal closure through tensing or elevating the velum are damaged. Which of the following muscles help in velopharyngeal closure? A. Tensor veli palatini, levator veli palatini, salpingopharyngeus B. Stylopharyngeus, salpingopharyngeus, levator veli palatini C. Levator veli palatini, genioglossus, salpingopharyngeus D. Palatoglossus, tensor veli palatini, levator veli palatini

D. Palatoglossus, tensor veli palatini, levator veli palatini

What is a pink or white wart-like growth that can be found anywhere in the airway and make a person's voice sound hoarse, breathy, and low pitched? A. Hyperkeratosis B. Leukoplakia C. Hemangioma D. Papilloma

D. Papilloma

The following are considered nonstuttering-like disfluencies, except: A. Interjections B. Incomplete sentences C. Revisions D. Part-word repetitions

D. Part-word repetitions

What structure of the thorax provides the mechanism for air movement through muscular action? A. The mediastinum B. Bronchioles C. Alveoli D. Pleura

D. Pleura

A free morpheme can stand alone and still carry meaning. A bound morpheme must be attached to other (free) morphemes to carry meaning. Which of the following is an example of a word containing both a free and a bound morpheme? A. Mess B. Happy C. Waterfall D. Replay

D. Replay

When evaluating a child with cerebral palsy, the following symptom is often noticed: A. Increased rate of speech B. Equal stress is produced on all syllables C. Fatigue is not an issue D. Resonance difficulties

D. Resonance difficulties

A mother brings her 3-year-old daughter, Shannon, to you. The mother is a pediatrician and is worried that Shannon's "sounds are not developing the way they should." She wonders if Shannon needs speech therapy. What can you tell this mother? A. Shannon should be producing consonant clusters with 80% to 90% accuracy. B. Shannon should definitely be producing /r/, /l/, /th/, and /s/ accurately. C. Shannon should have mastered /f/ but will still be developing /p/, /m/, and /n/. D. Shannon should be in the process of mastering glides /w/ and /j/, but liquids /r/ and /l/ will probably develop later

D. Shannon should be in the process of mastering glides /w/ and /j/, but liquids /r/ and /l/ will probably develop later.

When working with a client with dementia, a clinician plans on utilizing an emotion-oriented approach aimed at reducing levels of anxiety and challenging behaviors by playing audio recordings of the voices of close relatives of the client. This type of treatment approach is known as what? A. Montessori-based treatment B. Reality orientation C. Reminiscence therapy D. Simulated presence therapy

D. Simulated presence therapy

Which type of reinforcer is exemplified by verbal praise, attention, and facial expressions? A. Primary reinforcer B. Secondary reinforcer C. Unconditioned reinforcer D. Social reinforcer

D. Social reinforcer

A kindergarten teacher refers Tomiko to you for a speech-language screening. Tomiko's first language is Korean, and she has been exposed to English for 8 months in school. The teacher is concerned because he thinks that Tomiko "has a speech problem." Which one of the following patterns (in English) would you not expect to find in a student who speaks Korean? A.Substitution of a/ae (e.g., bock/back) B.Final consonant deletion (e.g., be-/bed) C.Confusion of r/l (e.g., glow/grow, lay/ray) D.Substitution of t/k (e.g., tea/key)

D. Substitution of t/k (e.g., tea/key)

Hypotonia means

Hypotonia means decreased muscle tone. It can be a condition on its own, called benign congenital hypotonia, or it can be indicative of another problem where there is progressive loss of muscle tone, such as muscular dystrophy or cerebral palsy. It is usually detected during infancy. Also swallowing problem for down syndrome.

You are planning treatment for a child who needs to learn morphological features of language. You have selected four grammatical morphemes to teach at the word level. You will then teach each word to a training criterion. You show a stimulus card, ask a question, wait for the response, reinforce the correct response, score the response, and wait for a few seconds before presenting the next opportunity for the child to produce the target morpheme. What is this procedure called? A. The shaping method B. The successive approximation C. The naturalistic method D. The discrete trial method

D. The discrete trial method Discrete trial training (DTT) is a method of teaching in which the adult uses adult- directed, massed trial instruction, reinforcers chosen for their strength, and clear contingencies and repetition to teach new skills. DTT is a particularly strong method for developing a new response to a stimulus. Shaping can also be defined as the procedure that involves reinforcing behaviors that are closer to the target behavior, also known as successive approximations. ... The theory involves reinforcing behavior that are successively closer and closer to the approximations of the desired, or targeted, behavior.

What is the driving force behind bolus propulsion into the esophagus? A. The action of the swallowing reflex B. Peristaltic contraction and gravity C. All the mechanical movement in the oral stage of the swallow D. The pressurization of the pharynx

D. The pressurization of the pharynx

Which is not a prognostic indicator in traumatic brain injury (TBI)? A. Severity of brain injury is the magnitude and duration of alternation in consciousness. B. Time after coma during which the patient is unable to store new information and experiences in memory. C. Patient-related variables (e.g., age, substance abuse, education) D. Time that the patient passes through each level of the Rancho Los Amigos Scale (RLAS)

D. Time that the patient passes through each level of the Rancho Los Amigos Scale (RLAS)

Apraxia of speech (AOS) can be caused by any process that damages structures involved in motor speech programming. Which of the following etiologies is the most common cause of AOS? A. Degenerative diseases B. Traumatic injury C. Left hemisphere tumor D. Vascular lesions

D. Vascular lesions

When assessing pitch, the following subjective judgments are typically made, except: A. Whether the patient is monopitched B. Whether the patient is using optimal pitch C. Whether the pitch is appropriate to the patient's gender or age D. Whether the patient appropriately alters pitch to daily situations

D. Whether the patient appropriately alters pitch to daily situations

Lisa, a 19-year-old college student, was in a car accident and was airlifted to the trauma center at a local hospital. The paramedics at the accident scene had to perform an emergency intubation to permit her to breathe. A week after the accident, Lisa was discharged from the hospital and was breathing normally. A month later, she returned to the hospital complaining of hoarseness and breathiness. The laryngologist performed an evaluation and noticed that a unilateral localized inflammatory vascular lesion had developed on the vocal process of her arytenoid cartilage. The laryngologist believed that the intubation may have caused A. hyperkeratosis. B. leukoplakia. C. hemangioma. D. a granuloma.

D. a granuloma.

A 52-year-old patient visited a laryngologist and complained of lingering hoarseness that lasted over 2 weeks and voice changes which included a low, raspy voice, chronic cough, discomfort while swallowing, and post nasal drip. She mentioned to the physician that she was in no pain but mentioned that she talked loudly, smoked, drank alcohol heavily, had gastroesophageal reflux disease (GERD), and was often exposed to dust at work. The physician diagnosis was: A.a laryngeal web. B.vocal nodules. C.polyps. D.chronic laryngitis.

D. chronic laryngitis.

Case management for an elderly patient includes the following targets: establishing a simple routine, using various reminders, writing down a list of things to do every morning, and writing a checklist of things to do before leaving the house. Such a plan is most appropriate for a patient with A. Broca's aphasia. B. Wernicke's aphasia. C. right hemisphere syndrome. D. dementia.

D. dementia.

The concrete operations stage as delineated by Piaget states that the child A. displays the ability to think and speak in the abstract. B. underextends words. C. is able to make if/then statements. D. employs logical causality.

D. employs logical causality.

Senbo is a third-grade student who speaks Afrikaans. Her teacher refers her for assessment because she is having academic difficulty. To evaluate Senbo's language skills, you use dynamic assessment, an alternative approach in which the clinician A. collects various kinds of work samples from the child being evaluated. B. compares a child's performance with the established norms. C. seeks to sample the child's speech and language in naturalistic settings. D. evaluates her over time in a test-teach-retest format.

D. evaluates her over time in a test-teach-retest format.

HIPPA's Privacy Rule requires organizations to designate a privacy officer to: A. establish measures for obtaining a patient's written consent to use or disclose his or her health information for purposes alternative to treatment and payment. B. improve or amend policies to uphold a patient's rights under HIPPA. C. inform a patient about HIPPA. D. manage all activities related to HIPPA.

D. manage all activities related to HIPPA.

During your evaluation, you hear Tommy say things like "My mom is skinny than her mom" and "Our house is the big on the block." This shows difficulty with A. semantics, specifically with possessives and indefinite pronouns. B. semantics, specifically with comparatives and spatial relationships. C. morphology, specifically with personal pronouns and contractible auxiliary forms. D. morphology, specifically with comparatives and superlatives.

D. morphology, specifically with comparatives and superlatives.

A father tells you that he is concerned about his son Adam, who has had many episodes of otitis media with effusion. Adam has taken antibiotics for the last 7 months, but testing reveals that he continues to have middle ear fluid, although he is not sick. You refer Adam and his father to a pediatrician, who asks an ear, nose, and throat doctor to perform a surgical procedure in which a small incision will be made in Adam's tympanic membranes to relieve pressure. This procedure is known as A. pressure-equalizing tubes. B. myringoplasty. C. otosclerosis. D. myringotomy.

D. myringotomy.

A medical chart indicates that a patient's true vocal folds adduct instead of abduct during inhalation. The medical information also indicates that at times the patient's vocal folds remain closed throughout the respiratory cycle. According to the speech-language pathologist, the patient has A. unilateral vocal fold paralysis. B. bilateral vocal fold paralysis. C. spasmodic dysphonia. D. paradoxical vocal fold motion.

D. paradoxical vocal fold motion.

The Blom-Singer prosthetic device is used by laryngectomees to A.clean the surgically created fistula. B.shunt air from the esophagus to the trachea so that the salpingopharyngeus muscle vibrates during inhalation. C.assist in the development of competent esophageal speech. D.shunt air from the trachea to the esophagus so that the patient can speak on pulmonary air that enters the esophagus.

D. shunt air from the trachea to the esophagus so that the patient can speak on pulmonary air that enters the esophagus.

You refer a patient to an audiologist because you suspect that the patient might have a hearing loss. You ask the audiologist to inform you about the patient's threshold of hearing for selected frequencies. In response to your request, the audiologist will A. tell you the intensity at which tones are faintly heard at least 75% of the time they are presented in a pure-tone hearing test. B. inform you of the intensity level at which tones are faintly heard at least 25% of the time in a speech reception threshold test. C. inform you of the intensity level at which tones are faintly heard at least 50% of the time in a masked speech reception threshold test. D. tell you that the threshold of hearing is the quietest sound a human can detect.

D. tell you that the threshold of hearing is the quietest sound a human can detect.

Standardized tests are limited in their usefulness because A. they do not allow for comparative evaluation of performance. B. they draw nationally representative samples that may not represent local samples. C. they are not always accepted as a basis to determine service eligibility in schools. D. they sample participants (children) and responses in a limited manner.

D. they sample participants (children) and responses in a limited manner.

A 9-year-old child was seen for an evaluation because of a major dysfunction in his gastrointestinal tract. The child had short bowel syndrome, resulting in the removal of a major part of the intestines. The gastroenterologist most likely would recommend A. a jejunostomy tube (J-tube). B. a duodenal tube (duo-tube). C. a gastrojejunal tube (GJ-tube). D. total parenteral nutrition (TPN).

D. total parenteral nutrition (TPN). Total parenteral nutrition (TPN) is a method of feeding that bypasses the gastrointestinal tract

You are working with a 7-year-old Spanish-speaking girl, Rosa, who is in the process of learning English as a second language. Which of the following would not be typical for her in terms of predictable productions based on Spanish influence? A. Insertion of schwa before word-initial /s/ clusters B. Devoicing of final consonants C. b/v substitutions D. v/f substitutions in medial position of words

D. v/f substitutions in medial position of words

A 43-year-old high school football coach comes to you for an evaluation. He states that he has been hoarse for approximately 10 months. A subsequent medical evaluation reveals that he has bilateral lesions on the anterior third of the vocal fold. A naso-endoscopic evaluation confirms that the patient has developed A. polyps. B. traumatic laryngitis. C. contact ulcers. D. vocal nodules.

D. vocal nodules.

This theory identifies the process of therapy as an explanation of the process of language in order to achieve a desired function of language. Additionally, the theory defines aphasia as a series of processing deficits that can be specific to different modalities. Which theory is this? A. Schuell's (1964) stimulation approach B. Localization: Boston school C. Neurolinguistics D. Cognitive neuropsychological

D. Cognitive neuropsychological

Which of the following is not a perceived abnormality in rate and prosody in apraxia of speech? A. Rate for utterances more than one syllable in length is usually slow B. Silent pauses precede initiation of speech C. Prolonged consonants and vowels D. Equalized pitch, duration, and loudness contours within utterances

D. Equalized pitch, duration, and loudness contours within utterances

You are performing a diagnostic evaluation on Belle, a 4-year-old girl, who was brought to the clinic by her mother due to concerns about her daughter's speech. During the evaluation, you note that Belle often produces /t/ instead of /k/ in the initial position of words. She also produced /w/ for /l/ in the word, leaf, and /d/ for /dÊ'/ in the word jump. Which of the following is an active phonological process that is not appropriate for Belle's age and should be addressed during therapy? A. Gliding B. Final consonant deletion C. Stopping /dÊ'/ D. Fronting

D. Fronting

Severe gastroesophageal reflux has been implicated in all of the following conditions except? A. Gastrointestinal bleeding B. Barrett's esophagus C. Cancer D. Inability to phonate

D. Inability to phonate

When apraxia of speech is characterized by limited vocalizations, there are some techniques that may be successful during speech therapy. Which of the following is not a technique that can elicit vocalizations in patients with apraxia who are mute? A. Singing B. Automatic speech tasks C. Pushing on the abdomen D. Self-learning

D. Self-learning

What specifically prohibits practitioners from charging for future services or incomplete services? A. Joint Commission on Accreditation of Health Care Organizations (JCAHO) B. Health Insurance Portability and Accountability Act (HIPPA) C. Local Coverages Determinations (LCD) D. The List of Excluded Individuals and Entities (LEIE)

D. The List of Excluded Individuals and Entities (LEIE)-provides information to the health care industry, patients and the public regarding individuals and entities currently excluded from participation in Medicare, Medicaid, and all other Federal health care programs. Local Coverage Determinations (LCDs) A local coverage determination (LCD) is a decision made by a Medicare Administrative Contractor (MAC) on whether a particular service or item is reasonable and necessary, and therefore covered by Medicare within the specific region that the MAC oversees The mission of the Joint Commission on Accreditation of Healthcare Organizations is to continuously improve the safety and quality of care provided to the public through the provision of health care accreditation and related services that support performance improvement in health care organizations.

The definition of stuttering as "speech that contains 5% or more disfluencies" is based on A. an etiological theory of stuttering. B. the notion that stuttering is a certain moment and an expert judges it as such. C. the psychodynamic view of stuttering. D. certain listener evaluation studies.

D. certain listener evaluation studies.

2. Speech sounds are classified in various ways. Select the statement that is correct according to the classification mentioned in each answer. A. Voicing makes a distinction between vowels and consonants. B. The cognate pairs distinction is based on the place of articulation. C. Among others, speech sounds bilabials and labiodentals are based on the manner of articulation. D. Among others, speech sounds affricates, stops, and liquids are based on the manner of articulation. E. Among others, speech sounds glides and stops are based on the place of articulation.

D. Among others, speech sounds affricates, stops, and liquids are based on the manner of articulation. E. Among others, speech sounds glides and stops are based on the place of articulation.

43. Those professionals who conduct language treatment according to the principles of the behavioral theory do what? A. Focus treatment sessions around Vygotsky's principles B. Assess cognitive precursors to language and facilitate the development of those precursors C. Focus treatment on auditory processing skills D. Believe that language can be taught by targeting any observable behavior and manipulating the elements of a stimulus, a response, and reinforcement E. Focus on increasing children's syntactic skills

D. Believe that language can be taught by targeting any observable behavior and manipulating the elements of a stimulus, a response, and reinforcement

5. Which of the following statements is false regarding Public Law 99-457? A. It increased federal support for services to children with disabilities 3 to 6 years of age and provided funding for infants and toddlers. B. It requires the development of individualized family service plans. C. It allows at-risk preschool children (not just those with documented disabilities) to be eligible for special education services. D. It requires states to report preschool children by disability category. E. It was intended to provide early intervention that would reduce the number of children requiring special education services in later years.

D. It requires states to report preschool children by disability category.

An SLP has targeted the phonological process of "stopping on initial fricatives" for remediation and is using the word "shoes" to establish the new behavior. The SLP now wishes to investigate whether the speaker can generalize the newly learned pattern to untrained words. If it is assumed that generalization will occur on words whose phonetic characteristics are most like the trained word "shoes," which of the following words should be selected? (A) Shouting (B) Fished (C) Ocean (D) Shook

D. Shook

88. You are working with a 7-year-old Spanish-speaking girl, Rosa, who is in the process of learning English as a second language. Which of the following would not be typical for her in terms of predictable productions based on Spanish influence? A. Insertion of schwa before word-initial /s/ clusters B. Devoicing of final consonants C. b/v substitutions D. v/f substitutions in medial position of words E. d/th substitutions (e.g., dis/this)

D. v/f substitutions in medial position of words

In the scientific method, what is the experiment-first-and-explain-later approach?

Deductive reasoning

Deductive reasoning

Deductive reasoning, or deduction, starts out with a general statement, or hypothesis, and examines the possibilities to reach a specific, logical conclusion, scientific method

The researcher has accepted a stance that supports information-processing models of learning. This researcher believes that child language learning is

Dependent on short- and long term memory stores

Direct selection

Direct selection: The person using AAC accesses a target by directly pointing to a symbol, picture, or object with a body part (e.g. finger, hand, eye gaze) or adapted tool

Phonological processes that disappear before and afterr 3

Disappear: reduplication, weak/unstresed syllable deletion, consonant assimilation, prevocalic voicing, fronting of velars, fronting of velars, final consonant deletion, diminutization persist: final consonant devoicing, consoant cluster reduction, stopping, epenthesis, gliding, depalatization, vocalization

40. Approximately when is the past tense regular -ed mastered by typically developing children? A. 19-28 months B. 24-33 months C. 18-32 months D. 26-48 months E. 24-26 months

E. 24-26 months

86. A high school teacher refers a Mandarin-speaking 16-year-old to you for an evaluation. The student and his family came to the United States 2 years ago from China. The teacher says that the student does well academically, but she shares that she has difficulty understanding him when he speaks. When you screen the student, you find some articulation and language differences. Which one of the following would not be predictable based on the student's first language of Mandarin? A. Substitutions of t/th (e.g., tin/thin) B. Final consonant deletion C. Epenthesis in words with consonant blends D. Confusions of /r/ and /l/ E. Substitutions of f/th (e.g., fick/thick)

E. Substitutions of f/th (e.g., fick/thick)

42. The concrete operations stage as delineated by Piaget states that the child A. has difficulty with classification skills B. displays the ability to think and speak in the abstract C. underextends words D. is able to make if/then statements E. employs logical causality

E. employs logical causality

93. You refer a patient to an audiologist because you suspect that the patient might have a hearing loss. You ask the audiologist to inform you about the patient's threshold of hearing for selected frequencies. In response to your request, the audiologist will A. inform you of the intensity level at which tones are faintly heard at least 50% of the time they are presented in a pure tone hearing test B. tell you the intensity at which tones are faintly heard at least 75% of the time they are presented in a pure tone hearing test C. inform you of the intensity level at which tones are faintly heard at least 25% of the time in a speech reception threshold test D. inform you of the intensity level at which tones are faintly heard at least 50% of the time in a masked speech reception threshold test E. tell you the intensity at which tones are faintly heard at least 50%-60% of the time in a spondee word test

E. tell you the intensity at which tones are faintly heard at least 50%-60% of the time in a spondee word test

Primarily, what is the basic structure of the English language? Amalgamating or polysynthetic Inflectional or fusional Agglutinating or synthetic Isolating or analytic

English has primarily* an inflectional/fusional language structure; that is, it adds regular prefixes and suffixes to words to vary their meaning but also varies words for this purpose. Due to its many irregular forms, English and some other inflected languages are hard to learn. Languages with isolating/analytic structures include Chinese, Indonesian, and Creole and Pidgin languages. These feature invariable words and strict syntactical rules. (*Note: While English is primarily inflectional, it has begun to evolve some analytic/isolating features.) Adults find analytic languages easier to learn than children do. Agglutinating/synthetic languages include Turkish, Japanese, Finnish, Tamil and others. They are easy for children to learn. Their prefixes and suffixes are very regular. Amalgamating/polysynthetic languages include Basque and many Native American tribal languages. They feature complex words, each imparting as much information as a sentence and are very hard to learn.

Formant frequencies for vowels do NOT vary by age or gender. True False

False

Harmonics are integer multiples of F2. True False

False

Phonetic context has no bearing on how much a particular acoustic feature helps us with speech perception. True False

False

Speech perception only involves a bottom-up process. True False

False

Focused stimulation

Focused stimulation is a child speech therapy approach to repeat a word or phrase multiple times in a conversation to facilitate comprehension and possible language production. The caregiver chooses a target word and tries to incorporate the word in normal conversation. It should encourage the comprehension of the word more than the production of the word.

Irwin and Weston's paired stimuli approach

Assumption: an acceptably produced target phoneme can be generalized from selected phonetic contexts to a variety of phonetic contexts through a program of behavior modification• Used to teach one speech sound at a time through the use of a key word

35. What is an octave? A. The amount of molecular displacement per unit of time B. The amount of time between cycles C. An indication of the interval between two frequencies D. A measure of the magnitude (intensity, strength) of the sound signal E. The unit of measure for frequency; it is the same as the cycles per second

C. An indication of the interval between two frequencies

8. A researcher who was developing a new test of language acquisition in children correlated the scores of children studied with the scores on an established test of known validity. What kind of validity is this? A. Predictive validity B. Content validity C. Concurrent validity D. Construct validity E. Criterion validity

C. Concurrent validity

28. What are cartilages that are cone-shaped and are located under the mucous membrane that covers the aryepiglottic folds called? A. Cricoids B. Laminas C. Cuneiforms D. Corniculates E. Arytenoids

C. Cuneiforms

54. With regard to vowel formant the vowel quadrilateral demonstrates which generality? -First formant frequency F1 is influenced mostly by the height of the tongue body -LOw vowels have higher F1 values than do high vowels -third formant frequency F3 is influenced by the interaction of tongue position and heigh =F1 is affected most by the size of the oral opening

C. During low vowel production the descended positioning of the tongue effectively decreases the size of the pharyngeal resonanator and thus raises the value of F1, the size of the pharyngeal space is inversly correlated to the value of F1

which of the following formants typically characterizes a high vowel? a. high-frequency second formant (F2) b. high-frequency first formant (F1) c. low-frequency first formant (F1) d. low-frequency second formant (F2)

C. F1 is inversely associated with tongue height, such that high vowels tend to have low F1 frequencies

00. What would you call an experimental design involving one or a few subjects? A. Case study design B. Ex-post facto design C. Single-subject experimental design D. Single correlational design E. Single-subject case study design

C. Single-subject experimental design

81. You have been assigned to a school district where many of the children come from culturally and linguistically diverse (CLD) backgrounds. You have attended some workshops on culturally sensitive and valid assessment of the language skills of these students. What are you likely to remember from your workshops? A. It is best to evaluate CLD students in English with formal tests. B. The ideal practice is to use standardized tests that are translated into the students' first languages by interpreters. C. The development of formal tests has grown out of a framework that is Western, literate, and middle class. D. If the results of an assessment show that the CLD student has difficulty in English but not the primary language, you should enroll him or her for intervention to improve English skills. E. You should never alter the administration of standardized tests when you use these tests with CLD students.

C. The development of formal tests has grown out of a framework that is Western, literate, and middle class.

61. A silent prolongation is A. the same as a silent pause B. the same as a filled pause C. the same as an articulatory posture without voicing D. normal regardless of its frequency E. none of the above

C. the same as an articulatory posture without voicing

Babies typically imitate various adult speech sounds around which ages? Birth to 3 months 1 year to 18 months 7 months to 1 year 4 to 6 months

Babies typically demonstrate imitation of the speech sounds they hear between the ages of 7 months and 1 year. Between birth and 3 months, they typically coo, goo, differentiate their crying according to different needs, and smile at parents. From around 4 to 6 months, they normally babble including speech sounds like /m/, /p/, and /b/; laugh and chuckle; gurgle; and vocally express unhappiness and excitement. From 1 year to 18 months, children normally begin uttering words, use one- and/or two-word questions and phrases, and acquire many different word-initial consonants.

Baker and Ryan's Monterey Articulation Program

Based on behavioral principles and programming learned concepts 1) Establishment Phase: 18 steps; target sounds produced in isolation > nonsense syllables > words > phrases >sentences>contextual reading > story narration > picture description > conversational speech 2) Transfer Phase: 15 steps; home training, clinician training in different settings and training in the classroom 3) Maintenance Phase: 5 steps; periodic rechecks to help the child maintain accuracy in the production of newly acquire d sounds. Good for those that need a highly structured motoric approach. Repetition and motor practice are strongly emphasized. Speech sounds are learned motor behaviors (Bernthal & Bankson, 1998)

What can develop on the acoustic nerve and ultimately result in a sensorineural hearing loss due to decrease in nerve conduction of sound impulses to the brain? A. Brainstem lesions B. Acoustic neuroma C. Bony growths D. Bacteria

B. Acoustic neuroma

Giacino, Katz, and Schiff (2007) described three levels of disordered consciousness when determining Level of Consciousness and Responsiveness to Stimulation. Which is not considered a level of disordered consciousness? A.Coma B.Alert and oriented C.Vegetative state D.Minimally conscious state

B. Alert and oriented

The following characteristics are important regarding the adaptation effect in persons who stutter, except: A. The greater the time interval between readings, the less the degree of adaptation. B. All persons who stutter exhibit an adaptation effect. C. Adaptation is generally seen in both persons who stutter and typically fluent speakers. D. There is no transfer of adaptation from one passage to another.

B. All persons who stutter exhibit an adaptation effect.

The Western Neuro Sensory Stimulation Profile (WNSSP) was designed to do what? A. Compensate for some of the deficiencies of the Glasgow Coma Scale (GCS) B. Assess cognitive function and to monitor changes in a patient with a brain injury who is severely impaired and slow to recover C. Determine levels of consciousness of a patient D. Assess how responsive a patient is

B. Assess cognitive function and to monitor changes in a patient with a brain injury who is severely impaired and slow to recover

A father comes to you regarding his daughter, who is 8 months old. The daughter's hearing loss is bilateral, and she is profoundly deaf. The father states that he wishes for his daughter, as she grows older, to "fit in with children with normal hearing." He is interested in any possible amplification and says that he wants his daughter to lead a life that is "as normal as possible." Which training approach would best fit this father's wishes? A. Total communication B. Aural/oral method C. Manual approach D. Rochester method

B. Aural/oral method

A child with cerebral palsy is seen for an evaluation. During the assessment, Cara, the speech-language pathologist notices that the child's jaw is impaired relative to other structures. Cara is trying to focus on improving the child's speech by increasing tongue movement to improve articulatory proficiency. Cara would use the following to determine whether the child would be a candidate for constraint-induced movement therapy: A. Modified barium swallow study B. Bite block C. Tongue depressor D. Endoscopic evaluation

B. Bite block

Anthony, a 40-year-old, has been diagnosed with Wallenberg syndrome after suffering from a cerebral vascular accident. Based on the diagnosis, you should expect Anthony to have damage to which part of his brain? A. Frontal lobe B. Brainstem C. Temporal lobe D. Cerebellum

B. Brainstem

Brandon, a 72-year-old client with aphasia, can indicate his needs by pointing to items and objects. He does not initiate communication and requires extra support in routine conversation. What type of communicator will he be considered? A. Augmented-input communicator B. Controlled-situation communicator C. Comprehensive communicator D. Basic-choice communicator

B. Controlled-situation communicator

A high school teacher has referred Ashley to you for assessment. Ashley is 17 years old, and she possibly has a language impairment. When you assess Ashley, which specific problems will you look for as evidence of a language impairment? A. Sentences that are average in length but contain omissions of bound morphemes B. Difficulty using cohesion devices (e.g., therefore, for example) C. Normal ability to maintain a conversation, but some use of non sequiturs D. Normal ability to use figurative language, but difficulty with concrete language

B. Difficulty using cohesion devices (e.g., therefore, for example)

Shamim, your 74-year-old patient, has just been diagnosed with idiopathic Parkinson's disease. The patient's disability seems mild to moderate; however, there is unsteadiness as she turns or when pushed from a stable standing position with her feet together and eyes closed. On the Parkinson's Disease Disability Scale, how would you categorize Shamim's disability? A.Bilateral or midline involvement B.First sign of impaired righting reflexes C.Unilateral involvement only D.Fully developed, severely disabling disease

B. First sign of impaired righting reflexes

Your client is having difficulty appropriately using present progressive verbs. During play, you repeatedly model the correct targets without correcting incorrect productions made by the client. This strategy is called: A. Extension B. Forced stimulation C. Milieu teaching D. Expansion

B. Forced stimulation

Within-subject design

A within-subject design is a type of experimental design in which all participants are exposed to every treatment or condition. The term "treatment" is used to describe the different levels of the independent variable, the variable that's controlled by the experimenter

What five components should a clinician assess for a client with cerebral palsy? A. Respiration B. Phonation C. Rhythm D. Audiological examination E. Oculomotor examination F. Speech effectiveness G. Abnormal reflexes H. Oral-motor examination

A, B, C, F, H

What are the two most prominent air-filled cavities that compose the outer ear and have a resonant frequency to which they respond best? A. Concha B. Tympanic cavity C. Ear canal D. Ossicles E. Helix F. Anti-helix

A, C

What are the two subdivisions of traumatic brain injury? A. Penetrating B. Subdural hematoma C. Closed head injury D. Open head injury

A, C

You evaluated a child and have made the following conclusions: [r] is not stimulable at any level, [s] and [z] are stimulable at the sound and word level, and [θ] and [ð] are stimulable at the sound level. What two sounds should be initiated first in trial probe therapy? A. [s] B. [r] C. [z] D. [θ] E. [ð]

A, C

A treatment procedure is selected or rejected based on the level of evidence of the particular technique. What level of evidence is described if a technique has been experimentally evaluated with a control group or single-subject design with control conditions? A. Unreplicated B. Replicated C. Uncontrolled case studies D. Controlled experimental studies

A, D

You are conducting a language sample with an adolescent who speaks African American English (AAE). Which of the following utterances would be an example of the use of the perfective construction been to indicate an action that took place in the distant past? A. "I been had chicken pox when I was 5." B. "Our family been gonna see a movie." C. "My grandparents be watchin' TV." D. "We don't have no more Halloween candy for y'all."

A. "I been had chicken pox when I was 5."

You are evaluating the language skills of Pascal, who has come to the United States with his family from Mexico. As you are gathering a language sample, you hear many utterances that reflect the transfer of Spanish to English. Which of the following would be a typical utterance for a child from a Spanish-speaking home who is learning English in elementary school? A. "This balloon is more big." B. "I don't not have no more balloons." C. "The big house be red." D. "The girl's book done be gone."

A. "This balloon is more big."

You are assessing a preschool child who comes from an AAE-speaking home. Which of the following utterances reflects typical patterns of AAE? A. "You was helping me." B. "They be gonna there." C. "It not Spiderman cape be." D. "Them kids, not they be havin' fun."

A. "You was helping me."

As previously stated, Sydney has trouble swallowing. During the modified barium swallowing study, the clinician also noted that he has a delay in the pharyngeal stage of swallowing. The clinician observed that he had a 5-second delay before the pharyngeal swallow was triggered. The clinician noted in the chart that the average duration of a normal pharyngeal swallow is A. 1 second. B. 2 seconds. C. 3 seconds. D. 4 seconds.

A. 1 second.

Which of the following approaches provides the most useful challenges for an augmentative and alternative communication (AAC) user? A. A community-referenced approach along with inclusion in age-level classrooms B. An individualized and exclusion approach C. Inclusion in non-age-appropriate classrooms D. An environmentally referenced approach along with exclusion in age-level classrooms

A. A community-referenced approach along with inclusion in age-level classrooms

Within Huntington's disease, there are two neurotransmitters responsible for controlling choreiform movements. Choreiform movements mimic dance. What are the two neurotransmitters lacking in a person with Huntington's disease? A. Acetylcholine and gamma amino butyric acid (GABA) B. Acetylcholine and dopamine C. Dopamine and gamma amino butyric acid (GABA) D. Gamma amino butyric acid (GABA) and serotonin

A. Acetylcholine and gamma amino butyric acid (GABA)

Periodic waveforms can be thought of as a series of sinusoids. The difference between waveforms and spectrums is the measurements on the y axis and the x axis. What two measurements are needed for the x axis and y axis in a waveform? A. Amplitude and phase B. Amplitude and time C. Amplitude and frequency D. Time and phase

A. Amplitude and phase

Patients who have undergone total laryngectomy are not in danger of: A. Aspiration of food or liquid during swallow B. Pseudoepiglottis C. Lesions in pyriform sinus D. Swallowing difficulties

A. Aspiration of food or liquid during swallow

The following classes are part of which symbol system? Pictographs, ideographs, arbitrary, and international: A. Blissymbols B. PICSYMS C. Pictogram Ideogram Communication (PIC) D. Core Picture Vocabulary

A. Blissymbols

Which of the following could be a technique used in therapy when treating a pediatric case of tongue protrusion/thrusting? A. Change feeing positions; try head slightly reclined. B. Encourage lip closure around the spoon. C. Firmly hold the jaw in place. D. Apply tapping of the lips and nose.

A. Change feeing positions; try head slightly reclined.

What is the final stage of rehabilitation for many patients with traumatic brain injury? A. Community integration B. Orientation training C. Cognitive-communicative interventions D. Memory improvement programs

A. Community integration

Oliver, an 88-year-old patient, suffered from a stroke. He complains that he does not have sensation in the posterior one-third of the tongue. Damage to which cranial nerve would cause loss of general sensation of the posterior one-third of the tongue? A. Cranial Nerve IX - Glossopharyngeal Nerve B. Cranial Nerve X - Vagus Nerve C. Cranial Nerve XI - Accessory Nerve D. Cranial Nerve XII - Hypoglossal Nerve

A. Cranial Nerve IX - Glossopharyngeal Nerve

Language and communication impairments of individuals with traumatic brain injuries are often due to impairments in basic cognitive skills, including attention, memory, and executive functioning. These various impairments are most effectively and efficiently treated through _________. A. Direct treatment targeting social and interpersonal (pragmatic) aspects of communication B. Indirect treatment targeting visual processing C. Pharmacologic intervention D. Sensory stimulation treatment

A. Direct treatment targeting social and interpersonal (pragmatic) aspects of communication

During vocal fold vibration, what are the two forces needed for vibration? A. Elasticity and inertia B. Elasticity and energy C. Inertia and power D. Power and energy

A. Elasticity and inertia

Magdalena, a 10-year-old Austrian student, recently moved to your school. She was diagnosed by the speech-language pathologist at her previous school with selective mutism. You have never treated a client with selective mutism. With all new diagnoses, it is important to educate yourself on the general treatment options. Her parents report that Magdalena does speak at home when the immediate family is around. Additionally, once an unfamiliar person is present, Magdalena seems very tense and anxious. She was able to warm up to the speech-language pathologist from the previous school within a month of therapy; however, she received therapy twice a week. You decide to look up strategies that will be the best fit for Magdalena. Which of the following strategies is most appropriate for her? A. Exposure-based practice B. Systematic desensitization C. Stimulus fading D. Contingency management, positive reinforcement, and shaping

A. Exposure-based practice

Billy is a 16-year-old who recently suffered a traumatic brain injury as a result of a car accident. Billy remained in a coma for several days, and recently became minimally alert. During the early stages of treatment, the speech-language pathologist should focus on: A. Getting Billy to respond to sound, touch, or smell B. Helping Billy understand what day it is C. Finding ways to improve Billy's memory D. Working on Billy's social skills

A. Getting Billy to respond to sound, touch, or smell

Sam, a 4-year-old boy, recently started to use an augmentative and alternative communication (AAC) device to aid in his communication with others. Although he uses an AAC device, his cognition is within normal limits. Which of the following is true about Sam's language ability? A. He may be delayed in expressive language because of the AAC supports he has been given. B. He should not have the same receptive language as a typically developing child. C. He should not be following the same communication milestones as a typically developing child. D. He will likely have difficulties with phonology and syntax.

A. He may be delayed in expressive language because of the AAC supports he has been given.

A patient underwent a partial laryngectomy as a form of management for laryngeal cancer. You are explaining to a graduate student clinician that the area surrounding the larynx, including the pharyngoesophageal junction, right and left pyriform sinuses, lateral and posterior hypopharyngeal walls, and the post-cricoid region, is known as the: A. Hypopharynx B. Supraglottis C. Subglottis D. Pharynx

A. Hypopharynx

An individual who sustained a traumatic brain injury, presented with articulatory and phonological deficits. In this case, the injury was probably sustained in all of the following, except: A. Hypothalamus B. Cerebellum C. Peripheral nerves D. Brainstem

A. Hypothalamus

Patients have the right to change their own protected health information (PHI). Under what circumstance would a health-care provider deny a patient's request to amend his or her PHI? A. If the provider already created an accurate and complete document B. If the document is already part of the set of records C. If providers created themselves without any other contributor D. If the record was previously inspected by a governing agency

A. If the provider already created an accurate and complete document

Both the American Speech-Language-Hearing Association (ASHA) and state governments regulate the practice of speech-language pathology. Clinicians often have to meet different requirements for different professional settings. Among the following choices, which statement is incorrect? A. In many states, it is not essential to get the ASHA certification to work as a clinician in public schools. B. In most states, both the state licensure and ASHA certification are required to work as an SLP in public schools. C. Although widely recognized by employers, the ASHA certifications do not have the legal authority of state licensures. D. A state education department's credential is not the same as the same state's licensure.

A. In many states, it is not essential to get the ASHA certification to work as a clinician in public schools.

When establishing eye contact and eye gaze between the caregiver and infant, which of the following is not a type of gaze? A. Interactive gaze B. Social smile C. Gaze coupling D. Diectic gaze

A. Interactive gaze

Select the statement that applies to the normal distribution. A. It is based on the arithmetic mean of scores or values. B. It is based on the statistical mode of the scores C. In one, 14.13% of the scores fall within one standard deviation below the mean. D. The 50th percentile is equivalent to the mode.

A. It is based on the arithmetic mean of scores or values.

Select the statement that best characterizes the discrete trial procedure. A. It is the most researched procedure, and it is useful in establishing the skills, but it may not promote generalization to natural settings. B. It is not useful in any stage of treatment or for any client because it is too structured. C. It is useful in all stages of treatment, including the final stages in which discourse is taught. D. It is one of the least researched methods of treatment.

A. It is the most researched procedure, and it is useful in establishing the skills, but it may not promote generalization to natural settings.

If the hyoid bone is partially or completely removed secondary to cancer, which of the following would be damaged? A. Laryngeal suspension and elevation B. False vocal fold constriction C. Thyroid tilt D. Epiglottic inversion

A. Laryngeal suspension and elevation

What are the two steps of developing contrastive stress? A. Maintaining a single prosodic element and prosodic integration of two words into a one-tone unit B. Maintaining a single prosodic element and prosodic integration of two words into a two-tone unit C. Prosodic integration of two words into a one-tone unit and maintaining a single prosodic element D. Prosodic integration of two words into a two-tone unit and maintaining a single prosodic element

A. Maintaining a single prosodic element and prosodic integration of two words into a one-tone unit

Spectrums include frequencies that need to be attenuated as well as frequencies that are allowed to pass through a filter without being dampened. Which of these is not a type of filter used for spectrums? A. Mid-pass filter B. Low-pass filter C. High-pass filter D. Band-rejection filter

A. Mid-pass filter

Which of the following is considered a technique for the treatment of articulation and phonological disorders for a patient with cerebral palsy A. Modifying or eliminating inappropriate and ineffective compensatory postures B. Assessing the child's developmental milestones C. Treating velopharyngeal incompetence D. Considering how much the brain damage has affected motor planning

A. Modifying or eliminating inappropriate and ineffective compensatory postures

Which of the following is a main characteristic of hyperkinetic dysarthria? A. Myoclonus B. Decreased swallowing C. Disfluencies D. Slurred speech

A. Myoclonus

Which of the following is an approach that promotes natural, functional, and conversational communication for teaching language to children? This approach uses natural consequences as reinforcers. A. Naturalistic child language teaching method B. Milieu teaching C. Mand-model method D. Direct language treatment approaches

A. Naturalistic child language teaching method

The perturbation theory states that if a change in cross sectional area is applied (a perturbation), the acoustic effect depends on proximity to a node or an antinode. Which of the statements below is true about antinodes and nodes concerning formant frequencies? A. Near an antinode the formant frequency lowers; near a node the formant frequency rises. B. Near an antinode the formant frequency stays the same; near a node the formant frequency rises. C. Near an antinode the formant frequency rises; near a node the formant frequency lowers. D. Near an antinode the formant frequency lowers, near a node the formant frequency stays the same.

A. Near an antinode the formant frequency lowers; near a node the formant frequency rises.

Which of the following is a prosthetic effort that is appropriate for patients with ataxic dysarthria? A. Neck brace or cervical collar B. Nose clip or nasal obturator C. Palatal lift D. Vocal amplifier

A. Neck brace or cervical collar

The Joint Commission on Accreditation of Health Care Organizations (JCAHO) uses tracer methodology as a primary way to evaluate if organizations are complying with standards. What are the two types of tracers? A. Patient care tracers and patient system tracers B. Primary care tracers and patient system tracers C. Patient safety tracers and patient system tracers D. Patient care tracers and management system tracers

A. Patient care tracers and patient system tracers

You are evaluating a 15-year-old patient who was in an all-terrain vehicle (ATV) accident and sustained a traumatic brain injury (TBI). When evaluating this patient's memory, she mentions that she has no memory of events following the accident prior to and immediately after going to a hospital. You explain that this may be due to: A. Pretraumatic amnesia B. Posttraumatic amnesia C. Orientation to situation D. Comprehension deficits

A. Pretraumatic amnesia

Which of the following is an emergent literacy skill, important for preschool children? A. Print awareness B. Letter identification C. Sound-letter association D. Reading fluency

A. Print awareness

A 72-year-old patient has been depressed since her husband passed away 2 months ago. The onset of her symptoms was short and abrupt. When memory was tested, she often stated that she did not know the answer to a question. Her attention and concentration appeared to be intact; however, she seemed upset and distressed during the evaluation. After extensive testing, the psychiatrist diagnosed the patient with: A. Pseudodementia B. Dementia C. Pick's disease D. Alzheimer's disease

A. Pseudodementia pseudodementia (otherwise known as "depression-related cognitive dysfunction") is a condition where mental cognition can be temporarily decreased.

Tumors in the larynx may be managed primarily by surgery or: A. Radiotherapy B. Chemotherapy C. Layngectomy D. Partial laryngectomy

A. Radiotherapy

An investigator carries out a study to answer the question of whether an increased rate of sibling speech causes an increase in the frequency of stuttering in children. After pretesting rates of stuttering in selected children and the speech rate of their siblings, the investigator tells the siblings in the control group to speak as they normally would at home. She tells the siblings in the experimental group to speak much more rapidly than they would at home. In this study, what is the dependent variable? A. Rates of stuttering in the children in both groups B. The rate of speech of the siblings in the experimental group C. The rate of speech of the siblings in the control group D. The combined amount of stuttering by the children in both the experimental and the control groups

A. Rates of stuttering in the children in both groups

An audiologist presents a passage of connected speech to your patient. An excerpt from the passage is as follows: "I went to the beach last week. My mom and I had a lot of fun. We built a sandcastle and collected a bunch of seashells. At the end of the day, we got vanilla ice cream on the boardwalk." Your patient repeats the message syllable for syllable. The material that was selected was within your patient's language level and the passage was interesting to the patient as well. This technique can be used as a tool in aural rehabilitation and it has great face validity because the material approximates normal communication more closely than single words. It can also help to improve listening and spoken language skills. What is this technique called? A. Speech tracking B. Listening training C. Pure-tone testing D. Acoustic highlighting

A. Speech tracking

An audiologist is conducting a hearing test on an 84-year-old-patient he suspects has a hearing loss. He would like to evaluate baseline hearing of the patient and provides her with a list of two-syllable, or disyllable, words that are pronounced with equal stress on the first and second syllables (e.g., suitcase, pushcart). Which test is the audiologist administering? A. Spondee recognition test B. Whispered recognition test C. Rinne test D. Weber test

A. Spondee recognition test The Rinne test (/ˈrɪnə/ RIN-ə) is used primarily to evaluate loss of hearing in one ear. It compares perception of sounds transmitted by air conduction to those transmitted by bone conduction through the mastoid. Thus, one can quickly screen for the presence of conductive hearing loss. The Weber test is a screening test for hearing performed with a tuning fork

There are several different types of cancerous cells that may result in laryngeal cancer. What is the most common type of laryngeal cancer that develops in flat, scale-like cells lining the pharynx? A. Squamous cell carcinoma B. Spindle cell carcinoma C. Adenocarcinoma D. Lymphoepithelioma

A. Squamous cell carcinoma

A teacher directs a child to describe the actions of a story that is presented on their communication device via a picture sequence. What is this type of teaching called? A. Structured teaching B. Situational teaching C. Generalization teaching D. Integrated teaching

A. Structured teaching Situational Approach, and to insure that the language that is being taught is realistic, all the words and sentences must grow out of some real situation or imagined real situation.The learners know the meaning of the word "blackboard", not because they have looked it up in a dictionary, but because they have learned the word in situations; by hearing commands such as: "Look at the blackboard!"; An integrated curriculum connects different areas of study by emphasizing related concepts across subject matters. T

Melodic intonation therapy (MIT) is an aphasia treatment program for clients with nonfluent aphasia who have good auditory comprehension. The treatment has three levels and uses musical intonation, continuous voicing, and rhythmic tapping. Which of the following is a characteristic of MIT Level III? A.Tap and intone and let the client intone the phrase after a 6 second delay, while providing tapping assistance. B.Show a picture and say the target item. C.Intone in unison with the client and tap. D.Fade prompts halfway through the phrase.

A. Tap and intone and let the client intone the phrase after a 6 second delay, while providing tapping assistance.

A 60-year-old patient fell and hit his head on a bathtub, sustaining a traumatic brain injury (TBI). You have physician orders to evaluate his responsiveness and communication. When you screen this patient, he is generally unresponsive to verbal or tactile stimuli. The following assessments may be an appropriate choice for a full evaluation of this patient, except: A. The Galveston Orientation and Amnesia Test B. The Rancho Los Amigos Levels of Cognitive Function C. The Glasgow Coma Scale D. The Disability Rating Scale

A. The Galveston Orientation and Amnesia Test

A 2-year-old with an unrepaired cleft palate was recently seen for an evaluation. When comparing the child's data to developmental norms, the speech-language pathologist determines that the child has not mastered bilabial stops. Why would developmental norms not determine whether the child receives speech treatment? A. The child does not have the anatomical capacity to produce bilabial stops. B. The speech-language pathologist would need to consult with medical professionals before determining treatment. C. Developmental data do not apply to every child. D. Every child develops at a different pace.

A. The child does not have the anatomical capacity to produce bilabial stops.

To diagnose stuttering, the clinician needs a definition of it. Although there are different definitions of stuttering, a majority of them include forms of disfluencies. Which of the following is a definition of stuttering that includes disfluencies? A. The one offered by Van Riper B. The one offered by Johnson C. The one offered by psychoanalysts D. The one offered by Sheehan

A. The one offered by Van Riper

A speech-language pathologist who worked in a neonatal intensive care unit treated many children with developmental issues. She asked her student intern what the difference was between reflexive cries and vegetative sounds. How should the student respond? A. Vegetative sounds are associated with feeding while reflexive vocalizations are automatic responses about the physical state of the infant. B. Vegetative sounds are about the physical state of the infant while reflexive sounds are associated with feeding. C. Vegetative sounds are cries, coughs, burps, and grunts while reflexive vocalizations are grunts, sighs, and clicks. D. Vegetative sounds are produced in comfortable states and reflexive sounds are produced in uncomfortable states.

A. Vegetative sounds are associated with feeding while reflexive vocalizations are automatic responses about the physical state of the infant.

Babies with cleft palate show no difference in which aspect of their language development in comparison with their peers: A. Vocalization frequency B. Canonical babbling C. Consonant inventories D. Cry behaviors

A. Vocalization frequency

Bloodstein suggested that stuttering is A. a response of tension and speech fragmentation. B. an avoidance behavior. C. due to parental concern. D. due to genetic factors.

A. a response of tension and speech fragmentation.

A clinician who works in a skilled nursing facility has been referred a patient with swallowing disorders. The referring physician states that the patient has a weak cricopharyngeus, causing difficulties in passing the bolus through the cricopharyngeus muscle and past the 7th cervical vertebra. Most likely this patient has A. disorders of the esophageal phase of swallow. B. difficulties in propelling the bolus through the pharynx and into the P-E segment. C. difficulty in forming and holding the bolus, accompanied by slippage of food into the lateral sulcus. D. food residue in the vallecula, on top of the airway, and in the pyriform sinuses.

A. disorders of the esophageal phase of swallow.

A 45-year-old man was referred to a gastrointestinal physician for an evaluation because of respiratory distress and difficulties swallowing. The physician conducted a barium esophagram study and noted an oblique filling defect in the posterior esophagus. On completing her evaluation, the attending cardiologist also mentioned to the patient that she noticed an aortic arch anomaly (an aberrant right subclavian artery—the right subclavian artery arose from the left side of the aortic arch and was externally compressing the esophagus). The cardiologist mentioned to the patient that this vascular anomaly in the thorax, particularly in the aortic arch, was compressing the trachea and esophagus and resulted in the patient experiencing significant respiratory distress and feeding difficulties. This condition is called

A. dysphagia lusoria.

Lee, a 42-year-old male, has come to an outpatient center to be evaluated by a speech-language pathologist due to concerns expressed by his wife and children regarding his speech. Upon arriving at the clinic, the clinician notices that Lee's oral and facial structures appear normal at rest. During the evaluation, Lee admits that for the past 5 years, he has noticed moments of random slurred speech during conversation; however, he never wanted to bring up the issue with his family because he was too embarrassed of the occasional drunken sounding speech. He told the clinician, "I just hoped that no one would ever notice." His wife reported that within the past year his conversational speech has been characterized by irregular articulatory breakdowns, reduced rate of speech, and unsteady vowel prolongations. She expressed concerns for her husband's ability to communicate with others who are unaware of his deficits. Lee complained that he is unable to coordinate his breathing with speaking, and often bites his cheek or tongue when speaking. This creates discomfort and frustration for him, but his wife reported that when Lee slows down his speech, he usually improves his intelligibility. According to his family, Lee often seems discouraged that his speech will never improve and he will suffer from the deficits for the rest of his life. During the evaluation, the clinician notices that his wife and three children provide a strong support system for Lee. QuestionTopic: Motor Speech The first thing the clinician should do when initially evaluating Lee's speech is to: A. ensure that Lee possesses some form of functional communication. B. use strengthening exercises to address weakness. C. teach Lee facial relaxation techniques to use at home. D. educate Lee on the Lee Silverman Voice Treatment strategy.

A. ensure that Lee possesses some form of functional communication.

Stuttering in preschool children tends to occur somewhat more frequently A. function words than on content words. B. content words than on function words. C. words that begin with /s/ and /k/ than on words that begin with other sounds. D. second or third syllables in multisyllable words.

A. function words than on content words.

Acoustical, mechanical, or electrical resistance to motion or sound transmission is called A. impedance. B. admittance. C. immitance. D. velocity.

A. impedance.

An outpatient was seen for therapy for acquired apraxia of speech. The clinician instructed the patient to produce speech at one syllable per beat. The beat was set at a slower rate than the patient's actual speaking rate. Hand-tapping was also used simultaneously with this treatment technique. This is an example of A. metronomic pacing. B. metrical pacing. C. rate control. D. integral practice.

A. metronomic pacing.

A patient was referred for therapy to an outpatient rehabilitation facility because of a diagnosis of apraxia of speech post CVA. The clinician decided to use a shaping technique that focused on orofacial and articulatory postures with specific instructions about how to change current speech and non-speech movements to achieve the target sounds. This is an example of A. phonetic derivation. B. phonemic awareness. C. rate control. D. integral practice.

A. phonetic derivation.

A clinician is evaluating a 12-year old boy, Justin, who was in a car accident and sustained a traumatic brain injury. He is now having difficulty communicating and has complex communication needs. The clinician is attempting to determine which type of alternative/augmentative communication (AAC) technology to use with him. In evaluating Justin's willingness to use this technology, she must consider response efficiency, which involves: A. quality, rate, and immediacy of reinforcement as well as response effort B. the speed at which Justin can press keys on the AAC device C. whether or not a device's symbols are transparent D. whether or not a device's symbols are opaque

A. quality, rate, and immediacy of reinforcement as well as response effort

Sometimes specialists assess the lung volume of voice patients because breath support is inadequate. Specialists can measure _______, or the total volume of air in the lungs; other measurements can include _______, or the amount of air inhaled and exhaled during a normal breathing cycle; and _______, or the volume of air that the patient can exhale after a maximal inhalation. A. total lung capacity, tidal volume, vital capacity B. vital capacity, tidal capacity, total lung volume C. vital capacity, total lung capacity, tidal volume D. tidal volume, total lung capacity, vital volume

A. total lung capacity, tidal volume, vital capacity

A 45-year-old woman was complaining of difficulty masticating. She had tonic spasms after irradiation. The physician explained to the speech-language pathologist that this condition is believed to exist secondary to fibrosis of the muscles involved in mastication. This condition is called A. trismus. B. osteoradionecrosis. C. necrosis. D. mucositis.

A. trismus. also called lockjaw, is reduced opening of the jaws Necrosis is the death of body tissue

Which of the following is not true about the fringe vocabulary set of an augmentative and alternative communication device? A. It includes words that occur frequently in typical speech. B. It typically includes nouns. C. It includes vocabulary specific to the individual. D. It may include names of people or places.

A. It includes words that occur frequently in typical speech. Fringe vocabulary refers to a specific set of lowfrequency vocabulary words that are specific to a particular person or activity. Fringe vocabulary words are typically nouns. Examples of fringe vocabulary words include bat, home run, base, and pitcher. Again, these words are highly specific to a single activity and are generally only useful when talking about baseball.

Joint attention is very important when establishing the interaction between the caregiver and infant. Caregivers and infants have constant joint attention. What are the two types of joint attention that occur between caregivers and infants? A. Joint reference B. Joint action C. Joint preference D. Joint perception

A. Joint reference B. Joint action

Which of the following could be a technique used in therapy when treating a pediatric case of reduced tongue movements? A. Oral stimulation program B. Lip stroking C. Nasal tapping D. Jaw closure

A. Oral stimulation program

Disorders of the oral phase of a swallow are mainly due to A. an anterior, as opposed to a posterior, tongue movement. B. slippage of food into anterior and lateral sulcus. C. a reduced range of lateral mandibular movement. D. a difficulty in holding the bolus.

A. an anterior, as opposed to a posterior, tongue movement.

You are evaluating Tatyana, a Russian-speaking child with a suspected language impairment. To accurately estimate her language skills, you engage a Russian-speaking interpreter and count Tatyana's responses to test questions in both Russian and English. You are employing: A. conceptual scoring B. dynamic assessment C. criterion-referenced testing D. assessment of working memory

A. conceptual scoring

There are many different diseases that affect muscles. Congenital myopathies and muscular dystrophy are muscle diseases that cause __________: A. hypotonia and weakness. B. breathiness and hypernasality. C. Parkinsonism and tardive dyskinesia. D. flaccidity and spasticity.

A. hypotonia and weakness.

7. In the scientific method, what is the experiment-first-and-explain-later approach? A.Deductive method B.Behavioral method C.Null hypothesis method D.Inductive method E.Alternative hypothesis method

Inductive

You have been asked to counsel with John, a 70-year-old man who has smoked and drank alcohol since he was a teenager. He now has laryngeal cancer, and, before surgery, the surgeon asks you to talk with John about esophageal speech. You explain to John that there are two basic types of esophageal speech. In one method, the patient is taught to keep the esophagus open and relaxed while inhaling rapidly. In the other method, the patient impounds the air in the oral cavity, pushes it back into the esophagus, and vibrates the cricopharyngeus muscle. What is the second method called?

Injection method

Christopher is an 18-year-old client with a repaired cleft palate. He has production accuracy on most sound classes; however, he still struggles with certain sounds during therapy. Based on his history of a repaired cleft palate, which two of the following sound classes would Christopher most likely have difficulty with? A. Stops B. Fricatives C. Affricates D. Glides

B. Fricatives C. Affricates

You receive a referral to assess Beth, who is a 76-year-old resident in a nursing home. The certified nursing assistants who work with her on a daily basis tell you that Beth has been experiencing changes in behavior. During group activities she makes inappropriate comments and appears impulsive and distractible. The certified nursing assistants also note that she has decreased energy and motivation. You notice she is having mild problems with language as well. Which of the following forms of dementia is Beth displaying? A.Vascular dementia B.Frontotemporal dementia C.Lewy body dementia D.Alzheimer's disease

B. Frontotemporal dementia

____________ frequencies are resonated with greater amplitude than ____________ frequencies. A. Distorted; lower B. Higher; lower C. Lower; higher D. Attenuated; impedance-free

B. Higher; lower

Rhythmic cueing is an appropriate rate approach to use when treating a client with all of the following types of dysarthria, except: A. Hypokinetic B. Hyperkinetic C. Ataxic D. Unilateral upper motor neuron

B. Hyperkinetic

Human communication is vital for language development. Which of the following is not a main cognitive principle? A. Equilibrium B. Imitation C. Organization D. Adaptation

B. Imitation Piaget 3 principles of equalibrium-bringing ideas into balance with reality organization-arranging mental structures in orders adapation-changing behavior to meet environment

An audiologist is testing a client for acoustic immittance. She places a sound stimulus in her client's external ear canal with an airtight closure and measures changes in the acoustic energy as the sound stimulates the auditory system. What instrument is the audiologist using for this particular procedure? A. Otoscope B. Impedance bridge C. Audiometer D. Tympanic meter

B. Impedance bridge

A lower prevalence of stuttering like dysfluencies has been noted for the following population: A. Children with autism spectrum disorder B. Individuals who are deaf or hard of hearing C. Children with intellectual disability D. Individuals diagnosed with a traumatic brain injury

B. Individuals who are deaf or hard of hearing

You perform an oral-mechanism examination on a client. You ask him to stick his tongue out. When he is sticking out his tongue, you notice that it deviates toward the left. Since tongue musculature is paired, this means your client probably has a weak: A. Right genioglossus B. Left genioglossus C. Right hyoglossus D. Left hyoglossus

B. Left genioglossus

Which of the following is not a treatment focus of speech-language pathologists who work with patients with Parkinson's disease? A. Physical exercise and increased attention to breathing B. Making modifications to feeding utensils because of tremors C. Increase swallowing abilities by altering eating habits D. Making modifications to food consistency

B. Making modifications to feeding utensils because of tremors

Ellen, an 80-year-old patient, was being seen for swallowing problems. She presents with minimal arousal and is unable to follow simple one-step commands to complete an oral mechanism examination. The clinician initiates ice chip trials as a part of the evaluation and notes oral acceptance and manipulation that is within normal limits; however, no pharyngeal swallow is noted as per palpation. The clinician, therefore, decides to trial exercises that stimulate the swallow reflex and determine Ellen's stimulability for trials. The following exercises are designed to stimulate the swallow reflex, except: A. Thermal stimulation B. Mendelsohn maneuver C. Practicing liquid swallow after stimulation D. Asking the patient to swallow after stimulation without food

B. Mendelsohn maneuver

A clinician is conducting an in-service in a hospital and is recommending treatment designed for patients with severe aphasia and apraxia whose repetition abilities are severely impaired. The clinician mentions that the purpose of this approach is to shape a variety of utterances that may eventually be used volitionally. She is referring to the following approach to therapy: A.Script training B.Multiple input phoneme therapy (MIPT) C.Prompts for reconstructing oral muscular phonetic targets (PROMPT) D.Melodic intonation therapy (MIT)

B. Multiple input phoneme therapy (MIPT)

Which of the following is a group of genetic skeletal muscle diseases associated with muscle fiber degeneration and replacement with fatty and fibrous connective tissue? A. Multiple sclerosis (MS) B. Muscular dystrophy (MD) C. Amyotrophic lateral sclerosis (ALS) D. Poliomyelitis (Polio)

B. Muscular dystrophy (MD) Multiple sclerosis (MS) is a chronic illness involving your central nervous system (CNS). The immune system attacks myelin, which is the protective layer around nerve fibers. Amyotrophic lateral sclerosis (ALS), also known as motor neurone disease (MND) or Lou Gehrig's disease, is a disease that causes the death of neurons controlling voluntary muscles

Which of the following is not a common compensatory gesture seen with clients with cleft palate? A. Pharyngeal fricative B. Pharyngeal glide C. Nasal fricative D. Glottal stop

B. Pharyngeal glide

During an evaluation, a clinician must determine whether the client is exhibiting stuttering-like dysfluencies or whether the client is normally disfluent. Which of the following is a characteristic of normal developmental dysfluencies in a young child? A. Mid-word insertions B. Phrase repetitions C. Part-word repetitions D. Word-final prolongations

B. Phrase repetitions

To determine Abby's hearing loss, the pediatrician recommended a A.relative measure test. B.Rinne test. C.Webster test. D.Wilcox test.

B. Rinne test.

A'dab, a 4-year-old, was referred to you by her teacher, Mrs. Kocher. The teacher comments that A'dab has limited vocabulary, has difficulty during transition of activities, and struggles to interact with her peers. Mrs. Kocher also notes that A'dab's art skills surpass all students in the class. She mentions that A'dab displays certain abilities that exceed the average, especially her ability to memorize. You performed a preliminary screening and noticed several communication deficits; however, you noticed that A'dab could calculate very fast, had good musical ability, and was very adept at map making. During the parent interview, A'dab's mother, Aabidah, mentions that she is worried about A'dab's ability to make friends because at home, "she does not ever talk about other kids in her class, and she tends to isolate herself with her toys during play dates." Aabidah indicated that A'dab is like a human calendar. Question Topic: Special Topics The neurodevelopmental specialist diagnoses A'dab with? A. Down syndrome B. Savant syndrome C. Attention-deficit/hyperactivity disorder D. Obsessive compulsive disorder

B. Savant syndrome

When analyzing spectrograms on a spectrograph, there are different types of spectrograms that allow you to analyze your sample to the best of its ability. These different types of spectrograms are called wideband spectrograms and narrowband spectrograms. Which of the following characteristics is expressed better in a wideband spectrogram? A. Long time window B. Short time window C. Good for showing harmonics D. Good for measuring harmonics

B. Short time window

Which of the following occurs when the behaviors of two individuals mutually influence the subsequent behaviors of the other? A. Social interaction B. Social integration C. Social influence D. Social communication

B. Social interaction

Brady is a 3-year-old whose mother is concerned about his language development. She reaches out to a speech-language pathologist to obtain information about normal language development. Based on normal language development in children, which of the following features is typical of a child who is 3 years old? A.Not easily understood by strangers B.Speaks in longer, complex sentences C.Put two words together to form a short sentence D.Uses jargon when speaking to adults

B. Speaks in longer, complex sentences

When assessing the communicative disorders of your 10-year-old client Sofia, you use charting—which provides a method of scoring her responses and identifying herabilities/deficits. Choose the incorrect statement about charting: A. If Sofia stays in her chair for 30 seconds, this is an example of a behavior that is worth charting. B. Speech pathologists use one way to chart behaviors and that is by noting each time a preselected behavior is exhibited. C. Even if certain behaviors are not necessarily caused by the communicative disorder, charting can still be appropriate for behaviors that are important in the treatment process. D. Different forms are available that are designed for specific age groups.

B. Speech pathologists use one way to chart behaviors and that is by noting each time a preselected behavior is exhibited.

Barry, a 42-year-old male, slips on ice and hits his head on a concrete sidewalk. He does not lose consciousness but has some swelling and a small laceration at the back of his head. He appears to be fine and has been moving around. Later, he complains of a severe headache and takes some aspirin. Forty-five minutes after taking the aspirin, he still has a headache and is feeling nauseous. Shortly after this he vomits and is complaining of a stiff neck. What neurologic event might explain Barry's symptoms? A. Epidural hemorrhage B. Subdural hemorrhage C. Subarachnoid hemorrhage D. Intracerebral hemorrhage

B. Subdural hemorrhage

Intrinsic muscles of the larynx serve important functions in phonation. Some are primarily vibrating muscles, whereas others, by their actions, affect the actions of the vocal folds and the resulting quality of phonation. Of the statements that follow, select the one that is incorrect. A. The thyroarytenoids are divided into two muscle masses. B. The transverse arytenoids are laryngeal abductors. C. The internal thyroarytenoids are also known as the vocalis muscle. D. The cricothyroid muscle lengthens and tenses the vocal folds.

B. The transverse arytenoids are laryngeal abductors.

You evaluate a client who has been in a car accident. You notice that she has hypernasal speech and her jaw hangs open. You can infer that there has been damage to which cranial nerve? A. Facial Nerve B. Trigeminal Nerve C. Vagus Nerve D. Glossopharyngeal Nerve

B. Trigeminal Nerve

Siobhan, a 74-year-old female, is a new resident at the nursing home. After reading her case history, you learn that she recently has had a history of multiple strokes. In her case history, it states that her cognitive impairments coincided with her strokes. She is disoriented, confused, has trouble speaking or understanding speech, and has vision loss. Other symptoms include uncontrolled laughing and crying, problems with judgment and planning, and a reduced ability to pay attention. Her daughter reports that Siobhan appears to display symptoms of depression, noting that "ever since the strokes, Mom has a difficult time with simply getting up in the morning and she just doesn't seem to enjoy being around her family like before." In person, Siobhan shows loss of motivation and attention during various tasks. She appears to have some memory loss, but primarily struggles with executive functioning. Siobhan also presents with small-step gait and unsteadiness. She is prone to falls and is required to walk with a cane. Her regular nurses report that she has urinary urgency not explained by urologic disease along with emotional incontinence. Due to concerns expressed by her family, Siobhan is being evaluated to determine the presence of dementia. QuestionTopic: Dementia Based on the information provided, Siobhan's most likely presents with: A. Traumatic dementia B. Vascular dementia C. Toxic dementia D. Lewy body dementia

B. Vascular dementia

You are working with Cornelius, a 7-year-old client who has autism spectrum disorder. The target behavior for the session is taking turns with little to no frustrations. Cornelius responds well to most types of motivators. During your session, he shows no signs of frustration and takes turns cooperatively. What is the most effective way to ensure increased frequency of the target behaviors? A. Verbal praise after each turn and an additional reward at the end of the session B. Verbal praise after each turn, short breaks as a reward, and an additional motivator at the end of the session C. Verbal praise after each turn and short breaks as a reward D. Short breaks as a reward and an additional motivator at the end of the session

B. Verbal praise after each turn, short breaks as a reward, and an additional motivator at the end of the session

In the source-filter theory, the source and the filter are assumed to be independent. It is implied that you can change the output of the source without changing the filter and vice versa. Which anatomical structure represents the source? A. Lungs B. Vocal fold vibration C. Vocal tract D. Vocal folds

B. Vocal fold vibration

One year post cleft lip and palate surgery you are revaluating a 3-year-old patient. It appears that after her surgery, she does not have any medical difficulties; however, she has mild hypernasality. Her family notes that they would be more comfortable addressing her mild hypernasality during speech therapy rather than getting surgery. Which of the following best describes the information that you should share with your patient's family? A. Surgical procedures to correct structural abnormalities is the only way to correct her mild hypernasality. B. Voice therapy could be provided to address her mild hypernasality. C. Articulation therapy could be provided to address her mild hypernasality. D. There is no treatment to reduce hypernasality.

B. Voice therapy could be provided to address her mild hypernasality.

You are working in an elementary school setting. A classroom teacher comes to you to refer an 8-year-old boy whose voice sounds consistently "hoarse" and "breathy" and gets more severe during recess and lunch. The child does not display symptoms of stridor, aspiration, or pain. As the speech-language pathologist, what should you initially suggest based on the child's symptoms? A. The child should be referred to an otolaryngologist. B. You should perform a complete head and neck evaluation. C. The child should go for a radiologic evaluation. D. The child should try singing to see if there is a difference in the voice.

B. You should perform a complete head and neck evaluation.

A patient was having difficulty swallowing solids and liquids and was regurgitating his food hours after eating. During a barium swallow the physician noted that there was a complete loss of peristalsis. She also noted that the nonrelaxing lower esophageal sphincter (LES) was preventing the downward passage of the bolus into the stomach. This condition is called A. trismus. B. achalasia. C. luminal deformity. D. extrinsic compression.

B. achalasia. trismus-lockjaw

Some researchers have claimed that stuttering may be an operant behavior, which is behavior that is A. controlled by internal emotions. B. changed by its consequences. C. unaffected by environmental stimuli. D. essentially reflexive.

B. changed by its consequences.

You are taking a language sample from an 8-year-old child. One of his utterances is "I will go to school tomorrow if I am not sick." This is an example of a A. compound sentence with an independent and a dependent clause. B. complex sentence with an independent and a dependent clause. C. complex sentence with two independent clauses. D. complex sentence with two dependent clauses.

B. complex sentence with an independent and a dependent clause.

John has cerebral palsy that has resulted in a motor speech disorder caused by central nervous system damage. This damage has caused him to have weakness and incoordination of the muscles of speech. His speech is classified as A. paraphasic. B. dysarthric. C. apraxic. D. aphasic.

B. dysarthric.

A researcher places surface electrodes on both sides of a patient's thyroid cartilage and passes a high-frequency electric current between the electrodes. The patient is asked to phonate while the electric current is applied to the electrodes. This procedure is called A. videostroboscopy. B. electroglottography. C. electromyography. D. laryngoscopy. Your Answer : C Correct Answer : B

B. electroglottography. Electromyography is an electrodiagnostic medicine technique for evaluating and recording the electrical activity produced by skeletal muscles.

Part C of the Individuals with Disabilities Education Act

is a federal grant program that assists states in operating a comprehensive statewide program of early intervention services for infants and toddlers with disabilities, ages birth through 2 years.

Differential reinforcement of incompatible behavior (DRI)

is a procedure in which the teacher would identify a behavior that's incompatible with, or cannot occur at the same time as, the problem behavior. The focus is on replacing negative behaviors with positive behaviors. Kevin engages in out of seat behavior so his teacher decides to implement a DRI procedure. She decides to reinforce a behavior that is incompatible with out of seat behavior.

Grayson, a 55-year-old restaurant owner from Canada, suffered a lesion because of a stroke he suffered at work 2 months ago. Ever since the stroke, he has severe difficulties with naming common objects, along with a slow rate of speech and limited word output during conversation. During his initial evaluation, it was noted that Grayson mainly produces content words, which allows some meaning to be translated during conversation; however, his speech sounds very agrammatical. He also showed difficulties with repetition when asked to repeat back a sentence that the clinician produced. Because of Grayson's acquired deficits, he has been unable to work at his restaurant; however, he wished to return to his beloved restaurant as possible. Grayson receives speech therapy at his home three times a week to address the speech and language deficits caused by his stroke. During therapy, his clinician works on increasing his length of utterances and the complexity of his responses. The clinician also works on decreasing grammatical errors and naming difficulties. Grayson's clinician reports that he appears to get frustrated during conversation, indicating that he is aware of his deficits. QuestionTopic: General Treatment Based on the location of Grayson's lesion, it is possible that he experiences motor deficits: A. on the left side of his body. B. on the right side of his body. C. bilaterally. D. everywhere.

B. on the right side of his body.

While conducting an evaluation of an infant in a neonatal ICU, a student intern informed her supervisor that she noticed that while stimulating the gums of the infant, it elicited a rhythmical opening and closing of the jaw. She also noticed that the infant gently nibbled on her finger. The supervisor informed the student that normally developing infants go through this stage, called A. tonic bite reflex. B. phasic bite reflex. C. jaw retraction. D. jaw clenching.

B. phasic bite reflex.

The goal of structured contexts in articulatory therapy is to preserve the accuracy of target sound production within: A.contexts of words. B.phrases and sentences. C.structured conversations. D.multiple vowel contexts.

B. phrases and sentences.

You are treating an 88-year-old woman with dementia. You introduce a treatment program to improve her quality of life. You begin your therapy by discussing past experiences and events in her life. You also use photographs of the patient's family members as well as music that she likes. You tailor the therapy specifically for the client by using objects and items that her son has provided to you. This treatment method is an example of A. reality orientation (RO). B. reminiscence therapy (RT). C. simulated presence therapy (SPT). D. cognitive stimulation therapy (CST).

B. reminiscence therapy (RT).

Rosalia is a third-grade Mexican American Spanish-speaking 8-year-old girl who is in the process of learning English. Her parents emigrated from Mexico 2 years ago; thus, Rosalia was exposed first to Spanish at home and to English in first grade, when she was 6 years old. The classroom teacher shares with you that she thinks Rosalia may have an articulation disorder, but the teacher is not sure. The teacher provides you with some examples of things that Rosalia has said in the past 2 or 3 weeks. As you look at these examples, which one of the following would not be a typical predictable production based on Spanish influence? A. Omission of /h/ in word-initial position (e.g., -elp/help) B. w/r substitutions (e.g., wing/ring) C. t/th substitutions in word-initial positions (e.g., tin/thin) D. Devoicing of final consonants (e.g., beece/bees)>

B. w/r substitutions (e.g., wing/ring)

Differential Reinforcement of Other Behavior (DRO)

is a procedure where positive reinforcement is provided only when the target behavior is not displayed for a specified period of time.

In the mand-model method of teaching language skills to children, the clinician does which of the following? A. Prompts an elaboration of a verbal response, and requires the child to imitate B. Asks questions like "Tell me what you want" and models or prompts the correct response if necessary C. Adds comments to the child's production, with no response requirement D. Repeatedly and consecutively models a particular structure but does not require the child to imitate it

B. Asks questions like "Tell me what you want" and models or prompts the correct response if necessary

All of the following could be techniques used in therapy when treating a pediatric case of tongue retracting except? A. Alter feeding position, try head slightly flexed with close chin-down position. B. Begin at the middle of the upper lip and tap to the corner of the mouth; repeat on the other side. C. Alter feeding utensil; try a longer nipple. D. Apply rhythmic tapping under base of chin.

B. Begin at the middle of the upper lip and tap to the corner of the mouth; repeat on the other side.

You have completed the case history and assessment of a 72-year-old man and have found, among other things, inappropriate social behavior, excessive eating, depression, impaired judgment, and dominant language problems. The neurologist's report suggests dense intracellular formation in the neuronal cytoplasm. The most likely diagnosis would be which of the following? A. Dementia of the Alzheimer's type. B. Frontotemporal dementia associated with Pick's disease C. Dementia associated with Parkinson's disease D. Dementia due to Creutzfeldt-Jakob disease

B. Frontotemporal dementia associated with Pick's disease

Select the statement that is incorrect about group designs. A. They are effective in establishing internal validity. B. They usually have results similar to those of single-subject experiments. C. They can help establish cause-effect relationships. D. Their requirement of randomization may be difficult to meet.

B. They usually have results similar to those of single-subject experiments.

A 51-year-old man was seen in a clinic because he was having sudden, severe, burning pain when washing his face and shaving. He also complained of pain when talking, drinking, brushing his teeth, or even when he was exposed to wind. He mentioned to the clinician that he rarely had pain at night or when he was sleeping. The physician diagnosed him with: A. Bell's palsy B. Trigeminal neuralgia C. Ptosis D. Ramsay Hunt syndrome

B. Trigeminal neuralgia ptosis-drooping eye ramsay hunt syndrome-shingles outbreak affects the facial nerve near one of your ears.

A study designed to evaluate a treatment procedure for hoarseness of voice recruited all subjects who had witnessed a football game the previous day. The treatment, conducted over the following 2 weeks and offered to all subjects, resulted in improved voice quality. A critic said that the study lacked internal validity because of A. median statistics. B. statistical regression to the mean. C. maturational variables. D. genetic influences.

B. statistical regression to the mean.

A correlation coefficient A. helps establish the effect of a variable. B. suggests the ways in which two variables are related to each other. C. allows researchers to make predictions about their subjects' future behaviors. D. confirms a cause-effect relationship between two variables.

B. suggests the ways in which two variables are related to each other.

29. Which of the following primarily vibrate and produce sound? A. External thyroarytenoids B. Internal thyroarytenoids C. Transverse arytenoids D. Cricothyroid E. Lateral cricoarytenoids

B. Internal thyroarytenoids

79. Select the statement that is true of conduction aphasia. A. It is caused by widespread lesions in the entire perisylvian region. B. It is characterized by good syntax, prosody, and articulation. C. Its lesion sites are more definitive than they are for other types of aphasia. D. Speech associated with it is as fluent as it is in Wernicke's aphasia. E. It is characterized by severe auditory comprehension problems.

B. It is characterized by good syntax, prosody, and articulation.

37. A clinician is gathering a language sample from a child who says, "The book was read by the boy." This is an example of what type of sentence? A. Active B. Passive C. Exclamatory D. Declarative E. Imperative

B. Passive

11. What is the procedure that uses a pulsing light to permit the optical illusion of slow-motion viewing of the vocal folds? A. Electroglottography B. Stroboscopy C. Electromyography D. Videofluoroscopy E. Nasoendoscopy

B. Stroboscopy

6. A 70-year-old man who recently had a single left hemisphere stroke has been referred to you. The referring neurologist suggested possible damage to Broca's area and the surrounding tissue. Your initial conversation reveals an impaired oral sensation and a general awareness of his problems. The patient spoke at a deliberately slow rate; he made many speech errors that were highly variable; his automatic speech was relatively unaffected; and he often substituted voiceless speech sounds for voiced ones. What would be your suspected diagnosis and assessment strategy for this patient? A. Suspecting Broca's aphasia, I would assess language production, including fluency, word output, grammaticality, naming, comprehension of syntactic structures, and so forth. B. Suspecting apraxia of speech, I would assess in detail speech production problems, including imitated, evoked, and repetitive productions of phonemes, syllables, shorter and longer words, phrases, and sentences, as well as automatic and spontaneous productions, and so forth. C. Suspecting transcortical motor aphasia, I would assess repetition skills, sentence completion, word fluency, grammaticality, echolalia, preservation, and so forth. D. Suspecting mixed transcortical aphasia, I would assess spontaneous speech, automatic and unintentional nature of communication, impaired fluency, and so forth. E. Suspecting conduction aphasia, I would assess automatic speech, repetition skills, fluency and phrase length, articulation skills, and so forth.

B. Suspecting apraxia of speech, I would assess in detail speech production problems, including imitated, evoked, and repetitive productions of phonemes, syllables, shorter and longer words, phrases, and sentences, as well as automatic and spontaneous productions, and so forth.

31. Which of the following is the cranial nerve that innervates the larynx and the levator veli palatini, palatoglossus, and palatopharyngeus muscles? A. Cranial Nerve X, the vagus nerve B. Cranial Nerve V, the trigeminal nerve C. Cranial Nerve XI, the spinal accessory nerve D. Cranial Nerve VII, the facial nerve E. Cranial Nerve XII, the hypoglossal nerve

A. Cranial Nerve X, the vagus nerve

99. An investigator carries out a study to answer the question whether an increased rate of sibling speech causes an increase in the frequency of stuttering in children. After pretesting rates of stuttering in selected children and the speech rate of their siblings, the investigator tells the siblings in the control group to speak as they normally would at home. She tells the siblings in the experimental group to speak much more rapidly than they would at home. In this study, what is the dependent variable? A. Rates of stuttering in the children in both the groups B. The rate of speech of the siblings in the experimental group C. The rate of speech of the siblings in the control group D. The combined amount of stuttering by the children in both the experimental and the control groups E. The rate of stuttering of the children in the experimental group

A. Rates of stuttering in the children in both the groups

83. Which articulation difference is not commonly observed among Asian speakers of English as a second language? A. Shortening of polysyllabic words B. t/k substitution (e.g., tin/kin) C. Omission of /r/ D. Confusion of /r/ and /l/ E. Substitution of a/ae (e.g., shock/shack)

B. t/k substitution (e.g., tin/kin)

27. Which of the following is an example of an entity + locative? A. That chair B. More juice C. Jump [on] bed D. Juice [in] glass E. Doggy bark

A. That chair

26. Broad phonemic transcription involves which of the following? A. The use of IPA symbols to transcribe phonemes by enclosing them within slash marks (e.g., /f/) B. The use of diacritical markers to transcribe phonemes by enclosing them within slash marks (e.g., /f/) C. The transcription of allophones by placing them within brackets (e.g., [f]) D. The transcription of allophones by the use of diacritical markers E. The use of orthographic symbols to transcribe phonemes by enclosing them within slash marks (e.g., /r/)

A. The use of IPA symbols to transcribe phonemes by enclosing them within slash marks (e.g., /f/)

The use of single words to express intention is typically seen in children

ages of 18 to 24 months

Which of the following is a typical social communication behavior for a child aged 3-5 years? A begins code switching and uses simpler langauge when talking to younger children B uses narratives characterized by causally sequenced events C uses language with the intent to persuade others and change their opinions D uses single words to express intention

Begins code switching and uses simpler langauge when talking to young children

Melodic intonation therapy MIT is generally considered most appropriate for a client with which of the following? A broca's B conduction C transcortical sensory aphasia D global aphasia

Broca's aphasia

left inferior frontal gyrus

Broca's area

A first-grade teacher refers Rashina to you. Rashina speaks AAE. The teacher says, "I am concerned about Rashina's grammar and her pronunciation of some of her sounds." Which one of the following would not be a typical utterance for a child such as Rashina? A. "Today I be walkin' to the speech room." B. "I gonna get my pencil off my des'." (I'm going to get my pencil off my desk.) C. "I don' yike dat one." (I don't like that one.) D. "That man, he be a foo' for not comin'." (That man is a fool for not coming.)

C. "I don' yike dat one." (I don't like that one.)

You are screening the speech of a third-grade Spanish-speaking student named Araceli. Which of the following would not be typical for Araceli based on the influence of her primary language of Spanish? A. "My sister Yulie [Julie] is coming." B. "I like berry much my teacher." C. "My mommy and me be goin' shoppin' later." D. "My friends always say 'ello [hello] to me."

C. "My mommy and me be goin' shoppin' later."

You just completed an assessment of an 8-year-old boy who stutters. When you are offering post-assessment counseling to the boy's parents, they ask you, "What do you think caused stuttering in our son?" How would you answer their question? Select the best among the alternatives given. A. "Although we have many theories, we don't know the cause of stuttering, because no theory has been substantiated." B. "Stuttering is a genetically inherited disorder in almost all cases because more males than females stutter; there is a certain concordance rate for identical twins." C. "We can't say for sure in individual cases, but both complex genetic susceptibility and environmental factors may be involved in its causation." D. "Research suggests that stuttering is caused by parental pressure on the child to be more fluent; it is usually associated with high demands for fluency."

C. "We can't say for sure in individual cases, but both complex genetic susceptibility and environmental factors may be involved in its causation."

Given a sound pressure of 20,000,000 µPa, what is the dB SPL? A. 200 dB SPL B. 2 dB SPL C. 120 dB SPL D. 230 dB SPL

C. 120 dB SPL

A complex, periodic signal has a period of 1 ms. What is the frequency of the 8th harmonic? A. 28000 Hz B. 230000 Hz C. 128000 Hz D. 98000 Hz

C. 128000 Hz

In assessing children with language disorders, clinicians often count the number of morphemes in a child's utterance. This counting helps assess the length of single utterances the child typically produces. Clinicians follow certain rules in counting morphemes. Of the following rules, which one is correct? A. Inflected words, such as looked, are counted as one morpheme. B. When words are repeated for emphasis (e.g., "No, no, no!"), each repetition is counted as a morpheme (i.e., three morphemes in the example). C. A compound word, such as birthday, is counted as one morpheme. D. Contracted negatives (e.g.,won't, can't) are counted as two morphemes.

C. A compound word, such as birthday, is counted as one morpheme.

Constrictions in the vocal tract will raise or lower the formant frequency. How is the first formant frequency, which is also known as F1, raised? A. Any constriction in the oral cavity B. A constriction in the oral cavity near a volume velocity maximum C. A constriction in the pharynx D. A constriction in the oral cavity near a pressure minimum

C. A constriction in the pharynx

A researcher is interested in completing a single-subject design experiment to study the additive and subtractive effects of individual components of treatment rather than the comparison of two treatments. He decides to use an interaction design to study the interactive effects of two or more variables. The researcher is also interested in examining the effects of both variables alone and in combination. He is interested in isolating the components that are effective to any extent from those that are not at all effective and controls for phase lengths during the treatment. He also counterbalances the order of treatment in the experiment. The most appropriate design for this experiment would be A. ABAB. B. BAB. C. A-B-A-BC. D. A-B-BC-B-BC.

C. A-B-A-BC.

Certified speech-language pathologists are involved in hiring speech-language pathology assistants. What are the minimum proper training requirements for speech-language pathology support personnel as recommended by ASHA? A. Completion of a speech-language pathology training program with integrated fieldwork and on-the-job training B. A bachelor's degree from a 4-year institution with the course of study directed in speech-language pathology along with fieldwork and on-the-job training C. An associate's degree from a technical training program with course of study directed in speech-language pathology along with fieldwork and on-the-job training D. A bachelor's degree from a 4-year institution with at least 20 semester credit hours at the graduate level along with fieldwork and on-the-job training

C. An associate's degree from a technical training program with course of study directed in speech-language pathology along with fieldwork and on-the-job training

Which area of the brain connects Broca's area with Wernicke's area? A. Corpus callosum B. Choroid plexus C. Arcuate fasciculus D. Corona radiata

C. Arcuate fasciculus Corpus Callosum is the part of the mind that allows communication between the two hemispheres of the brain. It is responsible for transmitting neural messages The choroid plexus is a plexus of cells that produces the cerebrospinal fluid in the ventricles of the brain

A patient displays persistent aphonia in the absence of any organic cause. The clinician decides to implement behavioral therapy. The following techniques are characteristic of behavioral therapy to treat aphonia, except: A. Masking B. Progressive relaxation leading to reduction in muscle tension C. Chant-talk method D. Yawn-sigh approach

C. Chant-talk method The chant-talk approach uses pre-existing characteristics found in chanting-styled music, such as rhythm and prosodic patterns. The therapy is used to reduce phonatory effort, which causes vocal fatigue. Chant therapy is used to minimize hyperfunctionality by affecting loudness and voice quality

All of the following are true with regard to the behavioral theory approach to counseling, except: A. There is a focus on what is observable with an emphasis on environmental, external influences. B. Behavior is thought to be shaped and maintained by immediate consequences. C. Clinicians emphasize the discussion of thoughts and feelings. D. It is believed that positive reinforcement as well as negative reinforcement could cause a behavior to reoccur.

C. Clinicians emphasize the discussion of thoughts and feelings.

Though speech-language pathologists are less likely to be subject to liability claims, clinicians can still be held accountable for clinical actions and client-care judgments. What is the best way to avoid liability claims when existing medical conditions are involved? A. Clinicians should consult with a trusted colleague or supervisor regarding a patient's existing medical condition to guarantee that the correct decision is being made. B. Clinicians should complete the required course work regarding medical speech-language pathology. C. Clinicians should keep up-to-date records of the most recent and relevant medical information so that the information can be used to make better decisions. D. Clinicians should discuss possible surgical options with a patient to improve function and then recommend surgery.

C. Clinicians should keep up-to-date records of the most recent and relevant medical information so that the information can be used to make better decisions.

Rhianna, a 22-year-old female, is 6-days post-traumatic brain injury (TBI) that she suffered from a motor vehicle accident. She has no history of substance abuse and is considered to be in good health prior to TBI. She emerged from a coma 36 hours ago and is now at Rancho Los Amigos Level—IV. She is confused and agitated. A neurologist gives you an hour to evaluate Rhianna and to provide recommendations. What information would contribute to a fair-good prognosis? A. Younger patients have higher mortality rates. B. History of substance abuse C. Comatose to Level IV on RLAS in a 36-hour period D. Six days comatose

C. Comatose to Level IV on RLAS in a 36-hour period

A clinical supervisor is discussing intelligibility with her graduate student clinician. She asks the student, "Which of the following is not a factor that influences intelligibility of an utterance?" A. Frequent abnormality in the clients' speech B. Context in which the communication occurs C. Consistency between the target and its realization D. Loss of phonemic contrasts

C. Consistency between the target and its realization

A 92-year-old patient with dementia is experiencing severe oral-and-pharyngeal-stage dysphagia including aspiration. The clinician would like to teach the patient a swallowing therapy technique, as the family is concerned that he is losing weight. The patient is currently being tube fed and is monitored closely by the hospital staff and dietitian. Ethically, which is the only appropriate technique that the clinician should consider? A. Introduce the supraglottic swallow procedure to the patient. B. Teach the family members the chin tuck procedure, so that they can feed the patient during visiting hours. C. Continue with the tube feeding. D. Explain to the family that it is okay for the patient to ingest small amounts of food every hour.

C. Continue with the tube feeding.

A 73-year-old client is complaining of issues with regulation of motor movements of the pharynx and lack of sensation of the tongue. What cranial nerve regulates motor movements of the pharynx and sensations of the tongue? A. Cranial Nerve V B. Cranial Nerve VII C. Cranial Nerve IX D. Cranial Nerve XII

C. Cranial Nerve IX

This disease is a rare, degenerative brain disorder that results in death. Each year, worldwide, it affects 1 person in every 1 million. This disease usually manifests itself later in life. About 70% of patients die within a year. Initially, patients may have visual problems, failing memory, lack of coordination, and behavioral changes. As the disease progresses, patients display myoclonus and may become blind. They finally lose the ability to speak and enter into a coma. The scientific community believes that abnormal forms of normal cellular proteins (prions) cause this disease. The normal prion is called PrPC (C stands for cellular) and the abnormal form (which causes the disease) is called PrPSc (Sc stands for prototypical prion disease-scrapie). When the disease is acquired, the PrPSc comes from outside the body (e.g., contaminated meat). This disease is called: A. Lewy body dementia B. Wernicke-Korsakoff dementia C. Creutzfeldt-Jacob disease D. Pick's disease

C. Creutzfeldt-Jacob disease

In the case of vocal fold paralysis, the following treatment techniques can be trialed in voice therapy to achieve firmer vocal fold closure: A. Hard glottal attacks B. Elevation in pitch C. Decreased loudness D. Head turning or positioning

C. Decreased loudness

Which of the following is not a purpose of screening in the school setting? A. Identification of a problem B. Determine the need for further evaluation C. Diagnose speech and language disorders D. Detect the need to refer to other professionals

C. Diagnose speech and language disorders

Parkinson's disease is called a movement disorder because of the tremors and the slowing and stiffening that occur. The cells used to make dopamine are impaired in those with Parkinson's. Which of the following neurotransmitters is primarily responsible for controlling movement, emotional responses, and the ability to feel pleasure and pain? A. Acetylcholine B. Norepinephrine C. Dopamine D. Endorphin

C. Dopamine Acetylcholine (ACh) is an organic chemical that functions in the brain and body of many types of animals (and humans) as a neurotransmitter—a chemical message released by nerve cells to send signals to other cells, such as neurons, muscle cells and gland cells.

People with which of the following genetic disorders are at the highest risk for developing Alzheimer's disease? A. Turner syndrome B. Sickle-cell disease C. Down syndrome D. Williams syndrome

C. Down syndrome

What type of display allows the user to access a large amount of vocabulary in one device, uses spelling to convey the message, is changeable by the user, depicts language in electronic form, uses context-based pages, and uses conversational pages? A. Static B. Hybrid C. Dynamic D. Synthetic

C. Dynamic

Communication deficits are common in TBI. Which disorder occurs in one third of the TBI population? A. Aphasia B. Apraxia C. Dysarthria D. Dementia

C. Dysarthria

Darren, a client with cerebral palsy who was just added to your caseload, presents with uncontrollable movements that are rapid and jerky. You observe that Darren is experiencing trouble controlling movement in his hands, arms, feet, and legs. It appears that his tone changes throughout the day. What type of cerebral palsy does Darren have? A. Spastic B. Ataxic C. Dyskinetic D. Mixed

C. Dyskinetic

Following an evaluation of Louise, an 85-year-old patient who was admitted to a hospital with NPO status, you determined that she presented with severe pharyngeal phase dysphagia and added her to your caseload. Louise's surgeon contacted you inquiring about her ability to tolerate a diet to assist in his decision regarding the continuation of her feeding tube. The surgeon noted that the type of feeding tube that Louise currently had was intended for more short-term use and he was considering inserting a more permanent non-oral feeding method. Louise was most likely receiving nutrition via the following non-oral feeding method: A. Esophagostomy B. Gastrostomy C. Nasogastric D. Pharyngostomy

C. Nasogastric

_____________ is a central cognitive principle. This principle involves scheme, which defines the way humans organize behavior into identifiable patterns. A. Equilibrium B. Imitation C. Organization D. Adaptation

C. Organization

Extralinguistic aspects of human communication includes nonverbal features that accompany oral production of language and modify the actual meaning. This form of human communication also includes paralinguistic codes and nonlinguistic cues. What is the difference between paralinguistic codes and nonlinguistic cues? A. Paralinguistic codes are nonspeech behaviors that accompany the speaker's words and nonlinguistic cues are melodic components of speech. B. Paralinguistic codes transmit cues though facial expressions and nonlinguistic cues indicate stress or suprasegmentals in speech. C. Paralinguistic codes are the melodic components of speech and nonlinguistic cues are nonspeech behaviors that accompany the speaker's words. D. Paralinguistic codes and nonlinguistic cues are different words but have the same meaning.

C. Paralinguistic codes are the melodic components of speech and nonlinguistic cues are nonspeech behaviors that accompany the speaker's words.

When evaluating the voice of an individual with laryngeal cancer, what is the most valuable tool in the assessment of a voice disorder? A. Case history B. Past medical history C. Patient interview D. Treatment protocol

C. Patient interview

The principle of "transference plasticity" with regard to experience-dependent plasticity means which of the following: A. The training experience must be adequately noticeable to induce plasticity. B. Failure to drive precise brain functions can result in functional degradation. C. Plasticity as a result of one training experience can improve the acquisition of comparable behaviors. D. Plasticity as a result of one experience can impede the acquisition of other behaviors.

C. Plasticity as a result of one training experience can improve the acquisition of comparable behaviors.

You are treating a 4-year-old boy with specific language impairment (SLI) for intervention. You notice that he omits all grammatical morphemes in his speech. Which one of the following morphemes would you target first in therapy with him? A. Articles a, an, the B. Regular plural -s C. Present progressive -ing D. Regular past tense -ed

C. Present progressive -ing

The most basic requirement for the caregiver-infant attachment is ___________________ between caregiver and infant. A. Smiles B. Touch C. Recognition D. Shelter

C. Recognition

Unawareness is common among patients with traumatic brain injuries and can affect intervention on task performance in all of the following except when patients _______________. A. Begin tasks without planning or setting goals B. Focus on irrelevant aspects of tasks C. Recognize errors D. Do not spontaneously initiate strategies

C. Recognize errors

You are working with a child who stutters and are training the parents in indirect therapy. Which of the following is most appropriate? A. Response cost B. Fluent stuttering method C. Reducing time pressures for the child to speak D. Fluency reinforcement method

C. Reducing time pressures for the child to speak

Which of the following is not a speaker-oriented technique that is specific to flaccid dysarthria? A. Increasing subglottal air pressure B. Improving posture C. Relaxation exercises D. Establishing maximum breath groups for speech

C. Relaxation exercises

Resonance is a property of the vocal tract. Resonance exists whether it is energized or not. What kind of effect does resonance have on speech? A. Resonance does not modify the energy of a speech signal. B. Resonance adds energy to the speech signal. C. Resonance does not add energy to the speech signal. D. It is still unknown.

C. Resonance does not add energy to the speech signal.

A 73-year-old patient presents with a language disorder characterized by empty, spontaneous speech and loss of word meanings as well as a perceptual disorder characterized by agnosia, yet he maintains his ability to read aloud and write regular words to dictation orthographically. These symptoms would most likely lead to a diagnosis of: A. Vascular dementia B. Pick disease C. Semantic dementia D. Alzheimer's disease

C. Semantic dementia

Speech is a dynamic process requiring precisely coordinated articulatory movements for the sequencing of sounds to ultimately produce words. What is common among children with cerebral palsy? A. Apraxia B. Flaccidity C. Speech impairment D. Hyponasality

C. Speech impairment

In the past 20 years, there have been changes to the Medicare statute that affect speech-language pathologists. Under which change was there a 1.5% increase in the Medicare payment schedule for services paid? A. The Medicare, Medicaid, and SCHIP Balanced Budget Refinement Act of 1999 (BBRA) B. The Medicare, Medicaid, and SCHIP Benefits Improvement and Protection Act of 2000 (BIPA) C. The Medicare Prescription Drug, Improvement, and Modernization Act of 2003 (MMA) D. The Medicare Percentage Based Payment Act of 1998

C. The Medicare Prescription Drug, Improvement, and Modernization Act of 2003 (MMA)

Which model contends that a child who is unable to cope with the expectations of fluent speech production may begin to stutter? A. The diagnosogenic model B. The expectancy deconfirmation model C. The demands and capacities model D. The approach-avoidance model

C. The demands and capacities model

You have been referred an 8-year-old boy because the teacher suspects that he stutters. Before rendering a diagnosis, you wish to determine the different kinds of disfluencies he exhibits. You have taken an extended speech sample, and you are now counting the different forms of disfluencies. Select the following statement that is correct in measuring the types and the number of disfluencies in specified utterances. A. The utterance "I went-went-went to uh p-p-play videogames with my ffffriend on SSSSS well um Sa-Sa-Sa-Saturday" contains three word repetitions, two interjections, and two syllable prolongations. B. The utterance "I-I-I could not do-do-do it because I was af-af-afraid of it" contains two sound repetitions and one word repetition. C. The utterance "I wanted to-I wanted to-I wanted to ssssssee what um my mmmmmom was d-d-d-doing" contains one phrase repetition, two sound prolongations, one sound interjection, and one part-word (sound) repetition. D. The utterance "He-he-he didn't tell me anything be[silence]fore he [silence] before he d-d—decided to do it" contains one word repetition, two pauses, and one sound repetition.

C. The utterance "I wanted to-I wanted to-I wanted to ssssssee what um my mmmmmom was d-d-d-doing" contains one phrase repetition, two sound prolongations, one sound interjection, and one part-word (sound) repetition.

A researcher teaches a new book reading program to caregivers of children on the autism spectrum and evaluates the children's literacy skills one year later. The researcher's goal is to evaluate whether or not there is a relationship between caregivers' implementation of the program and children's literacy skills. The researcher finds that there is an r =.15 correlational relationship between caregivers' reported implementation of the program and children's literacy skills. The researcher can safely conclude that: A. There is a strong positive correlation between the caregivers' implementation of the program and children's literacy skills. B. There is a strong negative correlation between the caregivers' implementation of the program and children's literacy skills. C. There is no significant relationship between the caregivers' implementation of the program and children's literacy skills D. There is a mildly significant cause-effect relationship between the caregivers' implementation of the program and children's literacy skills.

C. There is no significant relationship between the caregivers' implementation of the program and children's literacy skills

Even with a proper diet, in which stage of Alzheimer's disease would you see the patient begin to develop swallowing problems and demonstrate significant weight loss? A. First stage B. Second stage C. Third stage D. Fourth stage

C. Third stage

A clinician is measuring communicative behaviors in a child with a cleft palate. The clinician measures the time intervals during which the speech behaviors selected for observation occurred. What are the methods of measurement called? A. Latency measures B. Self-report measures C. Time sampling measures D. Covert measures

C. Time sampling measures time latency (RTL), defined as "the time between speaker turn exchanges" self-report study is a type of survey, questionnaire, or poll in which respondents read the question and select a response Time Sampling: Refers to a variety of methods to record behavior at specific moments covert measures, observations of behaviors such as facial expressions, voice tone, and body language.

When focusing on caregiver-infant attachment, it is important that the dyad relationship is properly assessed before exploring treatment options. From the choices provided, what would best constitute a dyad? A. Non-biological B. Emotional C. Two-way D. Biological

C. Two-way

There are several aspects involved in studying language. These include using language as a social tool, a learned behavior, and/or a linguistic means of communication. Which of the following defines language as a learned behavior? A. Verbal language that is learned through trial and error B. Verbal language that is learned through imitation C. Verbal language that is learned through social consequence D. Verbal language that is learned through intentions

C. Verbal language that is learned through social consequence

Which of the following factors is most important when designing an augmentative and alternative communication (AAC) system for individuals with progressive supranuclear palsy? A. Rigidity involving upper limbs B. Rigidity involving lower limbs C. Visual impairment D. Hearing deficits

C. Visual impairment

The role of a newborn developmental specialist focuses primarily on all of the following, except: A. Feeding and oral-motor development B. Aural rehabilitation C. Voice disorders and treatment D. Caregiver-child communication

C. Voice disorders and treatment

Which of the following components of communication is usually targeted when using augmentative and alternative communication (AAC) intervention in individuals with spinal cord injuries? A. Verbal communication B. Visual impairment C. Written communication D. Hearing deficits

C. Written communication

You are a new clinician in a hospital that has a neonatal intensive care unit (NICU). Your job title is that of newborn development specialist (NDS). Which one of the following would be false regarding your role as an NDS? A. You would support the families and infants with issues involving infant development and behavior. B. You would support the families and infants with issues involving hearing conservation and aural habilitation. C. You would not serve as primary care coordinator or case manager for the infants and families because a medical doctor usually fills this role. D. You would create individualized assessment and intervention strategies to support infants and their caregivers.

C. You would not serve as primary care coordinator or case manager for the infants and families because a medical doctor usually fills this role.

Imitation is important for the growth of language development in infants. Before assessing an infant with a language disorder, it is important to understand the imitation that occurred between the caregiver and the infant. How long immediately after birth can an infant imitate? A. 3 weeks B. 1 month C. a few hours D. 1 week

C. a few hours

A clinician is teaching a patient a technique for dysphagia that includes having the patient take a deep breath, hold the breath, swallow, cough on exhalation, swallow again before breathing, and then breathe again. This technique is called A. the Mendelsohn maneuver. B. an effortful swallow. C. a supraglottic swallow. D. a super-supraglottic swallow.

C. a supraglottic swallow. Super-supraglottic swallow —designed to voluntarily move the arytenoids anteriorly, closing the entrance to the laryngeal vestibule before and during the swallow. The super-supraglottic swallow is similar to the supraglottic swallow; however, it involves increased effort during the breath hold before the swallow, which facilitates glottal closure

Sarah, the mother of 18-month old toddler Brandon, has taken time off her job as an engineer to be with Brandon full time at home. Sarah is very conscientious about Brandon's speech and language development, and brings him to you for an evaluation "just to make sure he's on track." Which of the following would Brandon be expected to demonstrate at 18 months of age if he is developing in a typical fashion? A. 20-30 expressive vocabulary words, mostly consisting of nouns B. establishment of joint reference and 10-15 expressive vocabulary words C. an expressive vocabulary of 50 words as well as beginning to put 2 words together D. primary use of 3-4 word responses and use of "and" to join sentences

C. an expressive vocabulary of 50 words as well as beginning to put 2 words together

20. Brutten and Shoemaker hypothesized that stuttering, as they defined it, A. is caused by classically conditioned negative emotion B. is caused by operant conditioning C. is caused by hereditary factors D. is limited to word and phrase repetitions E. is limited to avoidance reactions

A. is caused by classically conditioned negative emotion

67. Variations in vocal frequency, or frequency perturbation, are known as A. jitter B. shimmer C. amplitude perturbation D. fundamental frequency variation E. frequency change

A. jitter

A patient was receiving irradiation to the oral and pharyngeal areas and was experiencing xerostomia, as well as weight loss and an increase in dental caries. During the evaluation, the clinician recommended that the patient A. swallow once prior to taking a spoonful of food in the mouth. B. swallow twice prior to taking a spoonful of food in the mouth. C. be given synthetic saliva just prior to eating. D. use a chin tuck procedure before swallowing.

C. be given synthetic saliva just prior to eating.

The lowest intensity of a sound that will stimulate the auditory system is called A. sound pressure level. B. decibel. C. hearing level. D. pitch.

C. hearing level.

A clinician at a trauma center receives a referral of a 19-year-old college student, Peter, who was involved in a motorcycle accident. He recently received his motorcycle license. His parents stated, "He is a carefree young man who has always believed he was invincible." The medical chart indicates that Peter was riding his bike at high speeds and was not wearing a helmet when he crashed into a stationary delivery truck. The paramedic report indicates that he was thrown from his bike and went head first into the delivery truck. He was unconscious when he entered the hospital and was rushed to the emergency room. He suffered several fractures. His right leg is in a cast, and he has injuries to several parts of his body. The medical chart also indicates that he has a traumatic brain injury (TBI) and suffers many symptoms of TBI. Emergency surgery was performed to repair hematomas that were a result of the accident. QuestionTopic: TBI The clinician conducts several tests on Peter and notices that he has lost half of his side of vision (right side). This type of vision loss is called A. hemiplegia. B. hemiparesis. C. hemianopsia. D. hemispherectomy.

C. hemianopsia. less vision or blindness hemiplegia-paralysis of one side of the body. Hemiparesis, or unilateral paresis, is weakness of one entire side of the body

Hearing loss that occurs when the middle ear and the inner ear are not functioning properly is known as A. sensorineural hearing loss. B. middle ear hearing loss. C. mixed hearing loss. D. conductive hearing loss.

C. mixed hearing loss.

A clinician is starting a private practice that focuses primarily on accent training for adult clients who speak English as a foreign language (EFL). Which one of the following is NOT an important consideration in the provision of accent training services? A. have one or two unfamiliar listeners listen to clients' speech samples and determine the percent of unintelligible words B. slowing down a client's rate of speech is often effective in making an immediate improvement in intelligibility C. most EFL clients cannot benefit from the use of the Visi-Pitch because the use of this type of technology is confusing D. many EFL clients will appreciate doing activities involving reading and discussing information about their countries of origin

C. most EFL clients cannot benefit from the use of the Visi-Pitch because the use of this type of technology is confusing

An 86-year-old patient has been diagnosed with a delayed swallowing reflex post CVA. The modified barium swallow study indicates that there is pooling in the vallecula until the swallowing reflex has been triggered. The clinician decides to give the patient a mechanical soft bolus and ask her to swallow while putting her head down (chin tuck). This technique will result in A. increasing the change of aspiration. B. narrowing or closure of the vallecula space. C. narrowing of the airway entrance, as well as pushing the epiglottis posteriorly. D. pushing the tongue base forward.

C. narrowing of the airway entrance, as well as pushing the epiglottis posteriorly.

A premature infant was referred for a speech-language evaluation. During the evaluation, the speech-language pathologist noted that the infant had difficulty bringing her hands to her mouth to initiate sucking. While breast feeding, the clinician noted that the infant had bursts and pauses, with about two up-and-down cycles of the jaw per second. This is called A. mouthing. B. sequential sucking. C. non-nutritive sucking. D. nutritive sucking.

C. non-nutritive sucking.

You are treating a patient who has been diagnosed with Alzheimer's disease. You decide to use an emotion-oriented therapy by playing audio recordings of relatives of the patient. You believe that this approach will decrease the agitation and improve the well-being of the patient. This type of approach is called A. reality orientation (RO). B. fast mapping therapy (FMT). C. simulated presence therapy (SPT). D. cognitive stimulation therapy (CST).

C. simulated presence therapy (SPT).

Which of the following signs of dysphagia is the most common oral-stage observation when assessing a patient with Parkinson's Disease? A.Repeated nonpropulsive lingual movements B.Eating quickly and impulsively C.Spilling liquids out of mouth because of poor labial seal D.Holding food in mouth because of poor sensation

A.Repeated nonpropulsive lingual movements

What is one of the reasons that production of speech sounds is restricted in infants? A. When a human is born, an inadequate larynx-to-lung ratio is present. The lungs are too small to create enough subglottal pressure for the infant's larynx; therefore, phonation is unable to occur. B. Infants do not have any teeth, so their phoneme inventory is limited to sounds other than dentals. C. An infant's tongue completely fills the oral cavity and the pharyngeal cavity is used for sucking and swallowing; therefore, the larynx and vocal tract are unable to perform any secondary functions. D. Infants breathe at a slow rate yet are unable to completely expand their rib cage. This means that infants are unable to create enough positive and negative pressure to perform the task of respiration with enough power to create phonation.

C. An infant's tongue completely fills the oral cavity and the pharyngeal cavity is used for sucking and swallowing; therefore, the larynx and vocal tract are unable to perform any secondary functions.

A 15-year-old high school sophomore with th/s substitution comes to you for therapy. She is frustrated because she wants to act in high school plays but has been told she cannot do that because of her "speech problem." She is highly motivated to produce /s/ correctly, and you begin seeing her for therapy. If you are using Van Riper's approach, what will you do? A. Begin therapy at the syllable level, focusing especially on principles of coarticulation as you help her produce /s/ accurately in various contexts. B. Use a training approach that emphasizes three phases: establishment, transfer, and maintenance and that heavily emphasizes imitation. C. Focus on phonetic placement, auditory discrimination, and drill-like practice at increasingly complex motor levels until accurate /s/ production is automatized. D. Use a cycles approach in which you do not drill /s/ to mastery but rather introduce the correct way to produce /s/, give the student limited practice producing /s/, and use auditory bombardment.

C. Focus on phonetic placement, auditory discrimination, and drill-like practice at increasingly complex motor levels until accurate /s/ production is automatized.

A researcher wishes to assess the efficacy of the new Riverton method of training children to say /r/ accurately. He decides to use a single-subject design because he has a private practice where he serves a number of children who have w/r substitutions. The researcher needs to keep several concepts in mind as he begins his research. Which one of the following concepts is false? A. An example of a multiple-baseline-across-settings design would be teaching a behavior (e.g., correct /r/ production) sequentially in different settings to demonstrate that the behavior changes only in a treated setting and thus treatment is effective. B. In a multiple-baseline-across-subjects design, several subjects are taught a behavior sequentially to show that only treated subjects change, and thus the treatment is effective. C. In the ABAB withdrawal design, a target behavior is base rated (e.g., /r/ in the A phase), taught to the subject (e.g., /r/ is accurately produced in the B phase), reduced by teaching its counterpart or an incompatible behavior (e.g., teaching w/r in the A phase), and then taught again (e.g., /r/ is accurately produced in the B phase) to show that the treatment is effective. D. For this researcher, an advantage of using a single-subject design to evaluate the efficacy of the Riverton method in training children to say /r/ accurately is that he can integrate research and clinical service by using the clients he serves as subjects in an experiment that attempts to answer a significant clinical question.

C. In the ABAB withdrawal design, a target behavior is base rated (e.g., /r/ in the A phase), taught to the subject (e.g., /r/ is accurately produced in the B phase), reduced by teaching its counterpart or an incompatible behavior (e.g., teaching w/r in the A phase), and then taught again (e.g., /r/ is accurately produced in the B phase) to show that the treatment is effective.

All of the following could be techniques used in therapy when treating a pediatric case of tongue protrusion/thrusting except? A. Tongue tapping B. Facial molding C. Pressure to the gums and lips D. Cheek support

C. Pressure to the gums and lips

After suffering from a severe stroke, Robert has difficulty with auditory comprehension but speaks fluently, though he does not make much sense. Which area of Robert's brain was affected by the stroke? A. The frontal lobe B. The occipital lobe C. Wernicke's area D. The angular gyrus

C. Wernicke's area

The lowest frequency of a periodic wave is A. natural frequency. B. formant frequency. C. fundamental frequency. D. displacement frequency.

C. fundamental frequency.

A teacher has referred Jeremy to you for an evaluation. Jeremy is an African American first grader who is reportedly doing well in class academically. When you observe him on the playground with his peers, you see that he has many friends and does not appear to have problems interacting appropriately with other children. His friends do not appear to have any difficulty understanding what he says. However, the teacher is concerned. She says, "I think Jeremy pronounces some of his sounds wrong. I think he needs speech therapy." When you conduct a speech screening with Jeremy, you will remember that which one of the following patterns indicates a possible disorder, not a difference, based on Jeremy's use of African American English (AAE)? A. f/th substitution in word-final position B. Production of [ks] instead of [sk] C. th/s substitution in all word positions D. Differing stress on some words (e.g., police instead of police)

C. th/s substitution in all word positions

17. At what age should a typically developing child be able to understand agent-action relationships? A. 2-3 years B. 5-6 years C. 3-4 years D. 6-7 years E. 8+ years

C. 3-4 years

According to the myoelastic-aerodynamic theory, vibration of the vocal folds depends on which four of the following factors?

Airflow, Elasticity, Muscle action, Pressure

The uses of narratives characterized by causally sequenced events is typically seen in

children over the age of 5

The radiographic imaging procedure that allows X-ray beams to circle through segments of the brain and pass through tissue while a camera takes pictures of sections of the structures being scanned is known as A. electroencephalography (EEG). B. computerized axial tomography (CAT) scan. C. magnetic resonance imaging (MRI). D. positron emission tomography (PET) scan.

computerized axial tomography (CAT) scan.

Craniosynostosis

condition in which one or more of the sutures close too early, causing problems with normal brain and skull growth.

A speech language pathologist will receive compensation for utilizing a specific brand of thickener in her private practice. Which ethical dilemma is present in this scenario? negligence conflict of interest fraud misrepresentation

conflict of interest Conflict of interest is defined as interference of professional practice due to personal gain.

Reliability

consistency with which a test measures or the degree to which repeated measures or the degree to which repeated measurement with the same instrument of the same individual would produce same result, .90 or greater is good

Nonlinguistic

conveys information without the use of language. Examples of Nonlinguistic Communication includes gestures, body postures, facial expression, eye contact, head and body movements, and physical distance or proxemics.

Cole is small for his age and has an elfin-like appearance characterized by a small chin, turned-up nose, puffiness around the eyes, a long upper lip, and a wide mouth. Cole's teeth are small and widely spaced. Question Topic: General Eval This syndrome is caused by a rare genetic disorder that affects an estimated 1 out of every 20,000 babies. It is caused by what? A. An abnormality on chromosome 7, including a gene that makes the protein elastin B. Autosomal dominant inheritance and deletion in the region of the long arm of chromosome 15 (15q11-15q13) C. A spontaneous autosomal dominant mutation whose gene and locus is FGR2 at 10q25-26 D. An expanded number of cytosine-guanine-guanine (CGG) nucleic acid repeats on a specific gene on one of the distal ends of the X chromosome

An abnormality on chromosome 7, including a gene that makes the protein elastin

Anosognosia

deficit of self-awareness, a condition in which a person with a disability is unaware of having it. It was first named by the neurologist Joseph Babinski

115. You have been asked to evaluate a kindergartener, Tommy, who is 5 years old. He attends a public school. According to Tommy's teacher, Tommy "is real quiet; when he talks, he usually only says 2 or 3 words at a time. I don't know if he will outgrow this or whether he is just shy, or what." The teacher is concerned because it is April of Tommy's kindergarten year, and she is wondering if his oral language skills will be sufficient for first grade in the fall. You screen Tommy and also observe him in the classroom, on the playground at recess, and in the school cafeteria at lunchtime. You find that Tommy rarely interacts with his peers and eats and plays alone. He does not appear to have any friends. When you talk with Tommy's parents, they tell you that Tommy did not speak his first word until he was almost 2 years old and did not put words together until he was over 3 years old. Tommy's parents state that "he has always been real well-behaved, and we never thought there was a problem." They reveal that they did not place Tommy in preschool because they felt that he might be "picked on by other children because he was so shy." You gather a language sample, and find that Tommy often says sentences like "My shoes hurting my feet today." This sentence has A. 6 words, 6 morphemes B. 6 words, 9 morphemes C. 6 words, 8 morphemes D. 6 words, 10 morphemes E. 6 words, 7 morphemes

C. 6 words, 8 morphemes

What is the muscle that exerts the pull that allows the eustachian tube to open during yawning and swallowing? A. Tensor palatini B. Levator palatini C. Tensor tympani D. Levator veli palatini

. Tensor palatini

A speech language pathologist is treating a patient who just underwent a total laryngectomy. Which of the following is not an appropriate speaking option for the patient? speaking valve electrolarynx TEP esophageal speech

1. Speaking valves redirect air to the vocal folds. A laryngectomee no longer has vocal folds therefore, this option would not be effective.

How many expressive words does a 15 month old have

10 words

112. An SLP must assess receptive language and expressive language in a developing language learner. Which of the following standardized assessments is best for each task? For each row, select all that apply. Some rows may be empty. Assessment Receptive language Expressive language Peabody Picture Vocabulary Test—Revised Clinical Evaluation of Language Fundamentals—Preschool Preschool Language Scales

112. An SLP must assess receptive language and expressive language in a developing language learner. Which of the following standardized assessments is best for each task? For each row, select all that apply. Some rows may be empty. Assessment Receptive language Expressive language Peabody Picture Vocabulary Test—Revised X Clinical Evaluation of Language Fundamentals—Preschool x X Preschool Language Scales x X The correct responses are, in order, receptive language, receptive language and expressive language, receptive language and expressive language. The Peabody Picture Vocabulary Test—Revised is used to assess receptive language skills. However, the Clinical Evaluation of Language Fundamentals—Preschool and the Preschool Language Scales can be used to asses both receptive and expressive language skills.

Which of the following lists the steps of the evidence-based practice process in the correct order? Make clinical decision → Generate clinical question → Evaluate evidence → Locate evidence Generate clinical question → Locate evidence → Make clinical decision → Evaluate evidence Generate clinical question → Locate evidence → Evaluate evidence → Make clinical decision Locate evidence → Generate clinical question → Evaluate evidence → Make clinical decision

3. A question must be generated to know what evidence to look for and what information to assess within the evidence. After evaluation of the evidence, a clinical decision can be made.

Blake, a speech language pathologist, is creating his assessment plan for a pediatric evaluation he has later this week. Which of the following is an example of a formal measure used during assessment? language sample criterion referenced test developmental scale behavioral observation

3. Developmental scales are formal assessment measures because they contain psychometric properties and assess overall achievement/ mastery of skill compared to peers.

Luke was born at 32 weeks weighing 3.5 lbs. He is now 9 months old and is exhibiting difficulty putting on weight. Luke is also experiencing symptoms of GERD and is hypotonic. When his mom turns on the lights, claps her hands, or sings a nursery rhyme, Luke barely reacts. Which of the following types of bottles would be most beneficial for Luke? a vented bottle to decrease the amount of air Luke receives a bottle with a slow flow nipple a brightly colored, angled bottle a soft bottle used for external pacing

3. Luke would benefit from a brightly colored bottle since he is hyporeactive to stimulus items in his environment. An angled bottle would allow Luke to sit more upright during feeding times to help reduce symptoms of GERD.

Luke was born at 32 weeks weighing 3.5 lbs. He is now 9 months old and is exhibiting difficulty putting on weight. Luke is also experiencing symptoms of GERD and is hypotonic. When his mom turns on the lights, claps her hands, or sings a nursery rhyme, Luke barely reacts. What is Luke's adjusted age? 5 months 6 months 7 months 8 months

3. Since Luke was born 8 weeks early (40 weeks- 32 weeks), and estimated 2 months must be subtracted from his age. Example: 9 months (his current age) - 2 months (how many weeks he was premature) = 7 months.

Katie is a full time vocal performer. She notices a problem with her voice and schedules an appointment with the ENT where she is diagnosed with vocal nodules. What type of vocal disorder was Katie diagnosed with? neurogenic voice disorder psychological voice disorder structural voice disorder conversion voice disorder

3. Structural voice disorders result in physical changes in the tissue.

Which of the following would be commonly observed in a typically developing 24-month-old child? 90% speech intelligibility 4-word combinations beginning to follow 1-step directions an MLU around 2.0

4. A MLU around 2.0 is commonly observed in 24-month-olds.

A speech language pathologist is evaluating a child who only speaks Spanish. Which of the following is not an erroneous assessment strategy for the SLP to utilize for the linguistically diverse patient? translating the standardized test to Spanish modifying the standardized test to make more relevant for the specific culture relying on language sampling/observation including an ethnographic approach

4. An ethnographic approach encourages the clinician to utilize the child's family for important information and promotes comparison of the child's language to expectations of the culture (e.g., gender roles, age roles, and syntax structure).

Kennedy is a 57-year-old woman who has noticed increased muscle fatigue during meals. After resting, the fatigue appears to disappear. Kennedy went to the neurologist and was diagnosed with damage to the acetylcholine receptors at the neuromuscular junction. What disease is Kennedy likely diagnosed with? Parkinson's disease Alzheimer's disease muscular dystrophy myasthenia gravis

4. Myasthenia gravis is caused when there is a miscommunication at the neuromuscular junction. The disease usually first targets jaw, facial, and neck muscles.

Which of the following is associated with a posterior cerebral artery (PCA) stroke? apraxia non-fluent aphasia fluent aphasia hemianopia

4. Typically, a posterior cerebral artery stroke results in hemianopsia. The PCA supplies blood to the occipital lobe which is responsible for vision.

How many areas are likely to be included in speech/language screening? 5 8 6 4

8 Explanation: There are 8 areas that may be included in a thorough speech/language screening. These include: articulation, phonology, comprehension, expression, voice, resonance, fluency, and pragmatics. In addition, an SLP may also complete a hearing screening.

mneumonic devices

memory aids, especially those techniques that use vivid imagery and organizational devices, JDS

Aprosodia

neurological condition characterized by the inability of a person to properly convey or interpret emotional prosody. Prosody in language refers to the ranges of rhythm, pitch, stress, intonation, etc

Which of the following activities is an example of a metalinguistic strategy used to assist a student with language difficulties? • A.Providing a definition of a word • B.Placing story elements in sequential order • C.Editing the writing of same-aged peers • D.Discussing similarities and differences between two pictures

Metalinguistics, or meta - awareness skill is to do with the ability of a person to reflect on and consciously ponder about oral and written language and how it is used. Option (C) is correct. Editing the writing of a same-age peer is an example of a metalinguistic activity because it involves the awareness, analysis, and control of language form, content, and use.

List the parts of the brainstem from superior to inferior:

Midbrain, pons, medulla

tympanometry

Next, they'll place a probe-type device in your ear canal. It may feel a little uncomfortable, and you may hear loud tones as the device begins to take measurements. This test changes the air pressure in your ear to make the eardrum move back and forth. Measurements of the movement of your eardrum are recorded in a tympanogram.

Non-iconic symbols are

Non-iconic symbols are geometric, abstract, and arbitrary and must be specifically taught.

The incidence of a disorder is defined as the A prevalence B number of new cases of the disorder reported within a certain time C most common etiology of the disorder D number of cases of the disorder that were successfully treated

Number of new cases of the disorder reported within a specific period of time

Which of these concern(s) the patterns, systems, and rules of speech sounds in a language? All of these Phonemics Phonetics Phonology

Phonology is the branch of linguistics concerning the patterns of speech sounds, their systems, and their rules in any given language. Phonetics is the branch concerning the production, representation, and acoustic properties of speech sounds. Phonemics is the branch concerning the study and development of the specific phonemes (= speech sounds) in any given language. Because only Phonology is correct, "All of these" is not.

Of the following theories of cognitive development, whose posits discrete developmental stages? Piaget's Bandura's Vygotsky's Skinner's

Piaget's theory posits four distinct stages of cognitive development, each with specific distinguishing characteristics and corresponding to approximate age ranges. Skinner's theory of cognitive development is behavioral. It does not include stages but focuses on the premise that changes in behaviors over time represent learning and that this learning occurs through the antecedent and consequent events immediately before and after a behavior, which increase or decrease the probability of the individual's repeating the behavior. Vygotsky's theory also has no stages and focuses on sociocultural influences as sources of learning. Bandura's social learning theory has no stages either; it focuses on learning through observation, imitation of models, and vicarious learning.

As the frequency of a sound goes up, listeners generally perceive the __________ of the sound to also go up. Displacement. Resonance. Pitch. Amplitude.

Pitch.

Projection Fibers

Projection fibers consist of both afferent and efferent nerve fibers that couple the brain with the spinal cord. These projection fibers are myelinated nerves that are arranged in columns within the white matter surrounding the gray matter. The dorsal column relays somatic sensory information to the brain.

F1

Prominent band in trachea, shape of trachea

Spastic cerebral palsy

jerky, uncontrolled movements and high muscle tone

Particular frequencies of sound are selectively reinforced when traveling through objects. This property is known as: Harmonic distortion. Resonance. Oscillation. Periodicity.

Resonance.

An SLP determines the mean length of utterance (MLU) of a language sample from a three-year-old child. Two weeks later, the SLP reevaluates the same sample and again determines the MLU. The extent to which the two scores are similar is most directly a function of the (A) validity of the scores (B) reliability of the scores (C) skewness of the score distribution (D) speededness of the measure

Reliability of the scores

Which of the following nerves provides efferent innervation to the stylopharyngeus muscle and contributes toward the elevation of the pharynx and the larynx? • A.The trigeminal • B.The hypoglossal • C.The vagus • D.The glossopharyngeal

Option (D) is correct. The functions described are controlled by cranial nerve IX9, the glossopharyngeal.

superior temporal gyrus

The superior temporal gyrus contains the auditory cortex, which is responsible for processing sounds. The superior temporal gyrus also includes the Wernicke's area, which (in most people) is located in the left hemisphere. It is the major area involved in the comprehension of language. The superior temporal gyrus is involved in auditory processing, including language, but also has been implicated as a critical structure in social cognition

Wide band spectrograms are best for viewing formant patterns. True False

True

1. Place the phonemes below in the order of typical acquisition. · /p/ - · /k/ - · /θ/ - · · /ʒ/ -

· /p/ - 1 · /θ/ - 3 · /k/ - 2 · /ʒ/ - 4

131. Which of the following is the most common phonological problem evidenced by young children aged 18-29 months? • Cluster reduction • Velar fronting • Nasal assimilation • Dimunitization

· Option (A) is correct. A summary of various studies has shown this to be the most prevalent pattern along with liquid gliding

1. A primary factor causing the patient's psychogenic aphonia is the presence of depression Bell's palsy COPD Hallucinations

· Option (A) is correct. Depression can lead to psychogenic disorders and given what this patient has been through, it is very likely that his depression has deepened during his hospital stay.

1. In Bloom and Lahey's model, morphology is considered form content use semantics

· Option (A) is correct. The Bloom and Lahey model classifies morphology as form. Content-semantics, syntax use-pragmatics

99. Mr. Aviz, a 62-year-old chemist, is recovering from hip-replacement surgery following a spontaneous fracture. The surgery was completed without complications. He is now in his second postoperative day, and his wife reports alterations in his memory and some confusion not previously noted. Which of the following is the most reliable screening instrument for the SLP to use in this case? • The Mini-Mental State Examination (MMSE) • The Test of Problem Solving (TOPS-3) • The Cognitive Linguistic Quick Test (CLQT) • The Global Deterioration Scale (GDS)

· Option (A) is correct. The MMSEM M S E is the only screening tool in the list appropriate for the age and condition described

69. An SLP works with a patient who has severe dysphagia. The patient is on a pureed diet and honey-thick liquids. During treatment, the patient becomes tearful and expresses frustration. The patient says, "I feel like I'm not making any progress and I'm never going to eat normal food again. I hate this diet, and I hate doing therapy. I just want to give up." The SLP responds by saying, "What you're feeling is normal, and I understand. You're doing everything you can right now to get better, and giving up isn't the answer." The SLP reaches across the table and places a comforting hand on the patient's arm and has a caring facial expression. Which of the following principles of counseling is the SLP primarily demonstrating? • Showing congruence • Having unconditional positive regard • Offering the patient sympathy • Providing emotional support

· Option (A) is correct. The SLP is demonstrating congruence by using words and body language that match.

90. The acoustic reflex is triggered in a person with typical hearing when the listener is exposed to a sound above approximately • 85 dB HTL • 130 dB HTL • 1000 Hz • 6500 Hz

· Option (A) is correct. The acoustic middle ear reflex in a person with typical hearing usually occurs at about 85 dB HL.

1. A large metropolitan school district wants to determine the prevalence of developmental stuttering among all enrolled students during the past year. Which of the following approaches is most appropriate for accomplishing the task? -Dividing the total number of students who currently stutter by the total number of students who were enrolled during the past year -Subtracting the total number of students who received treatment for stuttering during the past year from the total number of students who currently stutter -Multiplying the total number of students who currently stutter by the total number of students enrolled during the past year -Dividing the total number of newly identified students who stutter by the total number of students enrolled during the past year

· Option (A) is correct. The prevalence of stuttering is the percentage found by dividing the number of students who currently stutter by the number of students enrolled in school.

1. Which of the following sounds are typically mastered by the time a child turns 3? /k/ and /g/forward slash k forward slash and forward slash g forward slash /p/ and /b/forward slash p forward slash and forward slash b forward slash /l/ and /s/forward slash l forward slash and forward slash s forward slash /ch/ and /sh/

· Option (B) is correct. /p/ and /b/forward slash p forward slash and forward slash b forward slash should be mastered by age 3 in a typically developing child.

37. An SLP administers a language test to all kindergarten children in a particular school and finds that 40 percent fall below the tenth percentile of the normative sample. Which of the following is the most reasonable interpretation of this finding? • Forty percent of the kindergarten children in the school have language disorders. • The school's kindergarten population differs from the standardization population in the language skills measured. • The SLP administered the test incorrectly. • The standardization population is not representative of the nation as a whole

· Option (B) is correct. If disparities exist between examinees and the norm group in terms of skills and experiences, the conclusions based on the examinees' test performance may be misleading.

1. Sensorineural hearing loss resulting from Ménière's disease causes a sudden hearing loss without warning fluctuating levels of hearing loss complete hearing loss at all frequencies bilateral progressive hearing loss

· Option (B) is correct. Ménière's disease is a disorder of the inner ear and causes fluctuating hearing loss.

1. The commonly used chin-down posture was initially developed to eliminate thin-liquid aspiration in people with delayed pharyngeal stage onset after having a stroke, and when its efficacy was investigated, it was found to be 50% effective. For whom is the chin-down posture ineffective at eliminating thin-liquid aspiration? People who have cricopharyngeal dysfunction People who aspirate residue from the pyriform sinuses People who have impaired lingual function People who aspirate residue from the valleculae

· Option (B) is correct. Shanahan et al. (1993) found that 50 percent of people with stroke- and swallowing-related aspiration due to delayed onset of the pharyngeal response continued to aspirate from the pyriform sinuses using the chin-down posture. Moreover, Eisenhuber et al. (2002) found that people with a pyriform sinus residue with a height that was less than 50 percent of the pyriform sinus height aspirated significantly more than those with less or no pyriform sinus residue.

1. Which of the following structures is a relay center for sensory information? The reticular formation The thalamus The putamen The hippocampus

· Option (B) is correct. The thalamus is the major relay system for sensory information.

What dysphagia screening test for aspiration is not supported by ASHA? Bedside dysphagia eval 3 oz. water swallow test FEES Questionairre

3 oz. water swallow test

Which of the following types of cerebral palsy is characterized by low muscle tone, impaired balance, and tremor A ataxic B spastic C athetoid D hemiplegic

A ataxic

130. Place a check in the appropriate column below to indicate whether the characteristic listed is symptomatic of stuttering or cluttering. Characteristic Stuttering Cluttering Excessive frequency of part- and whole-word repetitions Excessive frequency of revisions Active attempts to avoid or conceal communication difficulties Reduced intelligibility in conjunction with rapid rate Misarticulation of multisyllable words Limited concern and awareness of communication difficulties Use of word avoidance and circumlocution in response to anticipated disfluency

130. Place a check in the appropriate column below to indicate whether the characteristic listed is symptomatic of stuttering or cluttering. Characteristic Stuttering Cluttering Excessive frequency of part- and whole-word repetitions X Excessive frequency of revisions X Active attempts to avoid or conceal communication difficulties X Reduced intelligibility in conjunction with rapid rate X Misarticulation of multisyllable words X Limited concern and awareness of communication difficulties X Use of word avoidance and circumlocution in response to anticipated disfluency X The characteristics of stuttering are excessive frequency of part- and whole-word repetitions, active attempts to avoid or conceal communication difficulties, and use of word avoidance and circumlocution in response to anticipated disfluency. The characteristics of cluttering, on the other hand, are excessive frequency of revisions, reduced intelligibility in conjunction with rapid rate, misarticulation of multisyllable words, and limited concern and awareness of communication difficulties. Any other pairings would be considered incorrect.

13. With a typically developing child who speaks English as a second language, approximately how long does it take for basic interpersonal communication skills (BICS) to develop to a level commensurate with that of native English speakers? A. 1 year B. 5-7 years C. 18 months D. 2 years

2 years

Public law 99-457 encompasses: -providing reimbursement for no more than 30 days in the hospital. -training family members to support the development of a child. -supporting children ages 3 to 21. -providing eligibility for special education for children in middle school.

2. Public law 99-457 implemented Individualized Family Service Plans (IFSP) to increase family member participation in a child's intervention plan.

Austin is a 24-month-old toddler who is utilizing gestures to provide additional meaning to his utterances. For example, he will point to the back door while saying, "go!". Which of the following is true regarding Austin's use of gestures? Austin's use of gestures does not predict his use of multi-word utterances. Austin is using supplementary gestures. Austin's caregivers should ignore his gestures and reinforce spoken words. Austin is using complementary gestures.

2. Supplementary gestures add information to spoken utterances. Pointing to the back door allowed the caregiver to know exactly where Austin wanted to go. Without the gesture, the caregiver would have been responsible for guessing what Austin wanted.

Which of the following might be included in a treatment plan for a child with cleft palate? targeting posterior sounds targeting increased oral awareness targeting use of nasal sounds targeting marked sounds

2. To reduce nasality, promoting oral awareness is crucial with this population.

Many children will present with conductive hearing loss, either unilateral or bilateral. This means that they are having difficulty based on an outer or middle ear problem. If the child has hearing thresholds of 40-50db, what are the frequencies that are likely to be affected? 500 - 4000hz 1500 - 3000hz 250 - 6000hz 1000 - 5000hz

250 - 6000hz Explanation: A conductive hearing loss can affect speech sound frequencies across the entire speech range. Often it can be medically treated and always requires medical evaluation for causes such as otitis media, perforated eardrum, impacted cerumen and Eustachian tube dysfunction. Many conditions are reversible.

Amemory consists of remembering facts and events, and refers to the memories that can be consciously recalled ? 1Procedural 2Short term 3Declarative

A => 3

between-subjects design

A research design in which different groups of participants are randomly assigned to experimental conditions or to control conditions.

Wihc of the following types of fibers facilitates communication between the right and left hemispheres by connecting cortical areas in the two hemispheres? A association B commissural C efferent D afferent

B commissural

Which of the following is true about supporting a young childs acquisition of English as a second language? A a focus in the family on the English-speaking culture as well as the languaage will support the child's acquisition of English B to support langauge development, family members should speak to the child in the language they are most comfortable using C if a child has a language disorder, a delay in learning English will best help the child achieve fluency D even without support, the child can be expected to be fluent in two years

B to support language development, family members should speak to the child in the langauge they are most comfortable using

Which of the following are subcortical degenerative dementias A. Alzheimer's disease B. Wilson's disease C. Parkinson's disease D. Progressive supranuclear palsy

B, C, D

A child with cerebral palsy typically presents with a host of impairments that can include: A. Dyskinesia B. Intellectual C. Sensory D. Orthopedic E. Myoclonus F. Socioemotional G. Communication

B, C, F, G

What would be the sound pressure level if the level from each source is 20 dB IL and the two sources are acting together? A. 20 dB SPL B. 23 dB SPL C. 26 dB SPL D. 40 dB SPL

B. 23 dB SPL

Constrictions in the vocal tract will raise or lower the formant frequency. How is the first formant frequency, which is also known as F1, lowered? A. A constriction near the velum B. A constriction near a volume velocity maximum C. A constriction near a pressure maximum D. A constriction in the pharynx

B. A constriction near a volume velocity maximum

Macy, 2-year-old girl, was referred to you by her mother because she was unintelligible. During Macy's oral examination, you discover a labioversion of her incisors. Macy's masseter muscle is evidently weak as she swallows and she has a disorganized method for creating a bolus. Macy also has an open bite. After consideration of all visible symptoms, you determine that this Macy has: A. Ankyloglossia B. A tongue thrust C. Class I Malocclusion D. Class II Malocclusion

B. A tongue thrust

After an articulation disorder has been diagnosed, the most appropriate sequence of therapy should be: A. Correct Production of the Sound in Specific Contexts B. Earlier Developmental Sounds C. Stimulability D. Sounds that Affect Intelligibility

C, A, B, D

An intervention to improve receptive vocabulary involves a computer program that presents three pictures on the screen and requests that the child point to the picture that the computer indicates via digitized speech. the childs internvetion goal is 80 percent correct responding. A response rate of 30 percent correct most likely idicates that the A software has been moderately effective in helping the child reach his goal B childs visual discrimination surpasses his auditory discrimination C childs reponses are essentially random D child is ready to progress to an on-screen array of four pictures

C, childs responses are random. 33 percent is at the level of chance

Siobhan, a 74-year-old female, is a new resident at the nursing home. After reading her case history, you learn that she recently has had a history of multiple strokes. In her case history, it states that her cognitive impairments coincided with her strokes. She is disoriented, confused, has trouble speaking or understanding speech, and has vision loss. Other symptoms include uncontrolled laughing and crying, problems with judgment and planning, and a reduced ability to pay attention. Her daughter reports that Siobhan appears to display symptoms of depression, noting that "ever since the strokes, Mom has a difficult time with simply getting up in the morning and she just doesn't seem to enjoy being around her family like before." In person, Siobhan shows loss of motivation and attention during various tasks. She appears to have some memory loss, but primarily struggles with executive functioning. Siobhan also presents with small-step gait and unsteadiness. She is prone to falls and is required to walk with a cane. Her regular nurses report that she has urinary urgency not explained by urologic disease along with emotional incontinence. Due to concerns expressed by her family, Siobhan is being evaluated to determine the presence of dementia. QuestionTopic: Dementia All of the following are probable etiologic subtypes for Siobhan's dementia, except: A. Hypoperfusion in border zones and granular cortical atrophy B. Multilacunar state C. Excess amount of neurofibrillary tangles D. Postischemic encephalopathy

C. Excess amount of neurofibrillary tangles Neurofibrillary tangles (NFTs) are aggregates of hyperphosphorylated tau protein that are most commonly known as a primary marker of Alzheimer's disease.

There are two broad levels of human communication: verbal communication and nonverbal communication. Verbal communication uses words as symbols to exchange ideas. Nonverbal communication includes many behaviors that communicate with or without the associated production of symbols. Which of the following is not an example of verbal communication? A. American Sign Language B. Written language C. Extralinguistic communication D. Pictographic communication

C. Extralinguistic communication

An audiologist is conducting a hearing test and sends an acoustic signal through a headphone at a level that is strong enough to obscure the tone heard in the opposite ear. Which of the following statements is not true about masking? A. Masking allows the clinician to eliminate the non-test ear from participation while measuring hearing thresholds for the test ear. B. The masking noise in the non-test ear must be enough to make any sound crossing the skull from the test ear inaudible. C. For testing hearing by bone conduction, masking is needed whenever the difference between the air-bone gap is 60 dB or more. D. The plateau method is a procedure used to determine how much masking is sufficient.

C. For testing hearing by bone conduction, masking is needed whenever the difference between the air-bone gap is 60 dB or more.

You are evaluating a 3-year-old child whose parents have concerns about his social aspects of communication, frequent echolalia, and perseverations. His parents mentioned that he frequently talks to himself, has anxiety, and displays hyperactive behavior. These concerns are evident throughout your evaluation. You notice that he has difficulty attending to tasks and has limited eye contact. Upon observation, his facial features are characterized by a high forehead, large jaw, and a poorly formed pinna. Following your evaluation, you refer this child and his family to a geneticist to rule out the possibility of a genetic syndrome. The patient most likely presents with: A. Down syndrome B. Pierre-Robin syndrome C. Fragile X syndrome D. Prader-Willi syndrome

C. Fragile X syndrome

When analyzing spectrograms on a spectrograph, there are different types of spectrograms that allow you to analyze your sample to the best of its ability. These different types of spectrograms are called wideband spectrograms and narrowband spectrograms. Which of the following characteristics is expressed better in a narrowband spectrogram? A. Short time window B. Good for measuring formant frequencies C. Good for showing and measuring harmonics D. Good for showing changes in articulatory movements

C. Good for showing and measuring harmonics

During the study in the radiology department, the clinician mentioned to her student intern that the following technique would result in widening of the vallecula space: A.Thermal stimulation B.Labial exercises C.Head down (chin tuck) D.Head back

C. Head down (chin tuck)

Debbie, a 42-year-old, was diagnosed with laryngeal cancer and has been noticing changes in her voice. Which of the following is the primary voice symptom of laryngeal cancer? A. Diplophonic B. Breathiness C. Hoarseness D. Strained

C. Hoarseness

You want to evaluate your client's ability to make rapidly alternating speech movements by using diadochokinesis (DDK) syllable rates. You evaluate how many repetitions of /pʌ/ the client can produce in 15 seconds. All of the following are true regarding DDK syllable rates except: A. There are two primary ways to obtain these measures. B. Before obtaining a client's rate, you should provide instructions for the tasks, model the target behaviors, and give the client time to practice. C. If the client stops or slows down intentionally before the allotted time is expired, you should take note of this, but the task does not need to be repeated. D. The DDK rate can help determine the severity of a communication disorder, its cause, and the best course of treatment.

C. If the client stops or slows down intentionally before the allotted time is expired, you should take note of this, but the task does not need to be repeated.

true? A. Myopathies have an affinity for lower motor neuron (LMN) cell bodies. B. Myopathies reflects mutations in genes that influence the suppression of tumors. C. Myopathies are not associated with sensory deficits or central nervous system (CNS) pathology. D. Myopathies are a group of disorders that involves regeneration of the stretch reflex.

C. Myopathies are not associated with sensory deficits or central nervous system (CNS) pathology. The myopathies are neuromuscular disorders in which the primary symptom is muscle weakness due to dysfunction of muscle fiber. Other symptoms of myopathy can include include muscle cramps, stiffness, and spasm.

Which of the following is a limitation of standardized speech-language tests? A. Generally exhaustive time required to administer them B. Lack of statistical norms C. Inadequate participant and response sampling D. Unnecessarily extensive testing of each individual skill sampled in the test

C. Inadequate participant and response sampling

What are the two forces needed for vibration? A. Mass and elasticity B. Mass and inertia C. Inertia and elasticity D. Weight and inertia

C. Inertia and elasticity

Speech-language pathologists in educational settings need to ask themselves questions that will guide curriculum-based language communication assessments. One question is "What communication abilities and tactics might the child obtain to become successful?" What can the clinician to do answer this question? A. Investigate how the student contributes to conversations. B. Observe the student in the classroom. C. Investigate what kinds of scaffolding might help the student. D. Collaborate with other educators to problem solve.

C. Investigate what kinds of scaffolding might help the student.

A 32-year-old patient was involved in a motor vehicle accident. He is a ventilator-dependent patient with a tracheostomy tube in place. Because the ventilator controls the respiratory cycle, the patient cannot lengthen exhalations; however, the physician recommends trial therapy for tolerance for swallowing. In preparing for feeding and swallowing, which of the following is recommended? A. Have the patient sit in the bed at a 120-degree angle before occluding the patient's tracheostomy. B. It is preferable to present food to the patient at the end of the exhalation phase of the respiratory cycle. C. It is preferable to present food to the patient at the beginning of the exhalation phase of the respiratory cycle. D. It is too dangerous to present food. Immediately recommend NPO.

C. It is preferable to present food to the patient at the beginning of the exhalation phase of the respiratory cycle.

As a speech-language pathologist, which of the following is not something you would treat if your client is diagnosed with cerebral palsy? A. Treatment of language disorders B. Treatment of articulation and phonological disorders C. Managing non-associated attentional problems D. Managing associated sensory problems

C. Managing non-associated attentional problems

Compensatory strategy training focuses on maximizing the skills of an individual with a traumatic brain injury (TBI) by modifying the environment and providing internal and external supports. This strategy capitalizes on intact skills to help an individual overcome deficits resulting from a TBI. Which of the following is an example of an external compensatory strategy that may be used to assist a client? A. Mneumonics B. Imagery C. Memory aids D. Association

C. Memory aids

Which of the following is not considered to be a communication deficit that may be present in individuals with apraxia of speech? A. High variability of speech errors, especially when repeating attempts B. Problems with spontaneous sequencing of movements required for speech C. More difficulty with vowels than consonants D. Metathetic errors

C. More difficulty with vowels than consonants

______________ causes damage to muscle fibers. A. Dysarthria B. Anterior horn cell disease C. Myopathy D. Spinal nerve disease

C. Myopathy

Statement: IDEA or Section 504 -A child is eligible under this law if he or she has any physical or mental condition that substantially limits a major life activity. 1. 2. -This law provides an eligible student with an IEPI E P. 3. 4. -A child must have one of 13 specified disabilities to receive services under this law. 5. 6. -This law prevents discrimination against persons with disabilities. 7. 8. -The Office of Special Education and Rehabilitative Services administers this law. 9. 10. -The Office for Civil Rights administers this law. 11. 12.

Correct Answer: 2, 3, 5, 7, 8, 9, 12 The correct answers are, in order, Section 504; IDEA; IDEA; IDEA and Section 504; IDEA; Section 504. Section 504 is correct because mental conditions include those conditions that interfere with academics, such as expressive and receptive language; and when a child may not qualify for an IEPI E P due to the mental condition, the child can receive a 504 plan. IDEAI D E A is correct because providing students with IEPsI E Ps falls under the province of IDEAI D E A. IDEAI D E A is correct because IDEA designates 13 different categories of eligibility for special education services. Both IDEAI D E A and Section 504 are correct because antidiscrimination laws protect students with disabilities and their right to public education. IDEAI D E A is correct because the Office of Special Education and Rehabilitative Services is responsible for administering IDEAI D E A regulations. Section 504 is correct because Section 504 of the ADAA D A and 504 Plans fall under the jurisdiction of civil rights law.

Christel and Sharon are 14-month-old fraternal twins. Christel has more intelligible words than her sister, although she talks far less frequently. Sharon speaks almost exclusively in running jargon that nobody understands. Their parents are worried about Sharon's development because her language skills appear to be behind those of her sister. Which of the following is an SLP'sS L P's best response to the parents' concern? • A.At Sharon's age, jargon is normal and may even continue for another three to four months. • B.Christel's language development is more abnormal because she should be beyond single words. • C.Both girls were exposed to the same language environments, so the relative lack of intelligible words indicates a language delay. • D.Jargon should not persist past the age of 12 months and thus Sharon may be presenting a language disorder.

Correct Answer: A Option (A) is correct. Jargon typically continues until around 18 months of age.

Which THREE of the following variables are considered risk factors for late language emergence? • A.Male gender • B.Access to print material • C.Low socioeconomic status • D.Moderately low birth weight • E.Exposure to rich and varied vocabulary

Correct Answer: A, C, D Options (A), (C), and (D) are correct. Being of male gender is a risk factor cited by both ASHAA S H A and researchers for late-talking toddlers. Having a low SESS E S is considered a risk factor by ASHAA S H A and researchers for late talking. Finally low birth weight is a risk factor as well.

Which TWO of the following statements about the results of a screening test are correct? • A.Sensitivity is the proportion of people who fail the screening test who are aspirators • B.Specificity is the proportion of people who pass the screening test who are aspirators • C.Sensitivity is the proportion of people who pass the screening test who are not aspirators • D.Specificity is the proportion of people who pass the screening test who are not aspirators

Correct Answer: A, D Options (A) and (D) are correct. The screening test's sensitivity is the proportion of patients who aspirated and failed the screen (top cell in the first column) to the total of all patients who aspirated (total of the first column). The screening test's specificity is the proportion of patients who did not aspirate and passed the screen (bottom cell in the second column) to the total of all patients who did not aspirate on the VFSSV F S S (total of second column).

A 55-year-old female presents for a speech evaluation because of concerns with recent changes in the clarity of her speech. She has also noticed occasional coughing when drinking, and her friends have commented that her voice sounds different. A recent visit to her primary care doctor could not determine the cause of her speech changes or coughing. Select the THREE most important factors for the SLPS L P to explore during the patient interview. • A.Difficulties with hearing, vision, and fine or gross motor skills • B.History of a psychological or psychiatric disorder • C.Employment history and current stressors in her life • D.Onset and duration of her speech changes • E.Consistency of the symptoms

Correct Answer: A, D, E Options (A), (D), and (E) are correct. This patient scenario is strongly suggestive of a late-onset or acquired neurological disorder based on the combination of speech changes across multiple subsystems and swallowing problems. In this case, the most important information for the SLPS L P to obtain includes whether changes in other motor skills and vision and hearing accompany the changes in the duration, onset, course, and consistency of the changes in swallowing, voice, and resonance. This would help the SLPS L P determine the most important procedures to include in the evaluation and referrals regarding the etiology of the symptoms.

Why would stroboscopy be used as part of a voice treatment program? Comparison to baseline Treatment termination Not needed Change treatment

Correct answer: Comparison to baseline Explanation: Speech-language pathologists and otolaryngologists can repeat stroboscopic evaluations periodically, compare with previous results and determine progress being made in enabling the patient to achieve their "optimal" voice.

What are some of the cognitive and functional disabilities found in children with Fetal Alcohol Syndrome? Rigid muscle tone, poor attending skills with normal language development. Poor language skills, even though cognitive development is usually advanced. Poor memory, attention deficits and impulsivity. Hearing loss and low muscle tone.

Correct answer: Poor memory, attention deficits and impulsivity. Explanation: The main effect of Fetal Alcohol Syndrome is permanent damage to the central nervous system. Developing brain cells and structures are underdeveloped or malformed by prenatal alcohol exposure. This can create an array of primary cognitive and functional disabilities including poor memory, attention deficits, impulsive behavior, and poor cause-effect reasoning. The risk of brain damage exists during each trimester since the fetal brain develops throughout the entire pregnancy.

You have a 16-year-old male on your caseload who has not yet developed facial hair and seems overall underdeveloped. He has expressive language disorder and has difficulty making friends due to social pragmatic impairments. You suspect that this young man presents with which genetic syndrome? Down Syndrome Klinefelter's syndrome Prader-Willi Syndrome Cerebral Palsy

Klinefelter's syndrome Explanation: Klinefelter's syndrome is a condition that occurs in males who have an extra X chromosome in most of their cells. The syndrome can affect different stages of physical, language and social development. They may have trouble using language to express themselves. They may also present with social pragmatic difficulties as well. The most common symptom is infertility. Because they often don't make as much of the male hormone testosterone as other boys, teenagers with Klinefelter's syndrome may have less facial and body hair and may be less muscular than other boys.

Which articulation therapy approach emphasizes both the syllable as the basic unit of speech and the concept of phonetic environment?

McDonald's sensory-motor approach

Which of the following actions will most effectively control the problem of overreferral in school screening programs that use impedance/immittance measurements? A.Obtaining the measurements in a professional sound-insulated room B.Including 500 Hz in the audiometric screening procedure C.Retesting immediately those who did not pass the first screening D.Waiting three to five weeks to retest those who did not pass the first screening

Option (D) is correct. Some children may have a temporary problem due to a cold or ear infection, which may resolve in a few weeks.

Some studies have indicated that this improvement may be related to an increase of prosodic variation. Studies have reported an increase of fundamental frequency (F0) variation with increased loudness, but there has been no examination of the relation of loudness manipulation to specific prosodic variables that are known to aid a listener in parsing out meaningful information. This study examined the relation of vocal loudness production to selected acoustic variables known to inform listeners of phrase and sentence boundaries: specifically, F0 declination and final-word lengthening. Ten young, healthy women were audio-recorded while they read aloud a paragraph at what each considered normal loudness, twice-normal loudness, and half-normal loudness. Results showed that there was a statistically significant increase of F0 declination, brought about by a higher resetting of F0 at the beginning of a sentence and an increase of final-word lengthening from the half-normal loudness condition to the twice-normal loudness condition. Question: Which of the following represent(s) the independent variable or variables used in the Watson and Hughes study? A.Prosody of dysarthric speech B.F0 declination and final-word lengthening C.Vocal loudness D.Speech intelligibility and communicative effectiveness

Option (C) is correct. The researchers manipulated vocal loudness to determine its effect on prosodic F0 and durational variables.

Which of the following groups of phonemes can best be described as the Six-Sound Test by Ling? • A. /u/u, as in the word ooze, /e/e, as in the word way, /o/o, as in the word own, /a/a, as in the word path, /æ/latin small letter a e, as in the word hat, /ə/latin small letter schwa, as in the word again • B. /f/, /s/, /θ/, /ʃ/, /z/, /h/f, as in the word fast, s, as in the word sit, greek small letter theta, as in the word thumb, latin small letter esh, as in the word she, z, as in the word zebra, h, as in the word hot • C. /m/, /a/, /u/, /i/, /s/, /ʃ /m, as in the word mat, a, as in the word path, u, as in the word ooze, i, as in the word he, s, as in the word sit, latin small letter esh, as in the word she • D. /d/, /g/, /e/, /o/, /s/, /ʃ/d, as in the word dog, g, as in the word go, e, as in the word way, o, as in the word own, s, as in the word sit, latin small letter esh, as in the word she

Option (C) is correct. The six phonemes that are part of Ling's Six-Sound Test are /m/, /a/, /u/, /i/, /s/, /ʃ /m, as in the word mat, a, as in the word path, u, as in the word ooze, i, as in the word he, s, as in the word sit, latin small letter esh, as in the word she. The Ling Six Sound Check is an easy-to-use tool that evaluates how well children are hearing with their cochlear implant and/or hearing aid.

87. Top of Form A 6-year-old child produces [t] for /s/, [d] for /z/, [p] for /f/, and [b] for /v/. Intervention for this problem would target language at the level of A.morphology B.syntax C.phonology D.semantics

Option (C) is correct. The speech patterns described in the question stem all pertain to speech sound production. Phonology is the aspect of language that deals with speech sound production.

Which of the following therapeutic techniques is most appropriate to treat hyperfunctional voice disorders? • A.Pushing, pulling, and isometrics • B.Lee Silverman Voice Treatment • C.Semi-occluded vocal tract exercises • D.Coughing and throat clearing

Option (C) is correct. The use of semi-occluded vocal tract exercises, whether use of straws, resonance tubes, lip trills, or fricatives, has demonstrated across multiple studies in the literature to optimize glottal configuration to reduce the impact of glottal closure and reduce potential phonotrauma and to optimize glottal closure to a slightly abducted position, also to reduce the potential for phonotrauma often found in hyperfunctional voice. These exercises have also demonstrated the ability to reduce voice fatigue, a classic complaint of those with hyperfunction, by optimizing glottal impedances in the vocal tract. The Lee Silverman Voice Treatment (LSVT) is a program designed to increase vocal intensity in patients with Parkinson disease.

127. Top of Form A videofluoroscopic study of a client with dysphagia revealed post-swallow vallecular residue occupying more than 50 percent vallecular height. Which of the following is the most likely overt symptom that the client will experience? A.Watery eyes during swallowing B.Oral pocketing of foods C.Coughing after swallowing D.Esophageal reflux

Option (C) is correct. The valleculae are depressions that lie lateral to the median epiglottal folds. Pooling of liquids in the valleculae gives a person the feeling that there is material remaining in the respiratory pathway, so coughing would be a natural reaction to expect in this case.

131. Top of Form A third-party reimburser asks the speech-language pathologist to demonstrate in a diagnostic statement that a child's communication problems have a physiological cause. Which of the following observations, if included in the statement, would best satisfy the request? A.The child's dentition is not yet fully developed but is within normal limits for a child of that age. B.The child has developmental delay, exhibiting speech that is not appropriate for a child of that age. C.The child demonstrates a motor-speech disorder and is unable to perform voluntarily the oral movements required for speech production. D.The child exhibits stridency deletion, consonant cluster reduction, stopping, and fronting. Bottom of Form Correct Answer: C

Option (C) is correct. The wording represents a well-written diagnostic statement for a child with communication problems that are physiological, or functional, in nature. None of the other answer choices demonstrates the presence of a physiological problem.

When completing a videofluoroscopic swallow study on a geriatric patient, which of the following findings is most appropriately considered typical development? • A.Vallecular retention of more than 50 percent of vallecular height • B.Nasopharyngeal regurgitation • C.Liquid bolus enters pharynx before hyolaryngeal excursion begins • D.Oral pocketing

Option (C) is correct. There is known variability in initiation of pharyngeal swallow, notably in the geriatric population. Research indicates that the initiation of the pharyngeal is not necessarily at the ramus of the mandible in the geriatric patient population.

63. Top of Form Which of the following muscles produces the opposing action to those that produce velopharyngeal closure? A.Musculus uvulae B.Levator veli palatini C.Palatoglossus D.Stylopharyngeus

Option (C) is correct. Velopharyngeal closure is largely produced by soft palate elevation, and the only muscle in the list that produces soft palate depression (the opposite of soft palate elevation) is the palatoglossus.

When completing an oral mechanism examination on a patient presenting with stroke-like symptoms, which of the following findings would be most likely to predict aspiration during an instrumental swallow evaluation? • A.Inability to swallow on command • B.Facial droop • C.Absent gag reflex • D.Breathy phonation

Option (D) is correct. Studies have demonstrated that breathy phonation would indicate possible vocal fold and/or pharyngeal paralysis.

64. Top of Form Michael is a 32-month-old boy who has been receiving early intervention services over the past ten months for delayed speech and expressive-language development. Although his birth was reportedly unremarkable, Michael does have a history of recurrent otitis media with effusion. His parents described him as having been a "well-behaved and quiet baby." When Michael began receiving services, he communicated mainly through gestures and crude vocalizations. An open resting mouth position with slight tongue protrusion was sometimes noted. However, his receptive-language skills were found to be age appropriate and he showed no oral motor deficits during feeding. Michael's expressive-language skills have shown some progress since he began working with the speech-language pathologist, but he remains poorly intelligible. Michael's imitation of tongue, lip, and jaw movements is characterized by inconsistent groping and errors of sequencing not observed in his spontaneous oral movements. Michael has an age-appropriate vocabulary and produces utterances of up to five words. Articulation errors, especially metathesis of phones and syllables, increase as his utterance length increases. Michael's intelligibility is greatest at the single-word level. Automatic speech and highly familiar utterances are much more intelligible than his imitated productions. Answer the question below by clicking on the correct response. Question: As part of ongoing assessment, the SLP's most appropriate action is to have Michael's parents consult with his primary-care provider for referral to A.a special educator B.a neurologist C.a psychologist D.an otolaryngologist

Option (D) is correct. Michael's history of recurrent middle ear infections, an otolarygologist would be able to clinically manage otitis media and make provisions to obtain an audiological assessment.

A 12-year-old native speaker of Spanish who has been studying English as a second language for three years is most likely to do which of the following when speaking English in casual conversation with teachers at school? A.Use the auxiliary "have" in place of "be" in progressive tenses B.Use incorrect word order within prepositional phrases C.Use conjunctions in place of prepositions D.Use multiple negation improperly

Option (D) is correct. Multiple negation is a grammatical feature of Spanish but not of Standard English. The contrast between the two languages can cause multiple negation to persist as a speaker attempts to learn Standard English.

During the course of treatment, as a child manipulates various toys in symbolic play, the clinician vocalizes the child's actions (for example, "You're putting the pan on the stove."). The intervention technique used by the clinician is identified as • A.the mand model • B.reauditorization • C.self-talk • D.parallel talk

Option (D) is correct. Parallel talk is a technique a clinician uses when talking aloud about what the child is seeing, hearing, or doing. In mand mandmodelling, sometimes known as child-cued modelling, the teacher or caregiver observes the focus of the child's interest (e.g., a ball) and models the correct verbalisation (e.g., "that's a ball").

Which of the following is a type of perturbation that can be measured to determine the amount of noise in the voice? A.Changes in the frequency range between F1 and F2 over time B.Changes in the frequency range between F2 and F3 over time C.F3 cycle-to-cycle variations in sound energy over time D.F0 cycle-to-cycle variations in sound energy over time

Option (D) is correct. Perturbation is a disturbance in the quality of the laryngeal tone, or fundamental frequency, of the voice.

Elisions and transpositions are referred to as phoneme A.additions B.blending C.deletions D.manipulations

Option (D) is correct. Phoneme manipulations change or modify the individual sounds in words. Elisions are the omissions of sounds in spoken words, and transpositions occur when the order of sounds in a word is switched.

3. Top of Form Primary motor innervation to the larynx and velum is provided by which cranial nerve? A.V B.VII C.IX D.X

Option (D) is correct. Primary innervation to the larynx and velum is provided by cranial nerve X, the vagus nerve. The other answer choices identify cranial nerves that are not primarily involved in motor innervation to the larynx and velum.

Which of the following is the most important acoustic cue that distinguishes between an unreleased final /p/ and an unreleased final /b/, as in "cap" versus "cab"? A.Locus frequency of burst B.Voice onset time C.Vocal fundamental frequency D.Duration of the preceding vowel

Option (D) is correct. Research shows that vowel duration influences a listener's perception of voicing. Vowels that precede unreleased voiced stop consonants are as much as 1.5 times as long as vowels that precede voiceless stops.

An SLPS L P is working with a sixth-grade student who is having difficulty comprehending written text. Which of the following strategies would be most effective for the SLPS L P to use to target this difficulty? • A.Creating a log in which unfamiliar sight words are written down for later practice • B.Developing the student's ability to chunk multisyllabic words into smaller units • C.Selecting words from a text that can be used as both spelling and sight words • D.Using semantic networks in which ideas are displayed in connected clusters

Option (D) is correct. Semantic networks improve reading comprehension by showing students not only how vocabulary words are related to each other in a conceptual hierarchy, but how ideas in texts are organized in an associative way.

A 24 month old who was screened for autism spectrum disorder (ASDA S D) attends a twice-weekly early intervention program to address developmental delays. Which of the following is the best way to determine the communication goals for the child? • A.Aligning with developmental norms for communication behaviors typical of 24 month olds • B.Waiting to address communication goals only after behavioral issues have been remediated • C.Aligning with goals of other children in the program to heighten intensity of intervention efforts • D.Setting goals that address communication challenges within daily activities and routines

Option (D) is correct. Setting goals that address communication challenges within daily activities and routines will best help determine appropriate communication goals for a child with ASDA S D.

Which of the following interventions could be deployed while the patient is swallowing to increase intrabolus pressure? • A.Tongue tether maneuver • B.Mendelsohn maneuver • C.Breath-holding with Valsalva maneuver • D.Effortful swallow maneuver

Option (D) is correct. Several studies have shown that when correctly performed, the effortful swallow maneuver increases intrabolus pressure during the swallow.

Posterior portion of the insula

Part of the cerebral cortex, significant role in pain perception, social engagement, empathy, emotions, and numerous other vital functions,

Mendelsohn Maneuver

Patients are instructed to swallow, to hold the swallow for 2-3 seconds, and then to complete the swallow and relax when the pharynx is in the uppermost stage. Repeatedly swallowing and washing food through the pharynx may be helpful to patients who have excessive residue in the pharynx after the swallow.

Place the following aspects of phonological awareness in developmental order starting with the earliest skill to emerge. Listing words that start with the same sound Recognizing words that rhyme Counting syllables in single words Creating words by blending onset and rime

Recognizing words that rhyme is first because this occurs between 30 and 36 months of age. Counting syllables in single words is next because this occurs by 4 years of age. Blending words into rime is third because this occurs near 5 years of age. Listing words that start with the same sound is last because this occurs between 6 and 7 years of age. Any other order is incorrect because phonological awareness builds on itself and it is necessary to master each area prior to moving on to the next.

Which term describes an abnormal increase in the perceived loudness of sound when it is presented with linear increases in intensity? A. Recruitment B. Air conduction C. Rarefaction D. Displacement

Recruitment

A student you evaluate at your clinic presents with speech and language delay, poor attending skills, delayed cognitive development and low muscle tone. Her mother reports that this child had difficulty feeding as an infant, had chronic middle ear infections, does not sleep well and has sudden and severe tantrums. You believe that this child could have which genetic syndrome? Apraxia. Smith-Magenis Syndrome. Rett Syndrome. Fragile X Syndrome.

Smith-Magenis Syndrome. Explanation: Smith-Magenis syndrome (or SMS) is a chromosomal disorder characterized by a recognizable pattern of physical, behavioral, and developmental features. Common features include: characteristic facial appearance, infant feeding problems, low muscle tone, developmental delay, variable levels of intellectual disability, early speech/language delay, middle ear problems, skeletal anomalies and decreased sensitivity to pain. The syndrome also includes a distinct pattern of behavioral features characterized by chronic sleep disturbances, arm hugging/hand squeezing, hyperactivity and attention problems, prolonged tantrums, sudden mood changes and/or explosive outbursts and self-injurious behaviors.

SCERTS

Social Communication, Emotional Regulation, and Transactional Support

A clinician is a member of a cleft palate and craniofacial team that asks her to conduct an objective assessment of a 6-year-old child's velopharyngeal mechanism. The clinician decides to do nasopharyngoscopy, in which the nasopharyngoscope is passed through the middle meatus and back to the area of velopharyngeal closure. What will this procedure enable the clinician to observe? A. The child's posterior and lateral pharyngeal walls, as well as the nasal aspect of the velum and the adenoid pad as the child produces sentences B. The child's posterior and lateral pharyngeal walls, as well as the adenoid pad as the child sustains /a/ C. The child's nasal aspect of the velum and the adenoid pad as the child produces CVC words D. The child's posterior and lateral pharyngeal walls, as well as the nasal aspect of the velum and the adenoid pad as the child produces nasals /n/ and /m/

The child's posterior and lateral pharyngeal walls, as well as the nasal aspect of the velum and the adenoid pad as the child produces sentences

Match spastic, ataxic, flaccid, hyperkinetic Practicing relaxation exercises, Using rhythmic or metered cueing, Performing pushing-pulling exercises, and Implementing sensory tricks.

The correct matches are Spastic - Practicing relaxation exercises, Ataxic - Using rhythmic or metered cueing, Flaccid - Performing pushing-pulling exercises, and Hyperkinetic - Implementing sensory tricks. People with spastic dysarthria need to practice relaxation exercises to reduce muscle tone and increase flexibility. Those with ataxic dysarthria benefit from rhythmic or metered cueing due to inability to adequately pace their speech production. Flaccid dysarthria is effectively treated with pushing and pulling exercises to help strengthen muscles and increase tone. People with hyperkinetic dysarthria benefit from sensory tricks to reduce involuntary movements. Any other order is incorrect because the treatment strategies cannot interchangeably treat other types of dysarthria and would be ineffective if applied to the wrong dysarthria type.

• A. Obtain measure of maximum tongue-press pressure generation • B. Calculate an exercise target that is below the patient's maximum pressure generation, and begin progressive resistance exercises • D. Increase exercise target and continue three sets of 10 repetitions per day, three days per week, of exercises at the treatment target • C. Perform VFSSV F S S to evaluate efficacy of tongue-press progressive resistive exercise program on swallow impairments

The correct order is B, D, A, C (2), (4), (1), (3). The above scenario is not specific to tonguepress exercises. It uses tongue-press exercises as a method of treatment to illustrate a well-designed, evidence-based intervention program of rehabilitation.

Which of the following is the most accurate statement regarding the word initial consonant? A. the high-amplitude concentration of aperiodic energy at 4 khz suggests that the initial consonant is a voiceless sibilant fricative. B. The low-amplitude concentration of periodic energy suggests that the initial consonant is a voiced sibilant fricative. C. the presence of prevoicing before the release burst suggests that the initial consonant is a voiced affricate. D. The low-amplitude diffuse distribution of aperiodic energy suggests that the initial consoant is a voiceless nonsibilant fricative.

The high amplitude concentration of aperioidic energy at 4khz suggests that the initial consonant is a voiceless sibilant fricative

"By next week, we will have completed all four reports." The italicized part of the previous sentence is an example of which grammatical category? Number Tense Case Person

The italicized phrase is an example of the future perfect tense of the verb. Tense is one grammatical category. Other grammatical categories include (but are not limited to) person, that is, first person = I/we/us; second person = you; third person = he, she, it, them. Another is case, that is, nominative/subjective, where a noun is the subject, accusative/objective where a noun is the object, or possessive/genitive indicating possession, for example, "John's hat". And the last, here is number, for example, singular (e.g., "hat") or plural (e.g., "hats").

What determines the formant structure of speech sounds, particularly vowels? The frequency of the ninth harmonic. The length and shape of the vocal tract. The day of the week. The size of the vocal folds.

The length and shape of the vocal tract.

The characteristic acoustic feature of fricative consonants is: The silent interval. The noise energy. The vibration of the vocal folds. The F2 transitions.

The noise energy.

Syllable-initial voiced and voiceless stop consonants are differentiated by: Slope of the formant transitions. Silence. Voice onset time. Duration of the noise burst.

Voice onset time.

The energy maxima of utterances is generally defined by the: Consonants. Vowels. Phrasal boundaries. None of the above.

Vowels.

Tertiary components

are often external to the system itself but affect long-term use and ongoing success with the system (e.g., switches, portability, mounts, training and support

Primary components

are those that perform the functions of natural language and have the greatest impact on communication performance (e.g., symbols, vocabulary, methods of utterance generation).

Secondary components

relate to the way the individual uses and interacts with the system (e.g., user interface, selection method and output).

29. Place the examples of assessment tasks with the type of attention that is primarily being evaluated. -sustained, working, alternating, selective • ----Listening to a list of spoken words for a target word • -----Focusing on hearing a person speaking while the television is on • -----Mentally solving a complex math problem • -----Sorting playing cards by color, then by number, and then by color again

• Listening to a list of spoken words for a target word - selective attention • Focusing on hearing a person speaking while the television is on -sustained attention • Mentally solving a complex math problem - working memory • Sorting playing cards by color, then by number, and then by color again - alternating attention Any other order is incorrect because as Sohlberg and Mateer (2010) lay out the types of attention, their descriptions match the assessment tasks described below. That is, sustained attention involves maintaining attention over a period of time; selective attention involves a simple short-term task such as listening for specific words in a list; working memory involves holding information in your head and manipulating that information (math problem); and alternating attention involves switching

An 82-year-old female patient is admitted to a skilled nursing facility following an acute hospital stay. Her diagnoses include urinary tract infection, frequent falls with subsequent hip fracture, and chronic obstructive pulmonary disease (COPD). No surgery was required for the hip fracture. Before admission to the hospital, she was living independently and able to complete all activities of daily living (ADLs) and instrumental activities of daily living (IADLs) on her own. She is referred to the facility SLP for cognitive screening because she has difficulty carrying over new information, confusion regarding weight-bearing status, and difficulty processing directions. The SLP administers the Montreal Cognitive Assessment, and the patient scores 17/30. The patient's main deficits are in the areas of short-term memory, executive functioning, and planning. After the patient is appropriately treated both medically and therapeutically, cognitive deficits remain and the interdisciplinary team does not recommend that the patient return home independently. The patient's family is frustrated with this recommendation and seeks input from other sources. 117. Which of the following treatment procedures is most appropriate for the patient? • Engaging in spaced retrieval • Naming divergent items • Following specific directions • Completing word searches

• Option (A) is correct. The patient's goal is to return to prior level of function (PLOF). The patient was able to live independently before her admission, so spaced retrieval techniques will help her to return to her previous level of memory ability or at least teach her compensatory strategies to assist with her memory loss, since she does not yet have a diagnosis of dementia.

65. As part of a study, the speech of several subjects with Wilson's disease is evaluated using an acoustic analysis system. The same subjects are assessed again, but with a different system from another manufacturer. Which of the following can be assessed by comparing the two data sets? • Content validity • Concurrent validity • Test-retest reliability • Interjudge reliability

• Option (B) is correct. Concurrent validity refers to the extent to which the results of a particular test, or measurement, correspond to those of a previously established measurement for the same construct.

52. Following placement of a tracheostomy tube, a patient recovering from cardiothoracic surgery is successfully weaned from mechanical ventilation. A day later the SLP receives a consult to assess patient candidacy for using a one-way tracheostomy valve. Which of the following observations is the most important contraindication for safe and successful patient tolerance of the one-way valve? • Oxygen saturation below 95% • Ability of patient to pass air to the oral cavity while exhaling with the tube cannula occluded by the SLP • Patient ability to expectorate lung secretions without suctioning • Patient tolerance of the deflated cuff

• Option (B) is correct. Inability to pass air to the oral cavity with expiratory occlusion represents an upper-airway obstruction, which is a contraindication for use of a one-way tracheostomy valve.

1. For a patient with potential pharyngeal phase dysphagia and pooling of secretions following open-heart surgery with suspected recurrent laryngeal nerve damage, which of the following instrumental examinations for swallowing provides the most direct view for evaluating the patient based on history and suspected difficulties? Upper gastrointestinal (GIG I) series Fiberoptic endoscopic evaluation of swallowing (FEES) Videofluoroscopic swallowing study (VFSS) Videostroboscopic examination of vocal folds

• Option (B) is correct. Recurrent laryngeal nerve damage during open-heart surgery would only involve the left vocal fold. This swallowing instrumental examination allows for both assessment for pharyngeal phase dysphagia, by presenting foods and liquids during the assessment procedure, and a direct view of the functioning of both vocal folds, as well as determining the pooling of secretions.

127. During an examination of the oral peripheral mechanism of an adult who has had a right hemisphere stroke in the territory of the middle cerebral artery, testing for lingual motor function reveals protrusion of the tongue to the left of midline. Which of the following is true? • The patient has a lesion affecting the left hypoglossal nucleus. • The patient exhibits unilateral left lingual weakness. • Left vocal fold paralysis should be expected because of the site of the lesion. • The right hypoglossal nerve is damaged.

• Option (B) is correct. The right side of the tongue is stronger and forces (or pushes) the tongue to the left. The imbalance in strength causes the tongue to deviate from midline to the left upon protrusion, hence left lingual weakness.

67. A 7-month-old infant presents to an SLP for a swallow evaluation prompted by parental concerns related to the child coughing and choking frequently when swallowing thin liquids. The parents have attempted to feed the child small amounts of pureed foods, but the child spits them out. Which of the following developmental milestones is a typically developing 7 month old most likely to demonstrate during a swallow evaluation? • Showing the ability to manipulate food • Creating vertical jaw movements • Using the rooting reflex when food is seen • Engaging tongue elevation patterns

• Option (B) is correct. Using jaw movements to initiate chewing begins to emerge around 5 to 7 months, and it would be appropriate for the SLP to assess whether the child has begun to use this skill.

105. A 23-year-old client recently completed an intensive 3-week-long summer program for stuttering and now exhibits stuttering symptoms on fewer than 3 percent of syllables during both in-clinic conversations with the SLP and beyond-clinic conversations with family members. The SLP is concerned that the client might relapse now that intervention has ended and wishes to enact a plan to help the client maintain fluency gains. Which of the following plans is the most likely to result in maintenance of the fluency gains? • Recommending that the client participate in an annual intensive review of stuttering management skills • Recommending that the client practice fluency management skills during monthly telephone calls with the SLP • Ensuring the client understands the need for continued use of stuttering management techniques and referring to a local SLP for ongoing treatment • Scheduling the client for reenrollment in the intensive 3-week-long intervention program

• Option (C) is correct. Continuing to regularly participate in scheduled intervention activities under the direction or supervision of an SLP is essential to maintaining fluency gains. It provides a client with the opportunity to address ongoing fluency challenges or new ones that may arise after completion of the intensive intervention program. This type of continued treatment does not necessarily imply that the client must participate in ongoing weekly or intensive intervention.


Set pelajaran terkait

Tostevin World since 1500 S1 Final: Terms

View Set

Personal Finance: Practice midterm exam (Ch. 7,8,&9)

View Set

Soil and Fertilizer Chapter 2 review

View Set

Histology and Cell Biology of blood vessels and lymphatics

View Set